Anda di halaman 1dari 208

UNIVERSIDAD

TCNICA
FEDERICO
SANTA MARA
Departamento de Electricidad

Conversin
Electromecnica
de Energa
J.Mller

2001

NDICE

1.

EL CIRCUITO MAGNTICO

1-5

1.1

Introduccin

1-5

1.2

Prototipo y aproximaciones

1-6

1.3

Circuitos magnticos

1-10

1.4

Imanes permanentes

1-14

2.

EL REACTOR

2-17

2.1

Introduccin

2-17

2.2
Efectos fsicos en el reactor
2.2.1
Dispersin magntica
2.2.2
Prdidas en el fierro
2.2.3
Corriente magnetizante compleja
2.2.4
Prdidas en el cobre

2-20
2-20
2-21
2-26
2-27

2.3
Circuito equivalente.
2.3.1
Circuitos electromagnticos.
2.3.2
Circuito equivalente del reactor

2-29
2-29
2-33

2.4

2-35

3.

Tensin inducida

EL TRANSFORMADOR

3-38

3.1

Introduccin

3-38

3.2

Dispersin inductiva y esquema de acoplamiento inductivo

3-39

3.3
El transformador de potencia.
3.3.1
Circuito equivalente
3.3.2
Diagrama fasorial
3.3.3
Funcionamiento en vaco
3.3.4
Funcionamiento en cortocircuito estacionario
3.3.5
Funcionamiento con carga

3-42
3-43
3-47
3-51
3-51
3-54

4.

DEVANADOS

4-59

4.1

Introduccin

4-59

4.2
Corrientes y campo magntico en el entrehierro
4.2.1
Bobinas acortadas, el factor de cuerda
4.2.2
Bobinas distribuidas, factor de zona
4.2.3
Devanados de corriente alterna
4.2.4
Campo giratorio mediante devanado trifsico
4.2.5
La distribucin de induccin en el entrehierro

4-60
4-66
4-68
4-70
4-73
4-76

4.3
Tensin inducida en un devanado
4.3.1
Tensin inducida en una bobina de paso completo
4.3.2
Tensin inducida en una bobina de paso acortado
4.3.3
Tensin inducida en un grupo de bobinas
4.3.4
Tensin inducida en un devanado de corriente continua

4-77
4-79
4-82
4-82
4-84

4.4

4-86

5.

Inductancias propias y mutuas de devanados

FUERZAS ELECTROMAGNTICAS

5-89

5.1

Introduccin

5-89

5.2

Fuerza y energa, una visin sistmica

5-90

5.3
Transductores de movimiento limitado
5.3.1
Torque de reluctancia
5.3.2
Torque de excitacin

5-93
5-95
5-98

5.4

Mquinas rotatorias, conversin continua de energa

5-101

5.5

Resumen

5-105

6.

MQUINA DE CORRIENTE CONTINUA

6-106

6.1

Introduccin

6-106

6.2

Caractersticas constructivas

6-107

6.3

Principio de funcionamiento

6-109

6.4

Ecuaciones de equilibrio elctricas

6-111

6.5

Ecuacin de equilibrio mecnica

6-115

6.6
Funcionamiento estacionario
6.6.1
Distribucin del campo en el entrehierro
6.6.2
Efecto desmagnetizante de la reaccin de armadura
6.6.3
Autoexcitacin
6.6.4
Conmutacin
6.6.5
Caractersticas estacionarias como generador
6.6.6
Caractersticas estacionarias como motor

7.

MQUINA SINCRNICA

6-116
6-117
6-118
6-121
6-122
6-125
6-128

7-132

7.1

Introduccin

7-132

7.2

Caractersticas constructivas

7-133

7.3

Principio de funcionamiento

7-134

7.4
Ecuaciones de equilibrio elctricas
7.4.1
Circuito equivalente por fase
7.4.2
Efecto de la saturacin
7.4.3
Diagrama fasorial

7-135
7-140
7-141
7-142

7.5

7-146

Potencia y momento

7.6
Condiciones de funcionamiento especiales
7.6.1
Cortocircuito estacionario
7.6.2
Carga reactiva inductiva pura

7-150
7-150
7-152

7.7

7-153

Determinacin experimental de la reactancia sincrnica

7.8
Funcionamiento en red infinita
7.8.1
Variacin de la excitacin
7.8.2
Variacin del momento
7.8.3
Lugar geomtrico de la corriente

8.

MQUINA ASINCRNICA

7-155
7-157
7-157
7-158

8-161

8.1

Introduccin

8-161

8.2

Caractersticas constructivas

8-162

8.3

Principio de funcionamiento

8-164

8.4

Ecuaciones de equilibrio elctricas

8-166

8.5

Circuito equivalente y diagrama fasorial

8-171

8.6

Potencia y momento

8-175

9.

EJERCICIOS

9-180

1. El circuito magntico

1.1

Introduccin

Desde los tiempos de Oersted (1820) se sabe que corrientes elctricas producen
campos magnticos. Esta relacin fundamental se expresa analticamente en la ley de
Ampere:
r

H ds = I

(1.1.1)

que recoge la evidencia emprica que, a lo largo de una trayectoria cerrada rs


cualquiera, la integral de la componente de la intensidad del campo magntico H
paralela al camino de integracin es igual a la totalidad de la corriente I abrazada por
ese camino de integracin.
La relacin (1.1.1) es completamente general e independiente de las caractersticas
del medio que atraviesa el camino de integracin cerrado. Mediante ella se rpuede
calcular la corriente abrazada por ste si se conoce la distribucin espacial de H .
En la prctica es ms comn el problema inverso, res decir, normalmente existe la
necesidad de determinar la distribucin espacial de H creada por una distribucin de
corrientes conocida.
La solucin de este problema requiere de un despliegue matemtico considerable y,
an as, slo es posible encontrar soluciones analticas rigurosas para medios
isotrpicos de geometras muy simples. Sin embargo, estos mtodos forman la base
para algortmos que permiten lograr soluciones numricas an para las situaciones
ms complejas.
Si bien los programas para el clculo numrico de campos dan respuestas
cuantitativas satisfactorias para situaciones especficas, estas, por su naturaleza, no
permiten una visin global y subsiste la necesidad de contar con soluciones analticas,
aunque estas sean aproximadas, que permitan apreciar la influencia de los
parmetros sobre la solucin.
Las formas geomtricas y las caractersticas de los materiales usados en la mayor
parte de los dispositivos electromagnticos prcticos, como los transformadores y las
mquinas elctricas rotatorias, permiten simplificar el problema y formular soluciones
analticas aproximadas.
Para concretar estas ideas y entender la naturaleza de las aproximaciones necesarias
se analiza primeramente el caso de una bobina toroidal.

1-6

captulo 1 : circuito magntico

1.2

Prototipo y aproximaciones

Considrese un solenoide de seccin circular formado por N vueltas de alambre de


cobre aislado, uniformemente distribuidas, doblado de manera que sus dos secciones
extremas se toquen, dejando en el interior un espacio toroidal (figura 1.2.1).
espiras

ri

re
seccin q

Figura 1.2.1 Electroimn toroidal


Si por la bobina circula una corriente i, crear en el interior del toroide un campo
magntico cuyas lneas de fuerza, por consideraciones de simetra, sern
necesariamente circunferencias concntricas.
Tambin es la simetra lar que permite concluir que a lo largo de una lnea de fuerza
de radio r el mdulo de H debe permanecer constante, ya que todos los puntos sobre
esa circunferencia son equivalentes en cuanto a su relacin con la distribucin de
corrientes.
Considerando estos hechos, que nacen de la geometra y de la distribucin de
r
corrientes considerada, la evaluacin de la integral en (1.1.1) es inmediata, ya que H
es tangente a la circunferencia. Para el espacio interior del toroide rige entonces:
H( r ) =

iN
2 r

(1.2.1)

Para el espacio exterior vale H( r ) = 0 , ya que la corriente total abrazada por un


camino de integracin concntrico con el toroide es cero y cualquier punto sobre esa

1-7

captulo 1 : circuito magntico

trayectoria est en la misma condicin respecto a la distribucin espacial de corrientes


(simetra).
El campo magntico est confinado al interior del toroide.
En la figura 1.2.2, que corresponde a la H(r)
representacin grfica de la relacin (1.2.1),
se puede apreciar que el campo en el interior Hi
del toroide no es homogneo. Slo si las
dimensiones del toroide son tales que (re-ri ) He
<< ri , vale decir, para una seccin pequea y
un radio interior grande, el campo puede ser
considerado aproximadamente homogneo
con un valor para H igual al promedio de los
valores extremos.
En adelante se supone que esa condicin
est satisfecha.

ri

re

Con campo homogneo, la induccin B es


constante sobre la seccin del toroide, por lo Figura 1.2.2 Distribucin H(r) en un
electroimn toroidal.
que el flujo se calcula simplemente como
= Bq = 0 H q = 0

Nq
i.
2r

(1.2.2)

Considrese ahora que el interior del toroide est relleno con material ferromagntico
de permeabilidad >> 0 . De acuerdo con (1.2.1) el valor de H no es afectado por el
cambio de ncleo. En cambio la induccin B y el flujo s se incrementan. La misma
corriente i produce ahora un flujo r = /0 veces mayor. La amplificacin del flujo
por el ncleo ferromagntico se puede explicar en trminos del efecto orientador
sobre los imanes moleculares, originalmente desordenados, ejercido por el campo
producido por la corriente i .
Supngase que el toroide haya sido dividido en dos partes iguales, de seccin
semicircular, una ocupada por material ferromagntico de permeabilidad y la otra
por material nomagntico de permeabilidad 0 (figura 1.2.3).
Como no se ha alterado ni la simetra, ni la corriente de excitacin i, la modificacin
planteada no afecta a la intensidad del campo H(r) , que sigue descrita por la relacin
(1.2.1). La diferencia de permeabilidad slo afecta a la divisin del flujo entre los dos
semitoroides.

1-8

captulo 1 : circuito magntico

fierro

Ni

aire

a)
b)
Figura 1.2.3 Semitoroide de material ferromagntico con
a)
Excitacin magntica distribuida
b)
Excitacin magntica concentrada
De acuerdo con la relacin (1.2.2) el flujo en el semitoroide de material ferromagntico
vale:
q
Nq
fe = Bfe =
i ,
(1.2.3)
2
4r
mientras que el flujo en el semitoroide de material nomagntico vale :
a = Ba

q
Nq
= 0
i .
2
4r

(1.2.4)

Como >> 0 , fe >> a , es decir, el flujo por el material nomagntico es slo una
pequea fraccin del flujo por el material magntico , por lo que en primera
aproximacin se puede suponer que el flujo por el material nomagntico es
despreciablemente pequeo y que todo el flujo se encuentra en el volumen del
semitoroide de material magntico.
Este resultado permite relajar la exigencia inicial de un enrollado uniformemente
distribuido sobre el ncleo, mediante la cual se garantizaba la circularidad de las
lneas de fuerza y se limitaba el campo al interior del toroide.
Al utilizar material de alta permeabilidad para el ncleo del toroide , la alta
permeabilidad hace que el flujo siga esencialmente confinado al volumen del toroide,
aunque las espiras del enrollado se concentren en un sector del ncleo, dejando al
resto del ncleo descubierto. En esta circunstancia, el campo creado por el devanado
uniformemente distribuido es aproximadamente igual al campo creado por el
devanado concentrado en un sector del toroide, por lo que en este segundo caso
tambin se puede usar la relacin (1.2.2) para calcular el flujo. Este recurso es de
gran utilidad para la obtencin de soluciones analticas aproximadas.

1-9

captulo 1 : circuito magntico

Siempre con la intensin de introducir


aproximaciones razonables que permitan
la formulacin de soluciones analticas,
considrese nuevamente al ncleo toroidal
de material ferromagntico, para analizar
las consecuencias de un pequeo corte
radial de ancho la sobre la distribucin del
campo magntico (figura 1.2.4).

la

Por efecto del corte desapareci la


simetra y, con ella, la lnea argumentativa
que anteriormente permiti obtener Figura 1.2.4 Relativo a la formulacin de
un modelo para un toroide
importantes
conclusiones
sobre
la
con entrehierro.
distribucin espacial del campo.
Sin embargo, la presencia de material ferromagntico de alta permeabilidad,
eventualmente apoyado por un enrollado uniformemente distribuido a lo largo del
ncleo, hace que el flujo quede confinado esencialmente al volumen del toroide,
excepto en la regin prxima al corte o entrehierro.
Dada la dificultad de determinar el campo en el entrehierro y en su entorno inmediato,
se hace una suposicin simplificatoria, es decir, se formula un modelo, asumiendo
que las lneas de fuerza siguen siendo circunferencias en la regin problemtica.
Esto implica que la induccin en el ncleo ferromagntico y en el aire debe tener el
mismo valor:
Bi = Ba ,
de lo que sigue que
Hi = 0 Ha ,

(1.2.5)

donde Hi es la intensidad del campo magntico homogneo en el interior del material


ferromagntico, mientras que Ha es su valor en el entrehierro.
De acuerdo con el modelo, el campo es homogneo tanto en el ncleo como en el
entrehierro y HI y Ha son constantes. En consecuencia
r

H ds = H

l i + Ha l a = N i
con l i = 2 r la .
i

(1.2.6)

A partir de las relaciones (1.2.5) y (1.2.6) se determina la intensidad del campo en el


entrehierro como :

1-10

captulo 1 : circuito magntico

Ha =

Ni

0
l +l
i a

(1.2.7)

Si bien el valor numrico calculado mediante esta relacin es algo superior al real, la
expresin tiene el mrito de mostrar claramente la influencia de los parmetros sobre
el resultado.
Por las caractersticas del modelo, el flujo en el ncleo y en el entrehierro es
necesariamente el mismo y vale
= 0 Ha q =

Ni
li
l
+ a
q 0 q

(1.2.8)

donde q es la seccin del toroide, igual a la del entrehierro.


Si los parmetros en (1.2.8) fuesen conocidos, la relacin permitira determinar con un
grado de aproximacin razonable el flujo en el entrehierro producido por cierta
excitacin magntica, como tambin la excitacin magntica necesaria para obtener
un determinado flujo en el entrehierro. Lamentablemente la permeabilidad de los
materiales ferromagnticos no es constante, ni se conoce de antemano, por lo que el
valor prctico de la expresin (1.2.8) es limitado.
El toroide con entrehierro puede ser considerado como el prototipo de las mquinas
elctricas en lo que a la determinacin del campo magntico se refiere y las
aproximaciones y consideraciones practicadas en relacin con l pueden ser
aplicadas sin mayor dificultad a geometras ms generales si se respeta las
restricciones que limitan su validez. Los conceptos y tcnicas para ello necesarias son
el motivo del prrafo siguiente.
1.3

Circuitos magnticos

Para un circuito de corriente continua, formado por la conexin en serie de una fuente
de tensin V y de dos conductores de longitudes l1 y l2, secciones q1 y q2 y
conductividades 1 y 2 respectivamente, la corriente se calcula como :
I=

V
=
R1 + R2

V
l1
l
+ 2
1 q 1 2 q 2

(1.3.1)

1-11

captulo 1 : circuito magntico

Al comparar (1.3.1) con (1.2.8) salta a la vista la correspondencia formal entre ambas
relaciones. Esta correspondencia a llevado a introducir el concepto circuito magntico
en analoga con el circuito elctrico de corriente continua.
Para ello se establece las siguientes analogas :
Corriente
Tensin
Resistencia

I
V
R

Flujo
Excitacin magntica
Reluctancia

Extendiendo la analoga a las leyes de Ohm y de Kirchhoff, se tiene que en el circuito


magntico rige:
F

(en cada elemento)

(1.3.2)

=0

(en cada nodo)

(1.3.3)

=0

(en cada malla)

(1.3.4)

Como consecuencia de lo anterior, los elementos del circuito magntico, es decir, las
reluctancias, se combinan de la misma manera como se combinan las resistencias en
el circuito elctrico.
Sin embargo, hay una caracterstica fundamental del circuito elctrico que, en general,
no tiene su equivalente en el circuito magntico : la constancia de los parmetros.
Efectivamente, la reluctancia, que para cada tramo con campo homogneo -de
longitud l, seccin q y permeabilidad - se calcula como
=

l ,
q

(1.3.5)

en el caso de materiales ferromagnticos, depende fuertemente del grado de


saturacin, determinado por el flujo.
En consecuencia, los circuitos magnticos, en general, sern nolineales y por lo tanto
para ellos dejan de ser aplicables los mtodos de anlisis basados en el principio de
superposicin, que son justamente los que han hecho de la teora de circuitos una
herramienta tan poderosa .
Frente a esta situacin, y como la nolinealidad se expresa habitualmente a travs de
la caracterstica de magnetizacin Bmax(Hef ), en la prctica se prefiere usar
directamente las variables de campo B y H.

1-12

captulo 1 : circuito magntico

1.0

2.0

3.0

4.0

5.0

6.0

7.6

8.0

9.0

10

2.5

Wb
B 2
m

A
H
m
4
11 10

2.4

Chapa silicosa

2.3
2.2
2.1
2.0
1.9

Wb
B 2
m

1.8

Chapa silicosa

1.7

Acero fundido

1.6
1.5
1.4

1.0

1.3

0.9

1.2

0.8

1.1

0.7

Fierro fundido

1.0

0.6

0.9

0.5

0.8

0.4

0.7

10

0.1

0.2

0.3

0.4

0.5

0.6

0.7

0.8

0.9

1.0

1.1

A
H
m

Figura 1.3.1. Curva de magnetizacin B( H) de algunos


materiales ferrromagnticos
La caracterstica de magnetizacin es suministrada por el fabricante del material. La
figura 1.3.1 muestra esta caracterstica, a modo de ejemplo, para algunos materiales
comunes.
Para ilustrar el tratamiento de los problemas asociados con la nolinealidad,
considrese nuevamente el toroide con entrehierro de la figura 1.2.4., para el cual se
desea determinar la excitacin magntica necesaria para producir un determinado
flujo en el ncleo.
Como el flujo y las dimensiones del toroide son conocidas, se calcula B = q , valor
con el que se entra a la caracterstica del material del ncleo para determinar Hi .
Dado que el flujo supuestamente est limitado a la seccin del ncleo, la induccin en
el entrehierro es la misma que en el ncleo, por lo que Ha = B 0 . Finalmente se
calcula la excitacin magntica o fuerza magnetomotriz i N = Hi l i + Ha l a .
Supngase ahora que la excitacin magntica sea conocida y que se desea
determinar el flujo correspondiente.

1-13

captulo 1 : circuito magntico

Como slo se conoce

li

, pero no los sumandos, no se puede determinar el

valor de Hi en cada tramo del circuito magntico. Esto implica que el problema no
tiene una solucin directa y que es necesario recurrir a un procedimiento iterativo.
Para ello se asume un flujo y se calcula la excitacin magntica necesaria, que se
compara con el dato inicial. Si los dos valores no coinciden dentro de un margen de
error razonable, se modifica apropiadamente el valor supuesto para el flujo y se repite
el procedimiento hasta lograr la convergencia.
Otro problema que carece de una solucin analtica directa corresponde a la
distribucin de un flujo conocido t entre dos ramas en paralelo. La figura 1.3.2
muestra esta situacin y la relacin entre flujos y fmms para cada una de las ramas en
paralelo. Se aprecia que mediante el artificio de representar la relacin entre flujo y
fmm para una de las ramas en el cuarto cuadrante usando una abscisa comn para
las fmms, el flujo total t tambin queda representado por un trazo y es posible
obtener una solucin grfica para el problema.
1
t
t

1(F 1)

1
F1=F 2
0
2

F
2(F 2)

Figura 1.3.2 Relacin entre flujos y fmm en un circuito


magntico formado por dos elementos en paralelo

Pero tambin en este caso se puede recurrir al procedimiento iterativo y suponer un


valor para el flujo 1 por una de las ramas. El flujo por la otra rama se determina como
2 = t 1. Para cada flujo se determina la induccin correspondiente y con ella se
entra a la respectiva caracterstica de magnetizacin para obtener el valor de H en
cada rama. El criterio de iteracin, que verifica si la distribucin de flujos es la
correcta, es en este caso la igualdad de las fuerzas magnetomotrices en las dos
ramas : H1 l1 = H2 l 2 .

1-14

captulo 1 : circuito magntico

Para completar el anlisis de los circuitos magnticos es necesario incluir la


posibilidad que el flujo tenga su origen en un imn permanente en vez de tenerlo en
una corriente. Esto es materia del prrafo siguiente.
1.4

Imanes permanentes

El material ferromagntico incluido en los circuitos magnticos hasta aqu


considerados est caracterizado por un lazo de histresis muy estrecho, cuyas ramas
ascendente y descendente pueden ser consideradas en primera aproximacin como
coincidentes. La induccin B y la intensidad de campo H estn relacionadas en forma
unvoca.
Bi
curva de desmagnetizacin

caracterstica del entrehierro

Br
BP

curva virgen

Hc
HP

Hi

Figura 1.4.1. Caracterstica magntica de un material


magnticamente duro magnetizado hasta la
saturacin y luego desmagnetizado.
Pero tambin existen materiales con un lazo de histresis muy marcado. Una vez
magnetizado el material, la induccin no vuelve a cero cuando se anula la corriente,
sino a un valor conocido como induccin remanente Br. Para llevar la induccin a cero
es necesario invertir la excitacin magntica. El valor de la intensidad de campo
(negativa) para el cual la induccin se hace cero se conoce como fuerza coercitiva Hc .
Las curvas virgen y de desmagnetizacin de la figura 1.4.1 ilustran la relacin entre B
y H descrita.
Los imanes permanentes (de creciente importancia tecnolgica) estn hechos de este
tipo de material (aleaciones NiCo,SmCo,NdFe) y para su caracterizacin importa el
segundo cuadrante de la caracterstica B(H).
Sea un
ncleo toroidal magnetizado mediante la aplicacin de una fuerza
magnetomotriz, que, despus de alcanzar cierto valor mximo, es reducida a cero. En

1-15

captulo 1 : circuito magntico

esas condiciones la induccin en el interior del toroide toma el valor de remanencia ,


Bi = Br, y la intensidad de campo es cero (Hi = 0).
Considrese ahora el corte radial de ancho la a
travs del ncleo representado en la figura 1.4.2

qi

De acuerdo con la ley de Ampere debe


cumplirse que
Hi l i + Ha l a = 0 ,

li

(1.4.1)

la

por lo que
H a = H i

li
.
la

(1.4.2)

Se puede apreciar que, como consecuencia del


corte, Hi es ahora distinto de cero y que su
sentido es inverso al de Ha en el corte.

Figura 1.4.2 Relativo a la aplicacin


de la Ley de Ampere a
un imn permanente.

En el entrehierro formado por el corte rige:


Ba = 0 H a

(1.4.3)

La condicin de continuidad para el flujo implica que


q i Bi = q a Ba .

(1.4.4)

En el caso del imn toroidal, suponiendo un entrehierro suficientemente estrecho, las


secciones qi y qa pueden ser consideradas como iguales, lo que hace que en ese
caso la induccin en el ncleo sea igual a la induccin en el entrehierro.
Las restricciones (1.4.2) a (1.4.4) implican la siguiente relacin entre Bi y Hi :
Bi = 0

l i qa
H
l a qi i

(1.4.5)

que en el plano B(H) corresponde a una recta por el origen en el segundo cuadrante.
Como, por otra parte, los valores de Bi y Hi estn relacionados por la caracterstica de
magnetizacin del material, los valores de Bi y Hi que satisfacen ambas condiciones
se encuentran necesariamente sobre la interseccin de las dos caractersticas: la
recta del entrehierro y la curva de magnetizacin. Este punto P se conoce como punto
de trabajo.

1-16

captulo 1 : circuito magntico

La representacin grfica de estas relaciones en la figura 1.4.1 permite apreciar que,


como consecuencia del corte, la induccin en el ncleo se reduce del valor de
remanencia Br a BP. De la relacin (1.4.5) se desprende que la reduccin est
directamente relacionada tanto con la geometra del imn como con la del entrehierro.
Para aclarar el papel de las geometras en el logro de un determinado valor de la
induccin en el entrehierro amplifquese (1.4.5) con Bi y luego reemplcese Bi por Ba
de acuerdo con (1.4.4). De esa manera se logra
Ba =

qi l i
BH ,
q a la 0 i i

(1.4.6)

donde se aprecia que, para volmenes dados, la induccin en el entrehierro slo


depende del producto Bi Hi , el que a su vez depende de la ubicacin del punto de
trabajo sobre la caracterstica de magnetizacin. Para un punto determinado (que
corresponde aproximadamente a la interseccin de la caracterstica con la diagonal
del rectngulo Br Hc ) el producto alcanza su valor mximo. Este valor mximo es un
parmetro bsico para la caracterizacin de los imanes permanentes.
Con los valores para Bi y Hi correspondientes al producto mximo se determina la
seccin ms favorable para el imn a partir de (1.4.6) y (1.4.4).

Material magntico
Tierras raras-Cobalto
Neodimio-Fierro-Boro
Alnico (Al-Ni-Co-Fe)

Br (T)
0,92
1,20
0,73

Hc (kA/m)
705
860
34

(BH) max ( kJ/m3)


167
240
10

Tabla 1.4.1 Valores caractersticos para imanes permanentes.

2. El reactor
2.1

Introduccin

Fue Faraday quien se pregunt (1822) si a la observacin fundamental de Oersted no


le deba corresponder una relacin causal inversa. Si una corriente estacionaria
convierte en imn al fierro que rodea, por qu un imn permanente no produce
corrientes estacionarias en las espiras que lo rodean?
Nueve aos despus encontr la respuesta a esa interrogante, la que hoy se conoce
como la ley de Faraday y que en su formulacin integral establece que:

r
r
d
E ds =
dt

(2.1.1)

es decir, que, a lo largo de un rcamino de integracin cerrado, la integral de la


componente del campo elctrico E paralela al camino de integracin es igual a la
rapidez de variacin del flujo enlazado por ese camino de integracin. Esta ley
constituye una de las piedras angulares de la Electrotecnia, pues establece la relacin
entre el campo elctrico y el campo magntico que lo origina.
Debido a la relacin existente entre la intensidad del campo elctrico y la densidad de
corriente asociada a ese campo:
r
r j
E=

y a la relacin entrerla intensidad del


campo magntico H y la corriente
abrazada por ste, planteada por la
ley de Ampere, esta y la ley de
Faraday constituyen tambin el nexo
entre la teora de campos y la teora
de circuitos.

(2.1.2)

2
Para ilustrar esta relacin considrese
nuevamente a un reactor toroidal con
ncleo de aire en cuyo interior se
desarrolle un campo que en primera Figura 2.1.1. Reactor en forma de toroide
aproximacin puede ser considerado
como homogneo (figura 2.1.1). El campo elctrico en el interior del conductor
enrollado sobre el ncleo tambin sea homogneo.

2-18

captulo 2 : reactor

Con esta aproximacin y considerando la relacin (2.1.2), la integral curvilnea cerrada


del primer miembro de la ecuacin (2.1.1) toma la forma:

r r 1
E ds =

1
r
r
r
r
j12 ds + E12 ds

(2.1.3)

donde el camino de integracin cerrado se ha dividido en dos tramos, el primero de los


cuales va de 1 a 2 por el conductor y corresponde al primer trmino del segundo
miembro de (2.1.3) mientras que el segundo va de 2 a 1 a travs de la fuente.
Como los campos son paralelos al camino de integracin, las integrales del segundo
miembro de (2.1.3) se convierten en integrales simples cuya integracin es trivial,
obtenindose que :

r
r
l
E ds = cu i v = Rcu i v
q cu

(2.1.4)

donde lcu es la longitud del conductor entre 1 y 2, qcu es su seccin transversal, la


conductividad y j12 la densidad de corriente constante sobre la seccin y a lo largo del
conductor. La tensin v corresponde a la diferencia de potencial entre 1 y 2, impuesta
por la fuente.
Se aprecia que el campo asociado a la corriente en el conductor del toroide ha
quedado representado circuitalmente por una resistencia equivalente, en la que se
disipa la misma energa que en el conductor original.
En cuanto al segundo miembro de (2.1.1), se haba establecido anteriormente (prrafo
1.2) la siguiente relacin entre el flujo en el interior del toroide y la corriente que lo
produce:
=

Nq
i
2 r

(2.1.5)

Considerando que este flujo es enlazado N veces por el conductor, el flujo total
enlazado por el camino de integracin 1-2 vale
= N =

q
N2 i ,
2 r

(2.1.6)

aprecindose que para medios de permeabilidad constante el enlace de flujo es


directamente proporcional a la corriente i. El factor de proporcionalidad lo constituye la
inductancia

2-19

captulo 2 : reactor

L=

q 2
N = N 2
2 r

(2.1.7)

cuyo valor depende de la geometra del circuito magntico a travs de la permeancia


.
Se aprecia que el campo magntico en el interior del toroide ha quedado representado
por una inductancia equivalente en la que se acumula la misma energa que en el
campo original.
i

Reemplazando
finalmente
las
relaciones (2.1.4) y (2.1.6) en (2.1.1)
se logra, despus de reagrupar los
trminos, la siguiente ecuacin:

Rcu =
v

di
v = Ri + L
,
dt

(2.1.8)

&

L=

l cu

cu

q 2
N
2 r

que corresponde a la ecuacin de


Kirchhoff
para
la
malla
RL
Figura 2.1.2. Circuito galvnico equivalente
representada en la figura 2.1.2.
para el reactor toroidal.

Las leyes de Ampere y de Faraday,


que son relaciones entre variables de campo que dependen del espacio y del tiempo,
se han reducido a las leyes de Kirchhoff, que son relaciones entre variables de circuito
que slo dependen del tiempo.
Desde el punto de vista energtico las ecuaciones (2.1.1) y (2.1.8) son totalmente
equivalentes y la malla de la figura 2.1.2 constituye el circuito equivalente del
dispositivo de la figura 2.1.1. Cada elemento del circuito equivalente representa un
efecto fsico del dispositivo original, como la conversin de energa elctrica en calor, la
acumulacin de energa magntica y la relacin entre la corriente y el enlace de flujo.
Las variables de terminales del circuito equivalente son idnticas con las del dispositivo
original.
Estas caractersticas, junto con la mayor simplicidad de la teora de redes, hacen
deseable disponer de un procedimiento para derivar en forma sistemtica el circuito
equivalente de dispositivos electromagnticos ms complejos, para poder caracterizar y
analizar su comportamiento en trminos de las variables de terminales.
Para ello es necesario examinar previamente los efectos fsicos ms comunes en los
dispositivos electromagnticos.

captulo 2 : reactor

2.2

2-20

Efectos fsicos en el reactor

El objetivo fundamental de un reactor es la acumulacin de energa magntica. Por


razones econmicas, en su construccin se trata de ocupar un mnimo de material
activo (fierro, cobre), lo que implica el uso de densidades de flujo y de densidades de
corriente lo ms altas que sea posible, sin que las prdidas en el fierro y las prdidas
en el cobre determinen un calentamiento superior al admisible para el material aislante
utilizado.
El uso de valores elevados para la induccin determina la saturacin del ncleo, que se
refleja en la disminucin de la permeabilidad de ste. Como consecuencia de lo
anterior, una cierta fraccin del flujo se dispersa del camino magntico previsto (a
travs del ncleo) y se cierra a travs del aire. Eso implica la aparicin de un circuito
magntico adicional, en paralelo con el correspondiente al ncleo (figura 2.1.1).
Cuando se excita al reactor con corrientes de alta frecuencia se hace sentir el campo
elctrico entre las capas del devanado y entre estas y el ncleo. Las correspondientes
corrientes de desplazamiento ahora se hacen significativas en comparacin con la
corriente por el devanado y alteran la distribucin de tensin a lo largo del devanado y
la relacin entre las variables de terminales. Este efecto se puede incluir en el circuito
equivalente mediante capacitancias, pero no ser considerado en este captulo.
2.2.1 Dispersin magntica
Por flujo de dispersin se entiende aquella
fraccin del flujo total que no contribuye a un
propsito determinado. El propsito del ncleo
del reactor es servir de camino de baja
reluctancia para el flujo creado por el
devanado. Por lo tanto, el flujo que no sigue
ese camino es considerado como flujo de
dispersin.
Del esquema de la figura 2.2.1 se desprende
que el flujo de dispersin del reactor se cierra
principalmente por el aire, a travs de vas
paralelas a las del flujo principal.
Figura 2.2.1 Flujo de dispersin de un
reactor.
La introduccin del concepto dispersin
magntica implica la divisin del espacio en dos regiones, una asociada al flujo
principal y otra asociada al flujo de dispersin. A cada una de estas regiones est
adscrita una fraccin de la energa magntica total.

2-21

captulo 2 : reactor

m
t

Figura 2.2.2. Esquema de un circuito magntico


equivalente para el reactor.
t = m + .

Este punto de vista es recogido


por el modelo de la figura 2.2.2,
formado por un circuito magntico
ideal con dos ramas en paralelo.
El entrehierro incluido en cada
rama es tal que la energa
magntica acumulada en l sea
igual a la de la respectiva regin
del
espacio
que
se
est
modelando.
De acuerdo con el modelo
(2.2.1)

Al introducir la fuerza magnetomotriz comn y las permeancias correspondientes a


cada rama se logra:
t = m F + F = ( m + )Ni .

(2.2.2)

Para representar correctamente la energa


Wm =

1
1
F = N 2 i 2 ,
2
2

(2.2.3)

asociada al campo de dispersin, la permeancia de la rama de dispersin debe ser tal


que
=

2W m
N 2i 2

. ,

(2.2.4)

donde el valor de W m se supone conocido.


La distincin entre campo en el aire y campo en el fierro se hace necesaria porque el
campo en el aire es conservativo, mientras que el campo en el fierro, cuando es
alterno, es disipativo. Este ltimo aspecto estaba explcitamente excluido en el toroide
de la figura 2.1.1 y ser el objetivo del prximo prrafo.
2.2.2 Prdidas en el fierro
La expresin para el flujo reproducida en (2.1.5) slo es rigurosamente vlida en el
caso de una excitacin continua o cuando el ncleo est formado por material no
conductor. Cuando el ncleo es de material ferromagntico y es excitado por corrientes

2-22

captulo 2 : reactor

alternas, el flujo alterno induce corrientes adicionales, que circulan en el interior del
ncleo, abrazando el flujo que las induce. Estas corrientes parsitas modifican la
distribucin del flujo sobre la seccin del ncleo, hacindola inhomognea, y tambin
son la causa de la conversin irreversible de energa elctrica en calor, conocida como
prdidas de Foucault o de corrientes parsitas.
El efecto amplificador de flujo de los materiales ferromagnticos puede interpretarse
cualitativamente postulando la existencia de imanes moleculares. Cuando el material
es sometido a un proceso de magnetizacin alterna estos imanes tienen que
reorientarse dos veces por ciclo, lo que requiere de energa, cuya transformacin en
calor puede atribuirse al roce entre los imanes moleculares durante su reorientacin.
La cantidad de energa convertida en calor por cada ciclo es proporcional al rea del
lazo de histresis, conocindose esas prdidas como prdidas de histresis.
Para los fines de la modelacin de estos efectos fsicos mediante el circuito equivalente
basta su anlisis cualitativo sobre la base de aproximaciones relativamente groseras,
que permiten evitar desarrollos matemticos ms complejos, pero mantienen la
informacin relevante.
2.2.2.1

Prdidas de Foucault.

Considrese una platina de material


ferromagntico de resistencia especfica ,
de longitud l y seccin rectangular tal, que
el espesor d sea mucho menor que el
ancho b (figura 2.2.3). En el interior de la
platina exista un campo alterno sinusoidal
de frecuencia angular que en primera
aproximacin puede ser considerado como
homogneo.

d
m
b

l
Un circuito coincidente con los lados de la
seccin rectangular abrazara un flujo cuyo
valor mximo sera m = Bm b d , al
Figura 2.2.3. Platina de material
suponer que el campo es homogneo, y
ferromagntico.
en l se inducira la tensin
vi =

d
( sen t ) = m cos t = 2Vi cos t
dt m

(2.2.5)

que hara circular una corriente limitada slo por la resistencia de ese circuito.
Dadas las proporciones de la platina, para la resistencia del circuito se puede plantear
en primera aproximacin

captulo 2 : reactor

2b
.
d
l
2

2-23

(2.2.6)

Considerando las relaciones (2.2.5) y (2.2.6), las prdidas por corrientes parsitas por
unidad de volumen estaran dadas por
PF 1 Vi 2
2 B m 2 d 2
= ( )
V V R
4

(2.2.7)

donde V = bdl es el volumen de la platina.


A pesar de las aproximaciones usadas en su obtencin, la relacin (2.2.7) refleja
adecuadamente la influencia de los principales parmetros sobre las prdidas por
corrientes parsitas. As se aprecia que stas pueden ser reducidas notablemente a
travs de la disminucin del espesor de la platina y mediante el aumento de la
resistencia especfica.
Esta conclusin se refleja en la prctica en el uso de chapas silicosas de 0,35 mm de
espesor, aisladas elctricamente entre s, para la construccin de ncleos y circuitos
magnticos sometidos a excitacin alterna.
Por otra parte, el uso de chapas aisladas no solamente atena la magnitud de las
corrientes parsitas, sino que, al fijarles los circuitos por los cuales pueden circular,
tambin limita su desarrollo espacial sobre la seccin del ncleo, con lo que se
recupera una distribucin de induccin prcticamente homognea.
En consecuencia, para frecuencias industriales (50Hz) los ncleos laminados pueden
ser modelados con el concepto de circuito magntico y el nico fenmeno adicional que
hay que considerar son las prdidas debidas a las corrientes parsitas.
Para fines prcticos las prdidas en el fierro debidas a las corrientes parsitas o
prdidas de Foucault se expresan como
PF = CF Bm

2
2
f d

m W
50 0,5

(2.2.8)

donde m es la masa en kg y CF es la cifra de prdidas por corrientes parsitas, que


corresponde a las prdidas en W en 1kg de chapas de 0,5mm de espesor, medidas
para una induccin mxima de 1T y una frecuencia de 50Hz.
La cifra de prdidas por corrientes parsitas CF vara entre valores del orden de 0,16
W/kg para chapas de grano orientado para uso en transformadores y valores del orden
de 0,8 W/kg para el uso en motores de potencia fraccionaria.

captulo 2 : reactor

2.2.2.2

2-24

Prdidas por histresis

En el anlisis precedente se haba supuesto tcitamente que la permeabilidad del


material del ncleo era constante. Ahora se relajar esa restriccin para examinar ms
detenidamente una caracterstica nolineal propia de los materiales ferromagnticos y
sus consecuencias.
Resulta que la caracterstica de magnetizacin de los materiales ferromagnticos no es
unvoca, vale decir, a un determinado valor de la intensidad del campo H no le
corresponde un valor de induccin B nico, sino que ese valor depende de la historia
magntica previa del material.
En un material ferromagntico sometido a una magnetizacin alterna de amplitud y
frecuencia constantes se establece finalmente un estado cclico que en el plano B-H
toma la forma del lazo de histresis .Esta caracterstica emprica refleja el efecto de la
saturacin y de la histresis sobre el campo magntico y constituye el punto de partida
para el anlisis que sigue.
Para fijar las ideas, considrese nuevamente un reactor de ncleo toroidal
ferromagntico de radio r y seccin q. El campo, confinado al volumen del toroide,
puede ser considerado homogneo. La resistencia del enrollado de N vueltas sea
despreciable.
La energa suministrada al campo a travs de los terminales de la bobina en el lapso dt
vale:
dW = p dt = iv dt = iNd = iNq dB
pero como

iN = 2 r H

dW = 2rq HdB = V HdB ,

(2.2.9)

donde V = 2rq es el volumen del toroide.


En consecuencia, la energa magntica suministrada al campo cuando la induccin B
vara desde un valor inicial B1 hasta un valor final B2 vale
B2

W = V HdB

(2.2.10)

B1

y como se trata de un campo homogneo, la densidad de energa, o energa por unidad


de volumen, queda expresada por la relacin:

2-25

captulo 2 : reactor

B2

w = HdB

(2.2.11)

B1

Esta ltima relacin es vlida para cualquier campo, ya que todo campo puede ser
tomado por homogneo si se considera regiones suficientemente pequeas. Su
interpretacin geomtrica corresponde a un elemento de rea en el plano B-H.
Esta interpretacin permite visualizar las prdidas de histresis por ciclo y por unidad
de volumen como el rea encerrada por el lazo de histresis.
B

Para comprobarlo, basta recorrer el


lazo de histresis de la figura 2.2.4
+Bmax
c
durante un ciclo de la excitacin. En el
+Br
primer cuarto de ciclo H vara entre 0 y
d
+Hmax y la induccin B lo hace entre Br y +Bmax . La energa absorbida
corresponde al rea entre la rama
ascendente (abc) de la curva H(B) y el
eje de ordenadas. En el segundo
+Hc
H
-Hc
cuarto de ciclo H vara entre +Hmax y 0
b +Hmax
0
-Hmax
y la induccin B lo hace entre +Bmax y
+Br. El rea bajo la rama descendente
(cd) de la curva H(B) y el eje de
ordenadas es ahora negativa y
corresponde a la energa devuelta a la
a
fuente. De manera que la energa por
-Br
unidad de volumen neta absorbida
- Bmax
desde la fuente durante el primer
semiciclo de la funcin de excitacin
(corriente) corresponde al rea abcd0a
Figura 2.2.4.Lazo de Histresis
en la figura 2.2.4. La continuacin del
anlisis durante el segundo semiciclo
de la corriente permite comprobar la relacin entre el rea del lazo de histresis y la
energa disipada por unidad de volumen del ncleo en cada ciclo debido a la histresis.
A Steinmetz se debe la siguiente expresin emprica para las prdidas especficas por
histresis
x
w H = Bmax

(2.2.12)

cuyos parmetros y x deben ser determinados experimentalmente para cada material


especfico. Para fines analticos se supone que el exponente de Steinmetz toma el
valor x=2.
En consecuencia, las prdidas por histresis para un ncleo de volumen V excitado
con corrientes de frecuencia f valen

captulo 2 : reactor

2
PH fBmax
V

2-26

(2.2.13)

Para fines prcticos se utiliza la frmula


f
2
m W
PH = CH Bmax
50

(2.2.14)

donde m es la masa del ncleo en kg, CH es la cifra de prdidas por histresis que
corresponde a las prdidas en W en 1 kg de material, medidas para una induccin
mxima de 1 T y frecuencia igual a 50 Hz.
CH vara tpicamente entre 0,4W/kg para chapas de transformadores y 1,6W/kg para
chapas de motores de potencia fraccionaria.
2.2.3 Corriente magnetizante compleja
La forma peculiar del lazo de histresis implica una relacin nolineal entre la induccin
B y la intensidad de campo H y por lo tanto entre la tensin y la corriente magnetizante.
Al aplicar al devanado de excitacin una tensin sinusoidal se fuerza que el flujo, y por
lo tanto la induccin, sea sinusoidal. La caracterstica B(H) nolineal determina que H y
por lo tanto la corriente magnetizante sean nosinusoidales, es decir, que junto a la
componente fundamental aparezcan armnicas impares.
La figura 2.2.5 ilustra la obtencin grfica de la forma de onda de la corriente a partir de
la forma de onda de la induccin y del lazo de histresis esttico, trazado con lnea
llena (no incluye el efecto de las corrientes parsitas).
Como el circuito equivalente est compuesto por elementos lineales y por esa razn no
puede reproducir efectos nolineales, la corriente magnetizante compleja tiene que ser
reemplazada por una corriente sinusoidal equivalente cuyos parmetros caractersticos:
amplitud, frecuencia y fase estn determinados por las siguientes exigencias:
Amplitud:
El valor efectivo de la corriente sinusoidal equivalente debe ser igual al
valor efectivo de la corriente compleja que reemplaza
I = I12 + I 32 + I 52 + .......

(2.2.15)

2-27

captulo 2 : reactor

B,

iH
i H+iF

iF

wt

i, H
Lazo esttico
Lazo dinmico

Figura 2.2.5. Determinacion grfica de la corriente de excitacin del


reactor.

Frecuencia: La frecuencia de la corriente sinusoidal equivalente debe ser igual a la


frecuencia fundamental de la corriente compleja que reemplaza
f = f1

(2.2.16)

Fase:
El ngulo de fase de la corriente sinusoidal equivalente respecto a la
tensin inducida Vi debe ser tal que las prdidas sean las mismas
P + PF
= arccos H
.
Vi I

(2.2.17)

2.2.4 Prdidas en el cobre


Al integrar la expresin (2.1.3) se haba supuesto que el campo elctrico en el interior
del conductor fuera homogneo. Esto se cumple en el caso de corrientes continuas, por
lo que la potencia disipada en el conductor al circular una corriente continua de
intensidad I por l vale
Pcu = V I = R I 2
l
donde R = es la resistencia de corriente continua.
q

(2.2.18)

2-28

captulo 2 : reactor

Esta relacin se puede reescribir como


Pcu =

l
2
(
q j ) = j 2 Vcu
q

(2.2.19)

con Vcu=ql volumen del conductor, de la que se desprende que las prdidas por unidad
de volumen estn dadas por
p cu = j 2
,
(2.2.20)
relacin de validez general, ya que cualquier campo puede ser tomado como
homogneo si se considera regiones suficientemente pequeas.
En cambio con excitacin alterna el volumen del conductor es ocupado por un campo
magntico alterno que induce en el conductor corrientes parsitas que alteran la
distribucin de la densidad de corriente sobre la seccin del conductor hacindola
nohomognea, por lo que las prdidas deben determinarse a partir de
Pcu ,ca = j 2dVcu = l j 2 dq
Vcu

(2.2.21)

Resulta que para distribuciones nohomogneas


I2
j dq > q
q
2

(2.2.22)

donde I es el valor efectivo de la corriente en el conductor.


En consecuencia, Pcu,ca > R I2 = Pcu,cc , es decir, las prdidas con corriente alterna de
igual valor efectivo que una corriente continua son mayores que las causadas por la
corriente continua.
En la prctica se considera este hecho definiendo una resistencia para corriente alterna
Rca > R tal que
Pcu ,ca = Rca I 2 ,

(2.2.23)

reduciendo de esta manera el problema a uno homogneo equivalente. El valor de la


resistencia equivalente para corriente alterna depende de la geometra de la bobina, de
la seccin de los conductores y de la frecuencia. Para frecuencias industriales (50Hz) el
valor es del orden de un 10% superior al de la correspondiente resistencia para
corriente continua.

2-29

captulo 2 : reactor

2.3

Circuito equivalente.

Por circuito equivalente de una mquina o dispositivo electromagntico se entiende una


red de elementos concentrados (resistencias, inductancias, capacitancias), donde cada
elemento representa un efecto fsico (acumulacin o disipacin de energa) asociado al
dispositivo original.
En forma ms general el trmino tambin se aplica a la red que se obtiene de la
anterior mediante transformaciones de esta que mantengan la identidad de los
terminales de la red (y dispositivo) original.
Para la derivacin sistemtica de estos circuitos equivalentes resulta conveniente
introducir previamente algunos elementos de la teora de los circuitos
electromagnticos.
2.3.1 Circuitos electromagnticos.
En general, la nocin de circuito involucra la aproximacin campos homogneos
(eventualmente equivalentes) limitados a una regin del espacio. Con esta
aproximacin se hace posible la integracin de las ecuaciones de Faraday y de Ampere
y con ello, la descripcin del problema en trminos de parmetros, que dependen de
las dimensiones geomtricas y de las propiedades elctricas o magnticas de los
medios, y de variables que slo son funciones del tiempo.
En el caso de los circuitos electromagnticos, a esta caracterstica fundamental de los
circuitos se agrega el hecho que su forma topolgica siempre puede ser obtenida por
inspeccin del dispositivo fsico que se pretende modelar.
Para fijar las ideas, considrese nuevamente el reactor toroidal de la figura 2.1.1, pero
ahora su ncleo sea de material ferromagntico de permeabilidad y resistividad finitas y
constantes.
En consecuencia, en l se acumular energa magntica y, en caso de flujo alterno,
tambin se producirn prdidas.
La energa acumulada en el campo, que se concentra en el ncleo, es igual a la
densidad de energa (en el caso lineal igual a 12 BH ) por el volumen del toroide:
Wm =

1
H 2 2rq
2

Reemplazando H =
Wm =

(2.3.1)
im N
queda
2 r

1 q
1

N 2 i m 2 = Li m 2
2 2 r
2

(2.3.2)

2-30

captulo 2 : reactor

Se aprecia que la energa magntica queda expresada en trminos del parmetro


inductancia (L) y de la variable corriente (im).
Si ahora tambin se consideran las prdidas en el fierro, estas se determinan como
Pfe = C( f )Bm2 2rq = C(f ) 2m

2r
q

(2.3.3)

donde C( f ) es la cifra de prdidas por unidad de volumen para cierta frecuencia f y una
induccin mxima de 1T.
Considerando que con excitacin sinusoidal V = Nm / 2 , se logra la expresin
Pfe = C( f )

2r
1 2
2
=
V .
2
2 2 V
q N
Rfe

(2.3.4)

donde las prdidas quedan expresadas en trminos del parmetro resistencia ( Rfe ) y
de la variable tensin (V).
De las relaciones (2.3.2) y (2.3.4) se desprende que desde el punto de vista energtico
el dispositivo original de la figura 2.1.1 es equivalente al modelo de la figura 2.3.1,
donde el ncleo real ha sido reemplazado por un ncleo ideal provisto de dos bobinas
ideales de N vueltas cada una, a cuyos terminales estn conectadas respectivamente
una inductancia, que acumula la energa magntica que estaba asociada al ncleo real
(impedancia magntica conservativa), y una resistencia, en la que se disipa la energa
equivalente a las prdidas en el fierro del ncleo real (impedancia magntica
disipativa).

A cada bobina ideal se puede asociar una


fuerza magnetomotriz, relacionada con el

flujo abrazado por esa bobina mediante una


i1
RFe
v

0 N
impedancia magntica:
v1

F = Zm

im
N

Figura 2.3.1. Circuito electromagntico


de un electroimn con
ncleo de fierro.

(2.3.5)

Al reemplazar la fuerza magnetomotriz en


trminos de la corriente, F = NI , y el flujo en
trminos de la tensin inducida por l en la
bobina, V = jN , se logra una relacin
entre la impedancia magntica y la
impedancia elctrica , Z = V / I , conectada
a los terminales de la bobina ideal:

2-31

captulo 2 : reactor

Zm =

jN 2
Z

(2.3.6)

En trminos de las variables flujo y fuerza magnetomotriz y del parmetro impedancia


electromagntica el dispositivo original puede ser reducido al circuito electromagntico
de la figura 2.3.2.

En el caso ms general, un circuito


electromagntico
est
constituido
por
combinaciones en serie y en paralelo de
impedancias electromagnticas, que pueden
Zm1
F1
ser reducidas a impedancias equivalentes.

Para encontrar la expresin correspondiente a


F
una combinacin serie de dos impedancias
electromagnticas
(figura
2.3.3)
debe
Zm2
F2
considerarse que el flujo es comn a los dos
elementos y que la fuerza magnetomotriz
equivalente es igual a la suma de las fuerzas
magnetomotrices correspondientes a cada
elemento:
Figura 2.3.2. Circuito electromagntico
serie
F= F +F
(2.3.7)
1

Expresando las fuerzas magnetomotrices de los elementos en trminos de las


correspondientes impedancias electromagnticas y del flujo comn se obtiene:
1
1
F = jN 2 +

Z1 Z2

(2.3.8)

F1

Z1

F2

Z1

Z2

Figura 2.3.3.Reduccin de dos impedancias


magnticas en serie

Z2

2-32

captulo 2 : reactor

Se aprecia que la impedancia magntica equivalente


Z + Z2
Z m = j N 2 1

Z1 Z2

(2.3.9)

est formada por una bobina ideal de N vueltas a cuyos terminales est conectada una
impedancia que corresponde a la conexin en paralelo de las impedancias asociadas a
cada elemento.

Z1

Z2

Z1

Z2

Figura 2.3.4.Reduccin de dos impedancias magnticas


en paralelo.
Para encontrar la impedancia magntica equivalente de una combinacin en paralelo
de dos elementos (figura 2.3.4) debe considerarse que ahora la fuerza magnetomotriz
es comn a ambos elementos, mientras que el flujo resultante es igual a la suma de los
flujos por cada elemento:
= 1 + 2

(2.3.10)

Reemplazando el flujo a travs de cada elemento en trminos de la fuerza


magnetomotriz comn y de las correspondientes impedancias magnticas se obtiene:
=

Z1 + Z 2
F
jN 2

(2.3.11)

de lo que se desprende que la impedancia magntica equivalente vale en este caso:


Z m = j N 2

1
.
Z1 + Z 2

(2.3.12)

Est formada por una bobina de N vueltas a cuyos terminales est conectada una
impedancia equivalente a la conexin serie de las impedancias correspondientes a
cada elemento.

2-33

captulo 2 : reactor

Topolgicamente las impedancias magnticas y elctricas se comportan como


elementos duales.
Los conceptos hasta aqu desarrollados son suficientes para la obtencin sistemtica
de los circuitos equivalentes de aparatos electromagnticos.
2.3.2 Circuito equivalente del reactor
El procedimiento general para la obtencin del circuito equivalente consiste en:

La fijacin de la topologa del circuito


magntico ideal que incluya a todos
los flujos que se quiera representar.

La inclusin, en los lugares que


corresponda, de las impedancias
magnticas correspondientes a los
efectos fsicos que se desee
representar.

La
reduccin
del
circuito
electromagntico resultante, mediante
combinaciones en serie o en paralelo
Figura 2.3.5. Circuito magntico ideal del
de
impedancias
magnticas,
reactor con dispersin.
manteniendo la identidad de los
terminales externos.
m

I
R

RFe

Lm

Figura 2.3.6 Circuito electromagntico del reactor

Para el caso especfico del reactor


se puede identificar un flujo
comn, abrazado por la bobina de
excitacin, que fuera de ella,
debido a la permeabilidad finita
del fierro del ncleo y a la
eventual
presencia
de
un
entrehierro, se divide en un flujo
por el ncleo y en un flujo por el
aire. La correspondiente topologa
del circuito magntico ideal se
muestra en la figura 2.3.5, donde
la bobina de excitacin real ha
sido
convenientemente
reemplazada por una bobina ideal
y una resistencia en serie que
representa las prdidas en el
cobre de la bobina real.

2-34

captulo 2 : reactor

Si ahora se supone en primera aproximacin que las prdidas en el fierro pueden ser
asociadas solamente al flujo en el ncleo, se las puede representar mediante la
correspondiente impedancia magntica (disipativa) ubicada en esa rama del circuito
magntico.
Las energas magnticas asociadas respectivamente a los flujos en el ncleo y en el
aire se representan mediante sendas impedancias magnticas (conservativas) en las
correspondientes ramas del circuito magntico.
La figura 2.3.6 muestra el circuito electromagntico obtenido en la forma descrita.

Figura 2.3.7 Reduccin del circuito electromagntico

R Fe

Los pasos siguientes son


puramente rutinarios y
consisten en la reduccin
de las dos impedancias
en
serie
a
una
impedancia equivalente y
de las dos impedancias
en
paralelo
a
otra
impedancia equivalente
(figura 2.3.7) y luego en
la reduccin de las dos
impedancias en serie,
resultantes
de
la
operacin anterior, a una
sola
impedancia
equivalente (figura 2.3.8).

El resultado final de estas operaciones es una red elctrica conectada en paralelo con
un reactor ideal. Este ltimo equivale a un circuito abierto, ya que no absorbe corriente
(H=0), y por lo tanto puede ser ignorado.
Toda la informacin relevante respecto al reactor est contenida en la red elctrica. Ella
constituye el circuito equivalente del reactor. Cada elemento representa un fenmeno
fsico de ste que ha sido considerado en el proceso de modelacin. Su impedancia de
entrada es igual a la del reactor original, si se asigna los valores adecuados a los
cuatro parmetros.
Si se intenta determinar los valores de los cuatro parmetros a partir de mediciones de
tensin, corriente y potencia en los terminales del reactor se encuentra que estas
mediciones slo permiten determinar dos parmetros: una resistencia equivalente y
una inductancia equivalente. No es posible determinar separadamente, por ejemplo, la
inductancia de dispersin. Este hecho pone lmites prcticos en el momento de
formular el modelo de un dispositivo electromagntico, pues un modelo cuyos
parmetros no pueden ser verificados empricamente es de poca utilidad prctica.

2-35

captulo 2 : reactor

m
I

i=0

L
Lm

RFe

circuito equivalente

reactor ideal

Figura 2.3.8. Resultado final de la reduccin del circuito


electromagntico del reactor

2.4

Tensin inducida

En su formulacin ms general de la ecuacin (2.1.1), la ley de Faraday no impone


ninguna restriccin sobre la forma en que vara el flujo con el tiempo. Slo establece
que cada vez que vare el flujo enlazado por un circuito cerrado se inducir una tensin
en ste.

max
min

Figura 2.4.1 Funcin peridica de perodo T


Considrese ahora el importante caso particular en el que el flujo es una funcin
peridica del tiempo (figura 2.4.1):
(t ) = (t + T )

(2.4.1)

2-36

captulo 2 : reactor

El valor medio de la tensin inducida por la variacin del flujo durante el perodo T est
dado por la expresin
Vmed

1
=
T

t 1 +T

v dt

(2.4.2)

t1

que, al reemplazar v =

d
y cambiar los lmites correspondientemente, toma la forma
dt

Vmed

1
1
= d = ( (t 1 + T ) (t 1 ))
T 1
T

(2.4.3)

donde se puede apreciar que el valor medio de la tensin inducida slo depende del
valor inicial y del valor final del flujo enlazado, siendo independiente de los valores
intermedios. Esto implica que sobre un perodo el valor medio de la tensin inducida es
cero.
Si (t) es tal que el valor mximo max y el valor mnimo min estn separados por un
semiciclo, el valor medio vale
Vmed =

2
( min )
T max

(2.4.4)

y si adicionalmente max = min = m , la expresin para el valor medio de la tensin


inducida se reduce a
Vmed =

4m
T

(2.4.5)

Si el circuito inducido corresponde a un devanado concentrado, cuyas N vueltas


enlazan todas el mismo flujo m , entonces m = Nm y
Vmed = 4 f N m .

(2.4.6)

La relacin entre el valor efectivo V y el valor medio se conoce como factor de forma
=

V
,
Vmed

cuyo valor depende de la forma de onda.


Para ondas sinusoidales el factor de forma vale

(2.4.7)

captulo 2 : reactor

2 2

= 111
,

2-37

(2.4.8)

y por lo tanto el valor efectivo de la tensin inducida vale en este caso:


V = 4,44 f N m .

(2.4.9)

Esta forma ms especializada de la ley de Faraday es el punto de partida para el


dimensionamiento de mquinas y dispositivos de corriente alterna.

3. El transformador

3.1

Introduccin

La ley de Faraday establece la relacin entre la tensin inducida en un circuito y la


rapidez de la variacin del flujo enlazado por ese circuito, dejando abierto el origen del
flujo y de la causa de la variacin del flujo.
Si el origen del flujo enlazado por un circuito (1) se encuentra en la corriente que circula
por otro circuito (2), se dice que esos dos circuitos estn acoplados inductivamente.
Esta influencia inductiva recproca se caracteriza mediante la inductancia mutua L12 =
L21, parmetro que en conjunto con las inductancias propias de esos circuitos L1 y L2
permite describir el flujo enlazado por cada circuito en trminos de las corrientes i1 y i2
en esos circuitos:
1 = L1 i1 + L12 i 2
2 = L21 i1 + L2 i 2

(3.1.1)

Los valores de las inductancias mutuas y de las inductancias propias dependen de la


geometra de los circuitos y de la permeabilidad del medio. En presencia de materiales
ferromagnticos el valor de la permeabilidad depende del grado de saturacin, por lo
que las inductancias dejan de ser constantes.
Para evitar las dificultades implcitas en el hecho que todas las inductancias sean
nolineales se ha buscado formas alternativas para describir el acoplamiento inductivo
entre bobinas a travs de la definicin de esquemas de acoplamiento inductivo
basados en flujos ficticios.
Debido a la distribucin espacial de los circuitos no todo el flujo enlazado por el circuito
inductor es tambin enlazado por el circuito inducido. El acoplamiento magntico es
imperfecto y se habla de dispersin inductiva.
En la teora clsica del transformador de dos devanados, cuyo estudio es el objetivo de
este captulo, el acoplamiento inductivo imperfecto se modela definiendo un flujo comn
a ambos devanados y sendos flujos de dispersin, cada uno acoplado slo con uno de
los devanados. Se supone los flujos de dispersin se cierran principalmente por el aire,
por lo que las correspondientes inductancias sern constantes.
El circuito magntico ideal as definido se completa para formar el circuito
electromagntico a partir del cual se logra en forma rutinaria el circuito equivalente del
transformador, de cuyos parmetros inductivos solamente uno depende de la
saturacin.

3-39

captulo 3: transformador

Sobre la base de ese modelo se analiza las caractersticas de funcionamiento del


transformador de dos devanados.
3.2

Dispersin inductiva y esquema de acoplamiento inductivo

Sean dos circuitos de geometra cualquiera, rodeados de un medio de permeabilidad


constante 0 . Los enlaces de flujo de esos circuitos estn definidos por la relaciones
(3.1.1) y los parmetros L 1 , L 2 y L12 pueden ser determinados mediante mediciones en
los terminales de los dos circuitos. La resistencia de los circuitos sea despreciable.
Para los circuitos rige respectivamente:
v1 =

d1
dt

(3.2.1)

v2 =

d 2
dt

(3.2.2)

Supngase ahora que el circuito 2 est cortocircuitado , es decir, v2 = 0.


De acuerdo con (3.2.2) esto implica que el flujo enlazado por el circuito 2 debe
permanecer constante, lo que en ausencia de corriente continua significa que debe ser
cero.
Considerando esto en (3.1.1) se logra la siguiente expresin para el flujo enlazado por
el circuito 1 estando el circuito 2 cortocircuitado:
L L
1 = 1 12 21 L1 i1
L1 L2

(3.2.3)

Como 2 = 0 , el flujo producido por la corriente i1 en esas condiciones no puede


enlazar el devanado 2 y debe cerrarse a travs de vas de dispersin.
La expresin entre parntesis se conoce como coeficiente de dispersin total
= 1

L12 L21
L1 L2

(3.2.4)

es una medida del grado de acoplamiento inductivo entre los dos circuitos y tiene una
estrecha relacin con el coeficiente de acoplamiento k de la teora de redes:
= 1 k 2

(3.2.5)

captulo 3: transformador

3-40

vara entre 0 para circuitos perfectamente acoplados y 1 para circuitos totalmente


desacoplados.
El sistema de dos bobinas de geometra indefinida hasta aqu considerado no posee un
circuito magntico en el sentido del concepto definido en el captulo 1.
Sin embargo, mediante una conveniente manipulacin de las ecuaciones (3.1.1) es
posible crear las ficciones flujo comn y flujos de dispersin.
Para ello las ecuaciones (3.1.1) se reescriben en forma amplificada como sigue:
1 = L1 i1 + L12 i 2 + 1L21 I1 1L2 1 i 1
2 = L2 i 2 + L21 i 1 + 2 L12 i 2 2 L12 i 2

(3.2.6)

donde 1 y 2 son constantes arbitrarias .


Reagrupando los trminos de (3.2.6) se logra
1 = ( L1 1 L12 ) i 1 + L12 ( 1 i1 + i 2 )
2 = (L2 2 L12 ) i 2 + L12 ( 2 i 2 + i1 )

(3.2.7)

donde puede apreciarse que como resultado de la manipulacin los enlaces de flujo 1
y 2 aparecen formados por dos componentes : una debida exclusivamente a la
corriente del propio circuito y otra en que participan las corrientes de ambos circuitos.
A las componentes
1 = ( L1 1L12 ) i1 = L1 i1
2 = (L2 2 L12 ) i 2 = L2 i 2

(3.2.9)

se las denomina enlaces de flujo de dispersin, mientras que a las componentes


m 1 = L12 ( 1 i1 + i 2 )
m 2 = L12 ( 2 i 2 + i 1 )

(3.2.10)

se las denomina enlaces de flujo principal.


Entre los coeficientes arbitrarios 1 y 2 se puede establecer una relacin, si se exige
que las inductancias de dispersin L1 y L2 definidas en (3.2.9) se anulen cuando el
coeficiente de dispersin total se hace cero.
Reemplazando

captulo 3: transformador

L1 = 1L12 + L1

3-41

(3.2.11)

L2 = 2 L12 + L 2
en la relacin (3.2.4) queda:
= 1

1 2 L212 + L1L2

L212
+ L12 ( 1L1 + 2 L2 )

(3.2.12)

de donde se desprende que con L1 = L2 = 0 slo se anula si


1 2 = 1

(3.2.13)

El establecimiento de una relacin, exigible desde el punto de vista de la fsica, entre el


coeficiente de dispersin total , que es una medida del grado de acoplamiento de los
circuitos reales, y las inductancias de dispersin ficticias L1 y L2 , que representan el
acoplamiento imperfecto en el esquema de acoplamiento inductivo, reduce el nmero
de parmetros arbitrarios a uno solo:
1 =

1
2

(3.2.14)

del que se puede disponer de acuerdo con la ventaja analtica que se busque. As, por
ejemplo, si se hace 1 = L1 L12 , resulta de (3.2.9) que L1 = 0 , lo que puede ser muy
conveniente en algunas ocasiones.
En el caso del transformador de potencia,
con sus dos devanados de N1 y N2 vueltas
m
respectivamente,
estrechamente
acoplados a travs de un ncleo comn
de material ferromagntico, la teora
1
clsica del transformador de dos
devanados dispone del parmetro 1
postulando el esquema de acoplamiento
2
inductivo de la figura 3.2.1 con un flujo
ficticio m , que enlaza todas las N1
vueltas del devanado (1) y todas las N2
vueltas del devanado (2).Es decir, impone Figura 3.2.1 Esquema de coplamiento
inductivo
m1 = N1 m
(3.2.15)
m 2 = N 2 m
que al reemplazar (3.2.10) y (3.2.14) toma la forma

captulo 3: transformador

3-42

L12 ( 1 i1 + i 2 ) = N1 m
L12 (

1
i + i ) = N2 m
1 2 1

(3.2.16)

Al multiplicar la primera de estas ecuaciones por N2 y la segunda por N1 y formar la


diferencia, queda finalmente
N
( N2 1 N1 ) i1 + (N2 1 ) i 2 = 0
,
(3.2.17)
1
relacin que debe cumplirse para cualquier valor de i1 e i2 , por lo que los coeficientes
de i1 y de i2 deben ser nulos, lo que se cumple si
1 =

N1
N2

(3.2.18)

Como se ver, la introduccin de un esquema de acoplamiento inductivo permite


describir el comportamiento del transformador en forma simple y superar la dificultad
asociada a la influencia de la saturacin del ncleo sobre las inductancias, pero no
autoriza a pensar que los flujos tan arbitrariamente definidos tienen existencia real.
Deben ser considerados como ficciones y en cada caso particular hay que averiguar
hasta qu punto son identificables con los flujos existentes en el dispositivo que se est
modelando.
La importancia prctica del esquema de acoplamiento inductivo reside en el hecho que
en las principales mquinas elctricas es efectivamente posible asociar
razonablemente los flujos del esquema con flujos existentes en diferentes regiones de
la mquina, siendo de ese modo posible determinar los correspondientes parmetros a
partir de la geometra de la mquina.
3.3

El transformador de potencia.

La transmisin eficiente de energa elctrica desde los lugares de generacin a los de


consumo requiere del uso de diferentes niveles de tensin que se logran mediante
transformadores de potencia.
El transformador de potencia monofsico consiste, en lo esencial, en un ncleo cerrado
de chapas de alta permeabilidad, sobre el cual estn dispuestas dos bobinas. La figura
3.3.1 muestra en forma esquemtica un dibujo en corte.
Para la construccin del ncleo se emplea casi exclusivamente chapas de grano
orientado, laminadas en fro, de 0,3mm de espesor, con cifras de prdidas del orden de
0,3W/kg, con las que se alcanza inducciones de 1,7 a 1,9T.

3-43

captulo 3: transformador

Ncleo
Bobina primaria

Bobina secundaria

Figura 3.3.1.Esquema del transformador


tcnico de dos devanados
La alta permeabilidad del ncleo hace que ste se constituya en camino preferencial
para el flujo, por lo que la mayor parte de ste se cierra a travs del ncleo, enlazando
as a ambos devanados. Pero como la permeabilidad del ncleo no es infinita, tambin
habr flujo por el aire.
Debido a la extensin geomtrica de las bobinas, una parte del flujo por el aire tambin
est enlazado con ambas bobinas, pero su efecto es insignificante, comparado con el
del flujo en el ncleo. Este hecho autoriza a identificar el flujo en el ncleo con el flujo
comn m definido en el esquema de acoplamiento inductivo.
Si bien la parte del flujo por el aire que enlaza a ambos devanados es despreciable en
comparacin con el flujo por el ncleo, no lo es en absoluto en comparacin con la
totalidad del flujo por el aire. En consecuencia no es posible identificar el flujo por el
aire con el flujo de dispersin del esquema de acoplamiento inductivo. Dispersin
magntica y dispersin inductiva son conceptos diferentes.
Slo si se anula el flujo comn, lo que de acuerdo con (3.2.10) ocurre si N1i1 + N2 i2 = 0 ,
el aire estara ocupado exclusivamente por flujo de dispersin. Esta situacin se da
aproximadamente en cortocircuito.

3.3.1 Circuito equivalente


La definicin del esquema de acoplamiento inductivo y la posterior identificacin del
flujo comn de ese esquema con el flujo por el ncleo del transformador de potencia ha
convertido la obtencin del circuito equivalente del transformador en un ejercicio casi
rutinario, si se considera la metodologa desarrollada en el captulo 2.

3-44

captulo 3: transformador

V2

V1
1

N1

N2

Figura 3.3.2.Esquema de acoplamiento inductivo


del transformador.
En la figura 3.3.2 se reproduce el dibujo en corte del transformador superponindole el
esquema de acoplamiento inductivo definido en el prrafo anterior. De l se aprecia
claramente la topologa del circuito magntico ideal de la figura 3.3.3.
m
R1

R2
2
N1

2
N2

Figura 3.3.3. Circuito magntico ideal del transformador


Tal como se hizo en el caso del reactor, las bobinas reales, de N1 y N2 vueltas
respectivamente, se reemplazan por bobinas ideales, sin prdidas, en serie con las
cuales se conectan sendas resistencias, cuyo valor es tal que las prdidas generadas
en ellas sean iguales a las prdidas que se producen en las bobinas reales.
Si ahora se incluye la energa magntica asociada a cada campo de dispersin
mediante una impedancia magntica conservativa por la cual circula el correspondiente
flujo y la energa magntica y las prdidas en el ncleo mediante sendas impedancias
magnticas, conservativa y disipativa respectivamente, en serie con el flujo comn, se
logra el circuito electromagntico de la figura 3.3.4

3-45

captulo 3: transformador

R1

Rfe

Lm

N1

N1

1
N1

R2

2
N2

L2 N2

N1 L1

Figura 3.3.4. Circuito electromagntico del transformador


con dispersin y prdidas.
m
R1

V1

L2

L1
Lm

Rfe

N1

R2

N2

V2

Figura 3.3.5 Reduccin del circuito electromagntico.

La reduccin de este circuito electromagntico segn las reglas vistas en el captulo 2.


lleva al circuito de la figura 3.3.5, donde el transformador real, con dispersin y
prdidas, aparece reemplazado por un transformador ideal, con ncleo de
permeabilidad infinita, sin dispersin ni prdidas, en cuyo primario y secundario estn
conectados elementos concentrados que representan los efectos ausentes en el
transformador ideal. El conjunto formado por el transformador ideal y los circuitos
elctricos en el primario y en el secundario es equivalente al transformador real al que
reemplaza.
Es costumbre hacer aparecer todas las resistencias e inductancias en un solo circuito
acoplado galvnicamente. Para ello basta reemplazar las dos impedancias magnticas
en serie de la figura 3.3.5 por una equivalente, con bobina ideal de N1 vueltas, cuya
fuerza magnetomotriz es igual a la suma de las fuerzas magnetomotrices
correspondientes a cada una de las impedancias.

3-46

captulo 3: transformador

N12 N22
1
2 1
F = F1 + F2 = ( Z m 1 + Z m 2 ) = j
+
= j N1 + 2

Z 1 N1
Z1 Z 2
Z

2
N22

(3.3.1)

Se aprecia que la impedancia magntica equivalente posee N1 vueltas, a cuyos


terminales est conectada la impedancia Z1 en paralelo con la impedancia modificada
N
Z 2 = Z 2 1
N2

(3.3.2)

conocida como la impedancia del secundario reducida al primario.


En el caso especfico de la figura 3.3.5, la impedancia vale:
2

V N
V
Z 2 = R2 + j X 2 + 2 1 = R2 + j X 2 + 2
I2 N2
I 2

donde
V2 =

N1
V
N2 2

(3.3.3)

(3.3.4)

se conoce como la tensin secundaria referida al primario y es la tensin inducida por


el flujo comn en la bobina de N1 vueltas e
I2 =

N2
I
N1 2

(3.3.5)

se conoce como la corriente secundaria referida al primario y es la corriente que en la


bobina de N1 vueltas produce la misma fuerza magnetomotriz que la corriente I2 en la
bobina de N2 vueltas.

El cuociente
n=

N1
N2

se conoce como relacin de transformacin del transformador.

(3.3.6)

3-47

captulo 3: transformador

Como resultado de la reduccin descrita se obtiene el circuito equivalente del


transformador referido al primario, representado en la figura 3.3.6, en la que se ha
suprimido la bobina ideal, ya que la corriente por ella es nula.
I 2

I1
R1
V1

j1

X1

X2
jm Xm

Rfe

R2
j2

V2

Figura 3.3.6. Circuito equivalente galvnico del transformador.

Ntese que en el proceso de reduccin de los parmetros del secundario al primario


estos se transformaron de manera que la potencia disipada y la potencia reactiva
permanezcan invariantes:
I 22 R2 = I 2 2 R2

I 22 X 2 = I 2 2 X 2

(3.3.7)

El circuito equivalente aqu derivado es el punto de partida para el anlisis de las


caractersticas de funcionamiento del transformador en estado sinusoidal estacionario.
3.3.2 Diagrama fasorial
Para el anlisis del funcionamiento en estado sinusoidal estacionario se recurre
convenientemente a la representacin de las variables en el dominio de frecuencias a
travs de la transformacin fasorial.
Las variables transformadas admiten una representacin grfica en el plano complejo
que se conoce como diagrama fasorial y que representa un modelo matemtico
equivalente a las ecuaciones de Kirchhoff.
La fundamentacin terica del mtodo fasorial fue desarrollada en el curso de redes,
por lo que aqu slo se insistir en la importante cuestin de los sentidos y polaridades
de referencia, sin las cuales un diagrama fasorial queda ambiguo.
Tensin y corriente son magnitudes alternas peridicas cuyo sentido cambia con cada
semiciclo. Se dice que la tensin o corriente es positiva cuando su sentido coincide con
una direccin de referencia establecida arbitrariamente como positiva y que es negativa
cuando su sentido es opuesto a la direccin de referencia.
Antes de poder establecer una relacin coherente entre las variables de un circuito es
pues necesario fijar las referencias positivas para la tensin y la corriente en cada
elemento, lo que se hace con las flechas de referencia usuales.

3-48

captulo 3: transformador

Existen dos combinaciones de referencias posibles:


La corriente positiva entra al elemento por el terminal positivo, lo que implica
considerar a la potencia absorbida por el elemento como positiva. Se habla de
convencin carga.
La corriente positiva sale del elemento por el terminal positivo, lo que implica
considerar a la potencia entregada por el elemento como positiva. Se habla de
convencin fuente.
Ambos sistemas de referencia son equivalentes y la eleccin de uno u otro es un
asunto de conveniencia.
Histricamente la convencin carga ha tenido una difusin ms amplia y suele ser
preferida por ese motivo. Esta preferencia conduce a expresiones como en una
inductancia la corriente est atrasada respecto a la tensin en 90 , que slo tienen
sentido si se explicita el sistema de referencia usado y que sin esa informacin
adicional son ambiguas.
Para aclarar esto considrese una inductancia con referencias correspondientes a la
convencin fuente. Cuando la corriente pasa por cero, la energa acumulada en la
inductancia tambin vale cero. Por lo tanto, durante el primer cuarto de ciclo que sigue
al paso de la corriente por cero el elemento absorbe energa de la fuente, energa que
es transferida al campo magntico. Debido al uso de la convencin fuente, la potencia
absorbida por el elemento es considerada negativa. Durante el segundo cuarto de ciclo,
la energa acumulada en el campo es devuelta a la fuente, lo que implica que en el
segundo cuarto de ciclo la potencia es positiva. Si durante el primer semiciclo la
i

V
t

L
0

Figura 3.3.7

Relacin de fase entre tensin y corriente en una


inductancia con convencin fuente algebraica.

corriente es positiva, el signo de la potencia exige que durante el primer cuarto de ciclo
la tensin tiene que ser negativa y que durante el segundo cuarto de ciclo debe ser
positiva. Esta relacin es satisfecha por una tensin que corresponde a una
cosinusoide negativa, lo que en trminos fasoriales significa que la corriente est
adelantada a la tensin en 90. La figura 3.3.7 ilustra la relacin descrita.

3-49

captulo 3: transformador

Del anlisis anterior se desprende que es imprescindible la fijacin de la referencia


positiva para tensin y corriente en cada elemento y si bien esto puede hacerse en
forma arbitraria, resulta conveniente usar sistemticamente el mismo sistema de
referencia para todos los elementos del circuito.
Con este prembulo, considrese ahora la construccin del diagrama fasorial del
transformador, para lo cual se fija convenientemente las referencias en la forma
indicada en la figura 3.3.8.
I1

R1

X1
Im

V1

Xm

X2

I0

I 2

R2

Ic

I fe

Vi RFe

V2

Figura 3.3.8. Circuito equivalente con carga referido al primario.

Debido a la gran diferencia entre los mdulos de los fasores por representar, slo tiene
sentido construir un diagrama cualitativo, cuya construccin comienza
convenientemente en la impedancia de carga conocida, supuesta hmico-inductiva, y
que fija una determinada relacin de fase entre tensin y corriente.
Para las referencias consideradas, la corriente Ic est atrasada respecto a la tensin
V2 en un ngulo menor que 90, por lo que la corriente I2 , cuya referencia es opuesta
a la de Ic , debe estar adelantada respecto a V2 en un ngulo mayor que 90. La cada
de tensin en la resistencia R2 est en fase con la corriente I2 , mientras que la cada
de tensin en la reactancia inductiva X 2 est adelantada en 90 respecto a esa
corriente. Restando estas cadas de tensin fasorialmente de la tensin V2 se logra la
tensin inducida por el flujo comn Vi .
La corriente magnetizante Im est atrasada en 90 respecto a Vi , mientras que la
corriente de prdidas Ife est en fase con Vi . La suma de Im y de Ife da lugar a la
corriente de vaco I 0 .
La corriente I1 se encuentra aplicando la ley de nodos de Kirchhoff al nodo a del
circuito equivalente, es decir, restando fasorialmente I2 de I 0 .
Ahora se puede determinar las cadas de tensin en X1 y en R1 , que sumadas a Vi ,
permiten determinar V1 . El diagrama fasorial del transformador est completo.

3-50

captulo 3: transformador

V1

I1R1

jI1X 1
Vi

jI2 X 2

I 2R2
V2
1

I1

I 2

I0
Im

IFe

I2
Figura 3.3.9. Diagrama fasorial del transformador
con carga hmica inductiva.
La aplicacin consecuente de un sistema de referencia a todos elementos y puertas del
circuito equivalente hace que el diagrama fasorial sea ms transparente . En el caso de
aplicar la convencin carga, en las puertas que absorben potencia el ngulo de fase
entre la tensin y corriente correspondientes es menor que 90 , mientras que en las
puertas que entregan potencia el ngulo de fase es mayor que 90. En el diagrama
fasorial de la figura 3.3.9 , dibujado con las referencias de la figura 3.3.8, se aprecia
que el transformador, visto desde la red, es una carga, mientras que visto desde la
carga es una fuente.

3-51

captulo 3: transformador

3.3.3 Funcionamiento en vaco


Se dice que el transformador funciona en vaco cuando sus terminales primarios estn
conectados a la red y sus terminales secundarios estn abiertos. En esas condiciones
la corriente en el devanado secundario es nula y el transformador se comporta como un
reactor. El circuito equivalente se reduce al de la figura 3.3.10, para el que rige el
diagrama fasorial de la figura 3.3.11.
I1

I 2 = 0

L1

R1

I Fe

Im
V1

Lm

RFe

V2

Figura 3.3.10.Circuito equivalente galvnico en vaco.


Para tensiones aplicadas iguales o menores que la tensin nominal el grado de
saturacin del ncleo es moderado y la corriente de vaco es muy pequea (< 1% de la
corriente nominal para ncleos con chapas de grano orientado ) y reactiva.
En consecuencia, en vaco las prdidas en el devanado primario son tambin muy
pequeas, por lo que predominan las prdidas en el fierro. Prdidas en vaco y
prdidas en el fierro pasan a ser sinnimos.
Por lo pequeo de la corriente de vaco, las cadas de tensin en la resistencia y la
reactancia de dispersin primaria son muy pequeas en relacin con la tensin
aplicada, por lo que rige aproximadamente:
V1
4,44 f N1 m
N
=
= 1 =n
V20 4,44 f N2 m N2

(3.3.8)

donde V20 es la tensin inducida en vaco en el devanado secundario. Esta


proporcionalidad se usa para determinar experimentalmente la relacin de
transformacin n.

3.3.4 Funcionamiento en cortocircuito estacionario


Se dice que un transformador funciona en cortocircuito cuando los terminales del
devanado primario estn conectados a la red y los terminales del devanado secundario
estn cortocircuitados (Zc = 0).

3-52

captulo 3: transformador

En esas condiciones la corriente absorbida suele ser tan alta, que, en comparacin, la
corriente en la rama de magnetizacin puede ser despreciada. El circuito equivalente
se reduce al de la figura 3.3.12 para el cual rige el diagrama fasorial de la figura 3.3.13.
En cortocircuito con tensin reducida las prdidas en el fierro disminuyen
cuadrticamente con la tensin inducida, por lo que pueden considerarse despreciables
en comparacin con las prdidas en los devanados. Prdidas en cortocircuito es
sinnimo de prdidas en los devanados.

Icc

Re

Vcc

Figura 3.3.12. Circuito equivalente del


transformador en cortocircuito.

Vcc

Icc
cc

Figura 3.3.13. Diagrama fasorial de un


transformador en cortocircuito.

Al despreciar la corriente en la rama de magnetizacin queda:


I1 = I2

(3.3.9)

lo que equivale a i 1 N1 + i 2 N2 = 0 , por lo que, de acuerdo con (3.2.10), el enlace de


flujo comn se hace cero y el flujo en el aire corresponde en buena aproximacin al
flujo de dispersin. Este hecho se aprovecha para calcular la reactancia de dispersin
total, o de cortocircuito, X , a partir de la geometra de las bobinas.

3-53

captulo 3: transformador

Los parmetros del circuito equivalente de la figura 3.3.12


X = X 1 + X 2

Re = R1 + R2

(3.3.10)

se pueden determinar a partir de mediciones de tensin, corriente y potencia en el


transformador cortocircuitado, excitndolo con tensin reducida. Estos parmetros son
constantes, por lo que en cortocircuito la relacin entre tensin aplicada y corriente es
lineal (figura 3.3.14).

Vn

Vcc

In

I cc

Figura 3.3.14 Relacin entre tensin y corriente


en cortocircuito
Se define como corriente nominal a aquella corriente que en rgimen estacionario
determina un calentamiento del devanado igual al admisible para la clase de aislacin
usada en la construccin de las bobinas (60C).
Se define como tensin de cortocircuito a la tensin que hay que aplicar a los
terminales de entrada, con los terminales de salida cortocircuitados, para que la
corriente de entrada sea igual a la corriente nominal.
Vcc = I n Z e

(3.3.11)

con Z e = Re2 + X 2

(3.3.12)

En la prctica se prefiere entregar la tensin de cortocircuito como fraccin de la


tensin nominal, o tensin base, del devanado en que fue medida:
v cc =

Vcc
(pu)
Vn

(3.3.14)

captulo 3: transformador

3-54

Expresada en esa forma, la tensin de cortocircuito relativa es numricamente igual al


valor relativo de la impedancia de cortocircuito:
v cc =

I n Ze Z e
=
= z e (pu)
Vn
Zb

donde Z b =

Vn
In

(3.3.15)

(3.3.16)

es la impedancia base del transformador.


Se define como corriente de cortocircuito nominal a la que circula por los terminales de
entrada, estando los terminales de salida cortocircuitados, cuando la tensin aplicada
es igual a la tensin nominal.
I cc =

Vn
Ze

(3.3.17)

Expresada en por unidad, es decir, referida a la corriente nominal, o corriente base, la


corriente de cortocircuito nominal es igual al valor recproco de la tensin de
cortocircuito en (pu).
Icc
V
1
1
= n =
=
(pu)
In
I n Z e z e v cc

(3.3.18)

As, un transformador cuya tensin de cortocircuito es de 5%, o 0,05 pu, tiene una
corriente de cortocircuito nominal de 20 pu, es decir, de veinte veces la corriente
nominal.
Tambin existe una relacin directa entre las prdidas en los devanados, o prdidas en
el cobre, y la resistencia equivalente. Expresadas en por unidad, ambas magnitudes
son numricamente iguales:
pcu n =

Pcu n
Pn

I n2 Re Re
=
=
= re (pu)
I n Vn Z b

(3.3.19)

3.3.5 Funcionamiento con carga


Transformadores de potencia se operan normalmente en redes de tensin y frecuencia
aproximadamente constantes, lo que implica que el flujo en el ncleo

3-55

captulo 3: transformador

V1
4,44 f N1

y, por lo tanto la saturacin, tambin es constante. Como se mencion anteriormente,


la corriente de vaco en esta condicin es muy pequea y puede ser despreciada frente
a la corriente de carga, por lo que el circuito equivalente se reduce al de la figura
3.3.15.
*

Re

V1

V2

Figura 3.3.15. Circuito equivalente simplificado para la


determinacin de la regulacin
Sobre la base de este circuito equivalente se puede determinar la variacin de la
tensin en el secundario a plena carga , o carga nominal, en relacin con el
correspondiente valor en vaco. Esta variacin se conoce como la regulacin del
transformador:
=

V20 V2
V20

(3.3.20)

Para obtener una expresin explcita para la regulacin en trminos del ngulo de fase
de la carga y de los parmetros del transformador, considrese el diagrama fasorial
correspondiente al circuito equivalente de la figura 3.3.15, representado en la figura
3.3.16.
Del diagrama fasorial se tiene que
V = V20 V2

(3.3.21)

y, de acuerdo con Pitgoras, que


V2 = V202 V 2 V

Reemplazando esta ltima expresin en (3.3.21) queda:

(3.3.22)

3-56

captulo 3: transformador

V = V + V20 1 1

V20

(3.3.23)

V
V1

V
Vr

V2
V1 = V20

V2

Figura 3.3.16 Diagrama fasorial


para la determinacin de
la regulacin.
Considerando que normalmente
2

<< 1 ,
V20
se puede aproximar la raz cuadrada mediante los dos primeros primeros trminos de
su desarrollo en serie de potencias:
2

V
1 V
1
1
+ ... ,
2 V20
V20
por lo que (3.3.23) se reduce a

3-57

captulo 3: transformador

V = V +

V 2

(3.3.24)

2V20

y (3.3.20) toma la forma:

Figura 3.3.17
Regulacin del transformador,
lnea llena (roja): frmula exacta (3.3.27),
lnea segmentada: frmula aproximada (3.3.29).

V
V20

V
V20

1 V
+
.
2 V20

(3.3.25)

Del diagrama fasorial de la figura 3.3.16 se desprenden las siguientes relaciones:


V = Vr cos + V sen
V = Vr sen + V cos

(3.3.26)

Dividiendo (3.3.26) por V20 y reemplazando el resultado en (3.3.25) se logra finalmente


la siguiente relacin para la regulacin:
= v r cos + v sen +

1
2
v r sen v cos ]
[
2

(3.3.27)

3-58

captulo 3: transformador

donde v r =

Vr
(pu)
V20

v =

V
(pu)
V20

son las cadas de tensin, en (pu), en la resistencia equivalente y la reactancia de


dispersin respectivamente. En virtud de la relacin (3.3.15) rige:
v r = r (pu)
y
v = x (pu)
(3.3.28)
por lo que (3.3.27) es la expresin buscada para la regulacin en trminos de los
parmetros y del ngulo de fase de la carga.
En la figura 3.3.17 el trazo con lnea continua corresponde a la evaluacin de (3.3.27),
mientras que el trazo con lnea segmentada corresponde a la expresin aproximada
= r cos + x sen

(3.3.29)

Se aprecia que para cargas capacitivas (<0) la regulacin puede resultar negativa, lo
que significa que la tensin secundaria aumenta con la carga por sobre el valor de
vaco.

4-59

captulo 4 : devanados

4. Devanados

4.1

Introduccin

Las ideas y conceptos desarrollados en los captulos precedentes son plenamente


aplicables al transformador de ncleo acorazado representado esquemticamente en la
figura 4.1.1. La inclusin de entrehierros en la columna central slo repercute en un
aumento de la corriente magnetizante y no cambia la naturaleza del aparato.

Figura 4.1.1. Transformador acorazado con un grado de


libertad mecnico.
Los rasgos topolgicos propios del dispositivo de la figura 4.1.2 - estructura cilndrica
concntrica, conductores alojados en ranuras practicadas en las superficies cilndricas no alcanzan a ocultar su identidad esencial con el transformador de la figura 4.1.1.
x1

R

fe=
Hfe=0

Sin embargo, la girabilidad del devanado


secundario - rotor - respecto al devanado
primario - estator - introduce un grado de libertad
adicional que hace de este dispositivo algo ms
que un transformador, convirtindolo en el
prototipo de las mquinas elctricas rotatorias,
cuyas caractersticas de funcionamiento se
explican en los captulos siguientes a partir del
campo magntico en el entrehierro.

Figura 4.1.2. Transformador giratorio Interesa entonces determinar la distribucin


como prototipo de la
espacial del campo en el entrehierro a partir de
mquina elctrica.
las corrientes que circulan en los conductores

4-60

captulo 4 : devanados

alojados en las ranuras del estator y del rotor.


El deseo de lograr una solucin analtica hace necesario modelar el problema e
introducir algunas suposiciones simplificadoras. En este sentido, se considera que el
campo en el entrehierro es homogneo en sentido axial, desprecindose el efecto de
los extremos. Igualmente se considera que la variacin del campo en direccin radial es
despreciable si el ancho del entrehierro es mucho menor que el radio interior del
estator.
Las consideraciones planteadas reducen el problema de campos tridimensional a uno
unidimensional equivalente. El campo en el entrehierro es considerado como un campo
unidimensional que slo depende de la coordenada tangencial. Este modelo permite la
obtencin de soluciones analticas simples y suficientemente exactas para fines
prcticos.
Estas soluciones se hacen ms transparentes si se considera en forma independiente la
determinacin del campo en el entrehierro creado por las corrientes en los devanados y
la determinacin de las tensiones inducidas por el campo en el entrehierro en los
devanados. Esto, sin perder de vista que en la mquina real las leyes de Ampere y de
Faraday se cumplen simultneamente.
4.2

Corrientes y campo magntico en el entrehierro

En las mquinas prcticas las ranuras estn distribuidas en forma regular a lo largo de
la periferia del estator o del rotor. Por ello resulta conveniente tratar primeramente el
caso de la corriente en una ranura, cuyo resultado se puede generalizar a distribuciones
de cualquier nmero de ranuras mediante la aplicacin del principio de superposicin.
Considrese
entonces
la
situacin
representada en forma esquemtica en la
figura 4.2.1, donde circula corriente por una
sola ranura infinitamente estrecha alojada en
un medio magntico de permeabilidad infinita.

La situacin en el estator es similar a la ya


analizada en el captulo 1, cuando se introdujo
la nocin circuito magntico para el ncleo
toroidal de alta permeabilidad y excitacin
concentrada: el flujo se cierra preferentemente
por el estator, desvindose slo una pequea
Figura 4.2.1. Modelo del devanado
fraccin a travs del entrehierro.
con una ranura.
Si el flujo ( que tiende a infinito ) se supone

4-61

captulo 4 : devanados

restringido al yugo del estator, las lneas de fuerza tienen que ser circunferencias
concntricas, a lo largo de las cuales -por razones de simetra- la intensidad del campo
magntico Hy es constante.
Por otra parte, el flujo en el rotor es finito, por lo que la induccin tambin es finita y la
permeabilidad infinita del fierro hace que la intensidad del campo magntico en el rotor
sea nula.
En consecuencia, la aplicacin de la ley de Ampere a lo largo de una de las lneas de
fuerza circulares de radio r del estator permite escribir:
i
(4.2.1)
2r
y su aplicacin al camino de integracin de la figura 4.2.2 establece que
H y ( x) =

H y r x + f (x ) f (0 + ) = 0 ,

(4.2.2)

donde f(x) es la fuerza magnetomotriz (fmm) correspondiente a la coordenada angular


x. El origen de x coincide con la ubicacin de la ranura por la que circula la corriente i.
x1

H
H

i
r

Figura 4.2.2. Distribucin esquemtica del campo debido


a la corriente en una sola ranura ubicada en
el estator.
De (4.2.2) se despeja
f ( x ) = f (0 + )

i x
2

(4.2.3)

Como el flujo neto que entra a la superficie del rotor debe ser nulo, debe cumplirse que

4-62

captulo 4 : devanados

f ( x )dx = 0

(4.2.4)

de donde se desprende que f ( 0 + ) =


f (x ) =

i
, con lo que
2

i
x
1
2

(4.2.5)

Se aprecia que la aplicacin elemental de la ley de Ampere permite establecer que la


distribucin de fmm causada por la corriente en una ranura corresponde a la funcin
diente de sierra representada en la figura 4.2.3.
F(x)

i
2

x
0

Figura 4.2.3. Distribucin a lo largo del entrehierro de la componente


radial de la fmm debida a la corriente de una sola ranura.
Esta funcin es peridica (con perodo 2), pero discontinua, por lo que se obtiene
ventajas analticas reemplazndola por su desarrollo en serie de Fourier:
f( x ) =

i 2 senx

2 =1

( = 1,2,3,.....)

(4.2.6)

En la figura 4.2.4 estn representados los primeros tres trminos del desarrollo, que
junto con la fundamental exhibe armnicas pares e impares.
Cada bobina tiene dos lados, alojados en sendas ranuras, separadas en un paso de
bobina. El paso de bobina corresponde al arco menor que media entre las dos ranuras y
en la prctica se expresa en nmero de ranuras (p.ej. 7 1-8). En un devanado de una
capa, que se caracteriza por tener un solo lado de bobina en cada ranura, el paso de
bobina est determinado por la periodicidad deseada para la distribucin espacial de
fmm.

4-63

captulo 4 : devanados

i 2
f (x ) =
2

F(x)

sen x

=1

=1

i
2

0
=3

=2

Figura 4.2.4. Representacin del desarrollo en serie


de Fourier de la funcin diente de sierra.
Considrese primeramente una bobina diametral (figura 4.2.5), cuya distribucin de
fmm se logra superponiendo las distribuciones correspondientes a dos ranuras
desplazadas en radianes, por las cuales circulan corrientes de igual magnitud pero de
signo opuesto (figura 4.2.6):
f (x ) =

i 2
2

sen x i 2

2
=1

sen ( x )

=1

(4.2.7)

x
R

p
R

Figura 4.2.5. Dos excitaciones de signo opuesto,


desplazadas en un paso polar, forman una
bobina de paso completo.

La distribucin resultante
i 4
f (x ) =
2

=1

sen x

( = 2g + 1

con g = 0, 1. 2, 3,.....)

(4.2.8)

4-64

captulo 4 : devanados

corresponde a una forma de onda rectangular (figura 4.2.6) que slo contiene
armnicas impares, ya que las armnicas pares de las dos distribuciones en (4.2.7) se
cancelan. Su componente fundamental tiene perodo 2, el mximo valor posible.

p
R
i
2

Figura 4.2.6. Distribucin de fmm de una


bobina de paso completo
obtenida por superposicin.

2'

1'

2
Figura 4.2.7. Distribucin de las excitaciones
magnticas para un devanado
de p=2 pares de polos.

Para disminuir el perodo a la


mitad es necesario aumentar los
puntos de excitacin alternados
- caracterizados por puntos y
cruces,
que
representan
respectivamente corrientes que
salen del y que entran al plano del
dibujo - al doble. La figura 4.2.7
muestra como se logra esta
distribucin mediante el empleo
de dos bobinas (1-1) y (2-2)
desplazadas en radianes, cuyos
pasos de bobina
han sido
reducidos a la mitad, es decir, a
/2 radianes. Las dos bobinas
deben conectarse elctricamente
en serie (1 con 2) para garantizar
la igualdad de las corrientes por
ellas.

La figura 4.2.8 muestra la distribucin de fmm correspondiente. Se aprecia que la fmm


es constante entre puntos de excitacin y salta en estos en un monto igual a la
excitacin (corriente total en la ranura). El valor medio de la distribucin de fmm es cero.
Estos resultados son generales y permiten trazar directamente la distribucin de fmm
correspondiente a una distribucin de corrientes dada. Se comienza en cualquier ranura
y se registran los saltos correspondientes a cada ranura con corriente, considerando el

4-65

captulo 4 : devanados

signo de esta. El resultado es una onda escalonada cuyo valor medio es cero. El eje de
abscisas se traza de manera que las reas sobre y bajo l sean iguales.
F(x)

2'

/2

1'
3/2

Figura 4.2.8. Distribucin de la fmm de un


devanado de 2 pares de polos.
La expresin analtica para la distribucin resultante se logra superponiendo las
distribuciones correspondientes a los 2p=4 puntos de excitacin, desplazados
relativamente en /2 radianes:
i 2
f (x ) =
2

(sen x sen (x 2 ) + sen (x ) sen (x


=1

3
))
2

(4.2.9)

donde = 1, 2, 3,....
El resultado
f( x ) =

i 4 p
senx con = p( 2g + 1) g = 0,1,2,3,......
2 =2

(4.2.10)

permite apreciar que ahora la armnica de orden ms bajo es =p=2, cuyo perodo es
2/p=, que se repite p veces a lo largo de la periferia. Esta componente se conoce
como la fundamental.
Dado que a la fundamental le corresponde la longitud de onda mayor, es costumbre
normalizar su perodo a 2 y expresar las armnicas en relacin a ella. Para ello se
introduce una nueva unidad de medida angular, el radin elctrico, definido por la
relacin
2p[rad el ] = 2[ rad geom ]

(4.2.11)

4-66

captulo 4 : devanados

que establece la equivalencia:


1 rad geom = p rad el

(4.2.12)

donde p es el nmero de perodos de la fundamental a lo largo de la periferia, o, en la


jerga elctrica, el nmero de pares de polos del devanado.
En trminos del arco medido en radianes elctricos la relacin (4.2.10) se reescribe
como

i 4
sen x
f (x ) =
2 = 1

donde =

= 2g + 1
p

con g = 0, 1, 2, 3,....

(4.2.13)

es el nmero de orden de la armnica en relacin con la fundamental.


Para la fundamental se tiene la relacin:
f 1( x ) =

i 4
sen x
2

(4.2.14)

La comparacin de (4.2.8) con (4.2.13) lleva a la conclusin que el uso de la medida


angular radianes elctricos reduce el anlisis de una distribucin de periodicidad p al
anlisis de una distribucin de periodicidad 1. En otras palabras, en trminos de
radianes elctricos el problema de la figura 4.2.8 se reduce al problema de la figura
4.2.6.
En los desarrollos que siguen se asume tcitamente que los arcos estn expresados en
radianes elctricos, salvo que se indique explcitamente otra cosa.

4.2.1 Bobinas acortadas, el factor de cuerda


La unidad prctica de un devanado es la bobina. El lado de bobina, como unidad del
devanado, slo es una abstraccin que facilita la determinacin sistemtica de la
distribucin de fmm en el entrehierro. Siempre existen al menos dos lados de bobina
excitados por corrientes de signo opuesto. En consecuencia, los trminos creados para
describir propiedades de la distribucin de los conductores a lo largo de la periferia se
refieren siempre a la bobina.
En las distribuciones de fmm de las figuras 4.2.6 y 4.2.8 se puede distinguir
semiperodos de radianes elctricos que coinciden con los de la fundamental. Los
semiperodos de la fundamental se denominan polos. El arco cubierto por un polo se
conoce como paso polar y siempre corresponde a radianes elctricos. En las figuras

4-67

captulo 4 : devanados

4.2.5 y 4.2.7 el paso polar es igual al paso de bobina y se habla de bobinas de paso
completo.
Considrese ahora la situacin
representada esquemticamente en
la figura 4.2.9, donde los lados de la
bobina no se encuentran sobre un
dimetro, sino sobre una cuerda. La
bobina es de paso acortado.

x
i

p
R

La distribucin de fmm resultante

para esta disposicin de corrientes


se
logra
superponiendo
las
distribuciones de diente de sierra Figura 4.2.9. Dos excitaciones desplazadas en
un paso de bobina ( b ) menor que
correspondientes a los respectivos
el paso polar ( p ), forman una
lados de bobina , segn se muestra
bobina de paso acortado.
en la figura 4.2.10.

Analticamente se tiene que


i 2
f (x ) =
2
i 4
f (x ) =
2

sen x i 2

2
=1

sen ( x + )

=1

sen ( x + 2)

cos
=1

donde = 2g + 1

(4.2.15)

(4.2.16)

con g = 0,1, 2, 3,...

Al comparar ahora (4.2.16) con (4.2.8) se aprecia que, como consecuencia del
acortamiento del paso de bobina, las
amplitudes de las armnicas resultan
ponderadas con el factor de cuerda
f(x)

-
0
b/R


f c = cos 1
2

(4.2.17)

Figura 4.2.10 Distribucin de fmm de una


bobina acortada

En el acortamiento del paso de


bobina el diseador de un devanado
tiene una poderosa herramienta para
atenuar fuertemente las amplitudes
de ciertas armnicas al costo de una
leve disminucin de la amplitud de la
fundamental.

4-68

captulo 4 : devanados

Por ejemplo, suponiendo que se pueda realizar el acortamiento = / 6 , resultara


fc5=0,259, fc7=-0,259 y fc1=0,966, es decir, las amplitudes de la quinta y de la sptima
armnica se habran reducido aproximadamente a la cuarta parte del valor que tendran
en una bobina de paso completo, mientras que la amplitud de la fundamental slo se
habra reducido en 3,4%.

4.2.2 Bobinas distribuidas, factor de zona


Normalmente se pretende crear
en el entrehierro un campo cuya
distribucin espacial se aproxime
a una sinusoide. Para realizar
esta
distribucin
ideal
se
requerira un nmero infinito de
ranuras y corrientes diferentes en
cada ranura.

i
q


q=3

Si bien esta solucin no es


realizable en la prctica, sugiere
que se puede mejorar la forma de
onda de la distribucin espacial
de fmm aumentando el nmero de Figura 4.2.11. Devanado distribuido en q ranuras,
en cada una de las cuales circula la
puntos de excitacin. Se aprecia
q-ava parte de la corriente total.
subjetivamente que la distribucin
de la figura 4.2.12 es ms
sinusoidal que la de la figura 4.2.6.
Considrese entonces que la excitacin magntica est distribuida igualitariamente en q
ranuras, espaciadas regularmente en el ngulo , que cubren una zona q de la
periferia, como se muestra en la figura 4.2.11.
La expresin analtica correspondiente a la distribucin de fmm de la figura 4.2.12 se
logra nuevamente a travs de la superposicin de las distribuciones parciales de cada
ranura y se resume en la siguiente serie de Fourier:
f( x ) =

i 4 sen( q 2 ) sen ( x ( q 1) 2 )
2

=1 qsen( 2 )

con = 2g + 1

(4.2.18)

g = 0,1, 2, 3,...

Al comparar los coeficientes de Fourier de las expresiones (4.2.18) y (4.2.8) se constata


que, como consecuencia de la distribucin de la excitacin sobre una zona, las

4-69

captulo 4 : devanados

amplitudes de las armnicas resultan ponderadas con el factor de zona o de


distribucin:
fz =

sen( q 2)
1
q sen( 2)

(4.2.19)

f
i
q
x

Figura 4.2.12. Distribucin de un grupo de q bobinas de paso completo.

f z
1

0.9

0.9

0.3

0.9

0.3
0.2

1 3 5 7 9 11 13 15 17 19 21 23 25 27

Figura 4.2.13. Espectro del factor de zona para un


devanado distribuido con q=3 y =30.

Este factor tambin es una funcin peridica de y la figura 4.2.13 muestra el espectro
(valor absoluto del factor de distribucin como funcin del nmero de orden de las
armnicas) correspondiente a un caso especfico. Se aprecia que en general el factor
de zona afecta selectivamente a las diferentes armnicas, pero que tambin hay
armnicas que no son mayormente afectadas.

captulo 4 : devanados

4-70

Del anlisis anterior se concluye que mediante el uso juicioso del acortamiento y de la
distribucin es posible generar distribuciones espaciales de fmm esencialmente
sinusoidales en el entrehierro de las mquinas elctricas.

4.2.3 Devanados de corriente alterna


Las mquinas prcticas estn provistas de Z ranuras uniformemente distribuidas a lo
largo de la periferia del estator o del rotor, de manera que el ngulo entre ranuras
consecutivas est dado por
=

2 p
Z

(4.2.20)

El ngulo de acortamiento del paso de bobina es necesariamente un mltiplo entero


de .
El devanado ocupa todas las ranuras y est formado por bobinas de igual nmero de
vueltas, interconectadas de manera de producir la distribucin espacial de fmm
deseada.
En el caso que las bobinas sean de paso completo se requiere de Z/2 bobinas para
completar el devanado, ya que cada lado de bobina ocupa una ranura. Se le conoce
como un devanado de una capa.
Si en cambio se utiliza bobinas de paso acortado, el nmero de bobinas necesarias es
Z y cada ranura tiene que alojar a dos lados de bobina. Se habla de un devanado de
dos capas.
Cuando se trata de devanados para alimentacin polifsica las bobinas se
interconectan formando grupos que ocupan zonas simtricamente desplazadas a lo
largo de la periferia. Las corrientes en todas las bobinas de un grupo estn en fase, por
lo que esos grupos de bobinas se denominan fases.
La figura 4.2.14 representa en forma esquemtica una fase (en desarrollo) de un
devanado trifsico cuyo paso polar es de 9 ranuras (paso 1-10), formado por bobinas
acortadas cuyo paso de bobina es de 7 ranuras (paso 1-8), donde se puede apreciar
dos caractersticas de un devanado acortado: la existencia de las dos capas (lnea llena
corresponde al lado de bobina que se encuentra en la capa superior, lnea segmentada
corresponde al lado de bobina que se encuentra en la capa inferior) y la existencia de
dos grupos de bobinas, desplazados en un paso polar, que forman la fase. Tambin se
muestra la interconexin de estos dos grupos para producir la distribucin de fmm
deseada, caracterizada por la distribucin de puntos y cruces indicada en la vista en
corte de la parte superior de la figura.

4-71

captulo 4 : devanados

Como consecuencia del acortamiento del paso de bobina el nmero de puntos de


excitacin aument de 3 a 5 y la intensidad de la excitacin dej de ser la misma en
todos los puntos, con lo que la distribucin espacial de corriente de la fase se acerca en
cierta manera a la distribucin ideal mencionada al comienzo del prrafo 4.2.2. Esto
tambin se refleja en la mayor sinusoidalidad que se aprecia subjetivamente en la
distribucin de fmm de la parte inferior de la figura 4.2.14.
La correspondiente expresin analtica en la forma de una serie de Fourier puede
obtenerse en principio recurriendo a la superposicin de ondas diente de sierra, a
pesar de existir un procedimiento directo ms eficiente para el caso de las ondas
escalonadas como la de la figura 4.2.14.

a)

Capa superior
Capa inferior

b)

f(x)
iN
2 q

c)

0
iN
2

Figura 4.2.14. Distribucin de bobinas y fmm correspondientes a


una fase de un devanado trifsico.

Sea N el nmero total de vueltas en serie de las bobinas de una fase de un devanado
m-fsico distribuido en Z/m ranuras.
Entonces cada una de las Z/m bobinas tiene Nm/Z vueltas y el salto producido por un
lado de bobina en la distribucin de fmm es
fmm =

Nm
i
Z

(4.2.21)

captulo 4 : devanados

4-72

Con esto se determina la siguiente expresin para el desarrollo en serie de Fourier de la


onda de fmm correspondiente a una fase de un devanado distribuido y acortado:
f( x ) =

i N4
sen (q 2 ) sen (x ( q 1) 2 + 2 )
cos

2 p =1

2 q sen( 2)

(4.2.22)

donde se ha introducido convenientemente el nmero de ranuras por polo y por fase


q=

Z
2pm

(4.2.23)

Se aprecia que la amplitud de cada armnica est ponderada por un factor que es igual
al producto del factor de cuerda por el factor de zona y que se conoce como el factor de
devanado correspondiente a esa armnica:
f d = fc f z

(4.2.24)

Para un devanado concentrado ( f z = 1) de paso completo ( f c = 1) el factor de


devanado vale 1 y la amplitud de la -sima armnica de ese devanado est dada por:
Fc =

4 1 iN
2p

(4.2.25)

mientras que la amplitud de esa misma armnica para un devanado distribuido est
dada por:
Fd =

4 1 iNf d
.
2p

(4.2.26)

De estas dos expresiones se desprende que para una determinada armnica un


devanado distribuido puede pensarse reemplazado por un devanado concentrado
equivalente de Nfd vueltas.
El producto Nef = N f d
(4.2.27)
se denomina el nmero de vueltas efectivo del devanado para la -sima armnica.
Para apreciar el efecto de la distribucin y del acortamiento del devanado sobre la
forma de onda de la distribucin espacial de la fmm la siguiente tabla resume el caso en
que q=3, =2/18 y =2.

4-73

captulo 4 : devanados

fd
Fc
Fd

1
0,9019
1
0,9019

3
0,3333
0,3333
0,1111

5
0,0378
0,2000
0,0076

7
0,1359
0,1429
0,0194

9
0,3333
0,1111
0,0370

11
0,1359
0,0909
0,0124

Si se introduce como cifra de mrito el factor de distorsin armnica total, definido como
11

%THD = 100

= 3

(4.2.28)

F1

se obtiene

concentrado
distribuido y acortado
distr.y acort. sin tercera armnica

%THD
36,4
13,3
2,7

donde en la ltima fila se ha incluido el caso sin tercera armnica y sus mltiplos, que
corresponde a la situacin normal para devanados trifsicos.
Al compara las amplitudes de las fundamentales de las ondas de fmm se observa que
en este caso la reduccin del contenido armnico tuvo el costo de una disminucin de
la amplitud de la fundamental en casi 10%.
4.2.4 Campo giratorio mediante devanado trifsico
La alimentacin de un devanado trifsico simtrico con corrientes trifsicas simtricas
crea una distribucin espacial de fmm de amplitud constante que se desplaza a lo largo
del entrehierro con velocidad angular constante. Tal distribucin se conoce como un
campo giratorio y ocupa un lugar central en la teora de las mquinas de corriente
alterna.
Sean tres grupos de bobinas, similares al grupo de la figura 4.2.14, distribuidas
simtricamente a lo largo de la periferia interior del estator. La figura 4.2.15 representa
esta situacin en trminos de tres fases concentradas equivalentes, cuyo nmero de
vueltas es igual al nmero de vueltas efectivo para la fundamental.
La representacin de la figura 4.2.15a corresponde a un corte transversal a travs del
estator y la de la figura 4.2.15b corresponde al desarrollo del manto de cilindro interior
del estator. Las cruces y las flechas indican los sentidos de referencia positivos para las
corrientes en los conductores as marcados.

4-74

captulo 4 : devanados

Supngase ahora que cada fase est alimentada con una de las corrientes pertenecientes al sistema de corrientes trifsico simtrico de la figura 4.2.16.

2'

3
1

1'

3
3'

2' 1
a)

3' 2

1' 3

b)

Figura 4.2.15. Devanado trifsico elemental.

i
i1

i2

i3

2i

La figura 4.2.17 muestra la


distribucin espacial de fmm
resultante y la fundamental
correspondiente para los tres
instantes sucesivos, separados
en t=/6, marcados en la
figura 4.2.16.

Debido al desfasamiento entre


las
tres
corrientes,
la
combinacin de los valores
instantneos correspondientes
a cada instante representado es
diferente, lo que se traduce en
t1 t2 t 3
el corrimiento espacial relativo
Figura 4.2.16. Sistema de corrientes trifsico simtrico. de la onda de fmm observable
en la figura 4.2.17.
La expresin analtica para la
0

4-75

captulo 4 : devanados

fundamental de la onda de fmm resultante se logra superponiendo las fundamentales


asociadas a cada una de las tres fases:
f ( x, t ) = f1 ( x , t ) + f 2 ( x , t ) + f 3 ( x, t )

(4.2.29)

donde, con el origen para la coordenada x definido en la figura 4.2.15 en el eje


magntico de la fase 1, las componentes correspondientes a las tres fases estn dadas
por:

f(x,t)
t1
2' 1

3'

1'

3
x

t2

t3

Figura 4.2.17. Distribucin de la fmm giratoria para 3


instantes sucesivos.

f1 ( x , t ) =

2IN ef 1 4
cos x cos t
2p

f2 ( x ,t ) =

2IN ef 1 4
cos( x 2 3 )cos( t 2 3 )
2p

f3 ( x ,t ) =

2IN ef 1 4
cos( x 4 3 )cos( t 4 3 )
2p

(4.2.30)

4-76

captulo 4 : devanados

Considerando la relacin trigonomtrica


cos( x )cos( y ) =

1
2

[cos( x + y ) + cos( x y )]

(4.2.31)

la suma del segundo miembro de (4.2.29) se reduce a


f ( x, t ) =

3 2IN ef 1 4
cos( x t )
2 2p

(4.2.32)

La amplitud de la onda resultante es constante e igual a 3/2 veces la amplitud de una de


las componentes . La coordenada para la cual se produce el valor mximo se obtiene
de la condicin cos(x-t)=1, como
x = t

(4.2.33)

y se traslada a lo largo del entrehierro con velocidad angular constante


dx
=
dt

en

rad el / s

(4.2.34)

La estructura de la relacin (4.2.32) corresponde a la de la onda propagatoria de la


acstica, por lo que en este contexto, donde sepropaga en direccin tangencial en el
entrehierro, se la denomina onda o campo giratorio.
En cambio las relaciones (4.2.30) tienen su equivalente acstico en las ondas
estacionarias, caracterizadas por nodos espacialmente fijos. En el contexto
electromagntico estas expresiones se conocen como campos alternos.
Adems de la fundamental, las distribuciones de fmm de las tres fases contienen
armnicas. Las armnicas impares no mltiplos de la tercera producen campos
giratorios como la fundamental, en cambio las terceras armnicas (y sus mltiplos) se
anulan y no producen campos resultantes en el entrehierro, como ya se haba
adelantado al final del prrafo 4.2.3.
4.2.5 La distribucin de induccin en el entrehierro
Los prrafos precedentes se refieren a la determinacin de la distribucin de fmm en el
entrehierro como funcin de la coordenada angular x .
Dado que el modelo en que se basa el anlisis presupone que la permeabilidad del
fierro, tanto del estator como del rotor, tiende a infinito y que por lo tanto el valor de H
en el fierro es despreciable, la intensidad de la componente radial del campo magntico
en cualquier punto del entrehierro est relacionada con la fmm en ese punto a travs de
la relacin

4-77

captulo 4 : devanados

H( x, t ) =

f ( x, t )
( x, t )

(4.2.35)

donde (x,t) es el ancho radial del entrehierro para la coordenada x en el instante t.


Como la permeabilidad en el entrehierro es constante e igual a 0 , se tiene que
B( x, t ) =

0
f ( x, t )
( x, t )

(4.2.36)

Se aprecia que slo si la permeancia por unidad de superficie


( x, t ) =

0
( x, t )

(4.2.37)

es constante, como en el modelo de la figura 4.1.2, la onda de induccin es


proporcional a la onda de fmm. En cambio, si el entrehierro no es constante, como en el
modelo de la figura 4.1.1, una onda de fmm sinusoidal produce una onda de induccin
con armnicas.
4.3

Tensin inducida en un devanado

La ley de Faraday relaciona la tensin inducida en un circuito con la rapidez de la


variacin del flujo enlazado por ese circuito sin pronunciarse sobre el origen de la
variacin de flujo. Este puede estar en la variacin de la corriente en el propio circuito,
como en el caso del reactor, en la variacin de la corriente en otro circuito, como en el
caso del transformador o en el desplazamiento relativo de los circuitos, como en el caso
de los dispositivos de las figuras 4.1.1 y 4.1.2.
En este ltimo caso el flujo enlazado por los circuitos es tambin una funcin de la
coordenada angular que caracteriza la posicin relativa de los dos devanados.
En consecuencia se puede anotar para la tensin inducida en el devanado del estator
del dispositivo de la figura 4.1.2
v1 =

d
1 1 d
1 ( ,t ) =
+
dt
t
dt

(4.3.1)

donde la aplicacin de la regla de la cadena separ las dos causas de la variacin del
enlace de flujo.
El primer trmino se debe a la variacin del flujo causada por la variacin de las
corrientes con el tiempo y se conoce como tensin transformatrica :

4-78

captulo 4 : devanados

v 1t =

1
t

(4.3.2)

El segundo trmino se debe al movimiento relativo entre los dos circuitos, ubicados
respectivamente en el rotor y en el estator, y se conoce como tensin rotacional :
v 1 rot =

1 d
dt

(4.3.3)

El enlace de flujo 1 se determina a partir de la distribucin de induccin en el


entrehierro.
Supngase ahora que el devanado del rotor est excitado con corriente continua y que
produzca una distribucin de induccin cosinusoidal en funcin de una coordenada x2
cuyo origen coincide con el eje magntico del devanado:
b( x 2 , t ) = B cos x 2

(4.3.4)

Para un observador ubicado en el rotor se trata de una distribucin invariante en el


tiempo. Esta situacin no cambia si el rotor gira uniformemente con velocidad angular
=d/dt .
Sin embargo, para un observador fijo respecto al estator, cuyo sistema de coordenadas
x1 tiene su origen en el eje magntico de la bobina del estator y est relacionado con el
sistema de coordenadas del rotor a travs de la relacin (figura 4.1.2)
x1 = x 2 + = x 2 + t ,

(4.3.5)

ese campo se ve como un campo giratorio


b( x 1, t ) = B cos( x 1 t )

(4.3.6)

y ese observador interpretara la tensin inducida en la bobina del estator como una
tensin rotacional debida al movimiento del campo giratorio respecto a la bobina.
Supngase ahora que el devanado del rotor est abierto y que la bobina del estator est
alimentada con una corriente alterna de frecuencia angular , que produce un campo
alterno cuya fundamental est descrita por
b( x1 , t ) = Bcos t cos x1

(4.3.7)

Para el observador fijo respecto al estator la tensin inducida por este campo en el
devanado del estator sera una tensin transformatrica.

4-79

captulo 4 : devanados

Pero en virtud de la relacin trigonomtrica (4.2.31) la relacin (4.3.7) se puede


reescribir como
b( x 1, t ) =

B
B
cos( x1 t ) + cos ( x 1 + t )
2
2

(4.3.8)

e interpretar como la superposicin de dos campos giratorios cuya amplitud es igual a la


mitad de la amplitud del campo alterno y que giran con frecuencia angular en sentidos
contrarios.
Frente a esta realidad el observador fijo respecto al estator puede cambiar de opinin
y sostener que la tensin en la bobina es la superposicin de dos tensiones
rotacionales.
De estas consideraciones se desprende que la determinacin de la tensin inducida en
un devanado puede reducirse a la determinacin de la tensin rotacional debida a uno o
ms campos giratorios, punto de vista que se adopta en lo que sigue.

4.3.1 Tensin inducida en una bobina de paso completo


Considrese una bobina (1-1) de paso completo ( b = p ) formada por N vueltas
alojadas en dos ranuras de un estator cuyo radio interior sea R y cuya longitud axial sea
l.

(t+1)

b(x,t)
B mcos(x 1 -t- 1 )

x1

x1

R
Figura 4.3.1.

Flujo enlazado por una bobina de paso


completo.

4-80

captulo 4 : devanados

En el entrehierro exista un campo giratorio de origen cualquiera


b( x 1, t ) = Bm cos( x 1 1t 1 )

(4.3.9)

El flujo enlazado por la bobina (1-1), representada esquemticamente en la figura 4.3.1,


se calcula integrando la densidad de flujo b( x 1, t ) sobre la superficie abrazada por la
bobina, limitada por la longitud axial del estator y por las coordenadas x1 = -/2 y x1 =
+/2 . Al formar el elemento de rea debe considerarse el arco en radianes geomtricos
(x1/p). As se logra
+ /2

(t ) = N

b(x ,t ) lRd ( x
1

p ) = N 2Bm

/2

p =

lR
cos( 1 t + 1 )
p

2
Bm p l

Bm

2
Bm

p =

R
p

Figura 4.3.2. Interpretacin geomtrica del


flujo por polo.
m = N p

(4.3.10)

Si se invoca la interpretacin
geomtrica de la integral como
rea bajo la curva, el flujo
enlazado por la bobina es
proporcional al rea neta bajo
la onda de induccin, es decir,
a la diferencia entre las reas
sobre y bajo el eje de abscisas
en la figura 4.3.1.
El flujo enlazado es mximo
cuando la onda de induccin
est centrada en el eje
magntico de la bobina (figura
4.3.2) y es igual a N veces el
flujo por polo:
(4.3.11)

La tensin inducida, interpretada como tensin de rotacin, con 1 t = , se calcula


como
v=

d d
=
= m 1 sen( 1t + 1 )
dt
dt

(4.3.12)

y su valor efectivo vale


V=

1 m
2

2
2

f N p = 4,44 f N p

(4.3.13)

4-81

captulo 4 : devanados

expresin que por supuesto es plenamente coincidente con (2.4.9), derivada en forma
ms abstracta al final del captulo 2 y que podra haberse invocado directamente, ya
que para la ley de Faraday no tiene significacin la causa de la variacin de flujo.
Para algunos fines resulta conveniente contar con una forma alternativa a (4.3.12) que
explicite la induccin en el entrehierro.
En la figura 4.3.3 se aprecia que el desplazamiento de la onda de induccin en d
determina la variacin diferencial del flujo enlazado por la bobina
d = N( B1 l R
por lo que

d
d
B1' l R )
p
p
N l R
=
( B1 B1' )

(4.3.14)
(4.3.15)

b(x1,t)

B1
1

B1'<0

d
1'

x1
d

B1

Figura 4.3.3. Variacin del enlace de flujo para un


desplazamiento relativo diferencial.
d
= 1 , que reemplazada junto con (4.3.15) en (4.3.12) permite
dt
obtener la expresin alternativa
Por otra parte,

v (t ) = N l R

1
( B1 B1' ) .
p

(4.3.16)

Para una bobina de paso completo B1 = - B 1 , por lo que (4.3.16) se reduce en ese caso
a
v ( t ) = 2 N l u B1

(4.3.17)

donde u = 1R/p es la velocidad tangencial del campo respecto a la bobina.

4-82

captulo 4 : devanados

Como en (4.3.17) B1 = b(x1) es el valor de la induccin en el entrehierro correspondiente


a la coordenada de la bobina, se tiene que con velocidad constante la funcin v(t) es
una rplica de la funcin b(x). Esta propiedad se utiliza para obtener una imagen de la
distribucin espacial de la induccin en el entrehierro mediante una bobina exploratoria
(paso completo, N=1),conectada a un osciloscopio.

4.3.2 Tensin inducida en una bobina de paso acortado


En el caso de una bobina de paso acortado (b < p) el flujo enlazado por la bobina
tambin es mximo, aunque menor que p, cuando el mximo de la onda de induccin
coincide con el eje magntico de la bobina (figura 4.3.4) y vale:
+( )/ 2

m =

b( x , t ) l R d ( x
1

p ) = cos( 2) p

(4.3.18)

( )/ 2

donde
cos( 2) = fc
, es el factor de cuerda para la fundamental obtenido
anteriormente en el prrafo 4.2.1.
Consecuentemente el valor efectivo de la tensin inducida se reduce a
m <p

x1

1
2

1'

V = 4,44 f N fc p

(4.3.19)

Se aprecia que la tensin inducida en


una bobina de paso acortado de N
vueltas es equivalente a la tensin
inducida en una bobina de paso
completo con un nmero de vueltas
igual al nmero de vueltas efectivo
Nef=fc N < N.

Figura 4.3.4. Flujo mximo enlazado por


una bobina acortada.

4.3.3 Tensin inducida en un grupo de bobinas


Considrese ahora un grupo de q bobinas iguales, cada una de N/q vueltas, ubicadas
en ranuras separadas en forma regular en un ngulo = 2p/Z (figura 4.2.11) y
conectadas en serie.
La tensin inducida en el grupo es igual a la suma de las tensiones inducidas en cada
bobina.

4-83

captulo 4 : devanados

Las tensiones inducidas por el campo


giratorio en cada una de las q bobinas
tienen la misma amplitud y frecuencia,
pero estn desfasadas relativamente en
un ngulo igual al de su desplazamiento
espacial.

/2

Vq

Figura 4.3.5. Tensin resultante de un grupo


de q=3 bobinas.
V
= r sen(q / 2)
2

La tensin resultante corresponde


entonces a la suma fasorial de la
tensiones inducidas en las bobinas
individuales, segn est representado
en la figura 4.3.5, de la que se
desprende que

Vq
= r sen( / 2)
2

(4.3.20)

es decir, que
V = qVq

sen( q / 2)
= qVq f z ,
q sen( / 2)

(4.3.21)

donde fz es el factor de zona o de distribucin para la fundamental, definido


anteriormente en el prrafo 4.2.2. y que ahora puede reinterpretarse como el cuociente
fz =

suma geomtrica de las tensiones inducidas


suma aritmtica de las tensiones inducidas

(4.3.22)

Como en principio las q bobinas pueden ser de paso acortado, se tiene, al reemplazar
(4.3.19) en (4.3.21), que
V = 4,44 f N f d p = 4,44 f Nef p

(4.3.23)

con f d = f z fc y Nef = fd N .
Esto lleva a la conclusin que , para el efecto de determinar la tensin inducida, un
grupo de bobinas, eventualmente con acortamiento, puede ser reemplazado por una
sola bobina de paso completo, cuyo nmero de vueltas sea igual al nmero de vueltas
efectivo del grupo de bobinas que reemplaza.

4-84

captulo 4 : devanados

4.3.4 Tensin inducida en un devanado de corriente continua


Un devanado de corriente continua (figura 4.3.6) es un devanado distribuido de dos
capas, formado por bobinas iguales, conectadas todas en serie de manera de formar un
devanado cerrado, sin principio y sin fin. Las conexiones entre las bobinas se realizan
en segmentos de cobre (delgas) aislados elctricamente entre si.

cabeza bobina
delantera

Pieza polar
1

Conmutador

cabeza bobina trasera

largo activo
7

6
cabeza bobina delantera
Conmutador

Figura 4.3.6. Esquema de un devanado de corriente continua.

Delga

El conjunto de delgas forma una


estructura cilndrica, el conmutador o
colector (figura 4.3.7) sobre cuyo manto
rozan
escobillas
o
carbones,
desplazados relativamente en un paso
polar, que dividen el devanado en dos
circuitos en paralelo (figura 4.3.6). El
devanado de corriente continua siempre
est alojado en el rotor de la mquina.

Interesa determinar la tensin inducida


entre un par de escobillas cuando el
rotor gira con velocidad constante
respecto a una distribucin espacial de induccin fija al estator.
Figura 4.3.7. El conmutador.

4-85

captulo 4 : devanados

Contrariamente al caso de las mquinas de corriente alterna, en mquinas de corriente


continua no se busca una distribucin espacial de induccin sinusoidal y la distribucin
ideal corresponde a una distribucin alternada donde la induccin es constante en casi
todo el paso polar, excepto en una zona estrecha en los extremos de cada paso polar,
donde la induccin es cero. La distribucin prctica tiende a tener forma trapezoidal
(figura 4.3.8).

Bm
1

Bi

i
2

Figura 4.3.8. Distribucin de la componente radial de la


induccin en el entrehierro
Segn se aprecia en la figura 4.3.6, las escobillas dividen el devanado en dos circuitos
(ramas) en paralelo, cada uno formado por Nd /2 bobinas de N vueltas por bobina,
siendo Nd el nmero de delgas del conmutador.
La tensin inducida en la i-sima bobina de una de las ramas es segn (4.3.16)
vi = NlR

1
Bi B j
p

j=i+

con

Nd
2

(4.3.24)

y la tensin total inducida en las Nd /2 bobinas de una rama es


Nd /2

V=

(4.3.25)

i =1

Nd /2

Considerando que Bj = -Bi y que

i =1

Bi +N d / 2 =

Nd

i =1+ N d / 2

el reemplazo de (4.3.24) en la expresin (4.3.25) permite reescribirla como

V = Nl R 1
p

Nd

i =1

(4.3.26)

4-86

captulo 4 : devanados

Para un nmero suficientemente grande de delgas, el valor medio de la induccin radial


en el entrehierro vale (figura 4.3.8)
Bm =

(4.3.27)

Nd

Por otra parte, el valor medio de la induccin o densidad de flujo est dado por la
relacin
p
(4.3.28)
p l R l
p
Al reemplazar (4.3.27) y (4.3.28) en (4.3.26) se logra finalmente la frmula buscada
para la tensin entre el par de escobillas de un devanado de corriente continua de dos
polos (p=1) como
Bm =

V = z f p

(4.3.29)

donde z = 2 N Nd

(4.3.30)

es el nmero total de conductores del devanado del rotor (armadura , inducido) y

f = 1
2
es la frecuencia de giro del rotor.
4.4

Inductancias propias y mutuas de devanados

Para fines analticos es frecuente la


descripcin de la accin de devanados
en trminos de las inductancias propias y
mutuas asociadas a ellos. Interesa
entonces establecer relaciones entre
esos parmetros y las dimensiones
geomtricas de la mquina en la que
estn montados los devanados
Con este objetivo considrese el
dispositivo doblemente cilndrico de la
figura 4.4.1. El entrehierro sea
constante y los devanados distribuidos
del estator y del rotor tengan
respectivamente N1 y N2 vueltas y p

x1

N1

x2
N2

Figura 4.4.1. Relativo al acoplamiento


inductivo de dos
devanados.

4-87

captulo 4 : devanados

pares de polos. El radio del rotor sea R y su longitud axial l.


Supngase ahora que el devanado del rotor sea excitado por la corriente i2 .
Al limitar el anlisis a la fundamental de la distribucin espacial de fmm se tiene que la
amplitud de esta, de acuerdo con la relacin (4.2.26), vale
F2 =

4 i 2 N ef 2
2p

(4.4.1)

y que la amplitud de la onda de induccin correspondiente, de acuerdo con (4.2.36),


vale
B2 =

4 0 i 2 Nef 2
2p

(4.4.2)

El flujo por polo vale (figura 4.3.2)


p =

2
R l
B2

(4.4.3)

a partir del cual se calcula el enlace de flujo del devanado, correspondiente a flujo en el
entrehierro, como
2 = Nef 2 p =

4
Rl 2
0
Nef 2 i2

p2

(4.4.4 )

El factor de proporcionalidad entre el flujo enlazado por la bobina y la corriente que


produce ese flujo se conoce como inductancia. En consecuencia, la inductancia propia
del devanado del rotor vale
L2 =

2 4
Rl 2
= 0
Nef 2
i2

p2

(4.4.5)

La onda de induccin producida por el devanado del rotor en el entrehierro es una


cosinusoide centrada en el eje magntico del devanado del rotor, el que est
desplazado en el ngulo respecto al eje magntico del devanado del estator. En
relacin con un sistema de referencia x1, cuyo origen se encuentra en el eje magntico
del devanado del estator, la onda de induccin tiene la expresin
b2 ( x 1 ) = B2 cos( x1 )

(4.4.6)

El enlace de flujo del estator debido a la corriente del rotor se calcula como

4-88

captulo 4 : devanados

+/ 2

12 = Nef 1B 2

cos( x 1 )lRd(

/ 2

x1
)
p

(4.4.7)

4
Rl
12 = 0 2 Nef 1 Nef 2 cos i 2
p

(4.4.8)

Se aprecia que el flujo enlazado por el estator vara como funcin del ngulo y que la
inductancia mutua entre los dos devanados vale
L12 =

12 4
Rl
= 0 2 Nef 1 Nef 2 cos
i2
p

(4.4.9)

Los coeficientes de inductancia propia y de inductancia mutua contienen toda la


informacin geomtrica del aparato pertinente al campo en el entrehierro.
En general, el acoplamiento inductivo entre los dos devanados del dispositivo de la
figura 4.1.2 puede ser descrito simplemente en trminos de las variables de terminales
y de las inductancias propias y mutuas mediante las ecuaciones
d
d
L1i 1 ) + ( L12 i 2 )
(
dt
dt
d
d
v 2 = R2 i 2 + (L2 i 2 ) + (L21i 1 )
dt
dt
v 1 = R1i1 +

(4.4.10)

Al limitar el anlisis a la componente fundamental del campo en el entrehierro las


inductancias propias y mutuas corresponden a expresiones como (4.4.5) y (4.4.9), con
las que (4.4.10) toma la forma ms simple
di1
d
+ L12 ( i2 cos )
dt
dt
di
d
v 2 = R2 i 2 + L2 2 + L21 ( i1 cos )
dt
dt
v 1 = R1i1 + L1

(4.4.11)

donde L 1 , L2 y L12 = L 21 son constantes.


Sistemas de ecuaciones como (4.4.11) constituyen el punto de partida para el anlisis
transitorio de las mquinas elctricas.
Para ese objetivo la informacin de detalle sobre el devanado se ha hecho innecesaria
y basta conocer los valores numricos de las inductancias involucradas, que pueden ser
obtenidos experimentalmente para cada mquina especfica. El problema se ha
reducido a un problema de circuitos acoplados inductivamente.

5. Fuerzas electromagnticas

5.1

Introduccin

La idea del campo electromagntico fue desarrollada originalmente (Faraday, Maxwell)


como un medio conceptual, alternativo a la teora de la accin a distancia, para
establecer la relacin entre fuerzas y sus causas. As, por ejemplo, la fuerza entre dos
conductores por los que circulan sendas corrientes vino a ser considerada como la
fuerza ejercida por el campo magntico creado por la corriente en uno de los
conductores sobre la corriente (cargas en movimiento) en el otro conductor.
Son estas fuerzas las que evidencian la existencia de un campo y que permiten
asociarle energa.
Desde la introduccin de la idea de campo se ha hecho uso de conceptos figurativos,
como las lneas de fuerza o las propiedades elsticas de estas, para permitir la
interpretacin fsica de la accin del campo. As, la visualizacin de la distribucin
espacial del campo mediante una red de cuadrados curvilneos informa tambin sobre
la distribucin de las fuerzas superficiales y sobre su carcter electrodinmico (fuerzas
sobre corrientes) o magntico (fuerzas sobre superficies limtrofes entre medios de
diferente permeabilidad).
Cuando slo interesa determinar la fuerza resultante sobre cierto cuerpo, en
abstraccin de su distribucin espacial, resulta conveniente determinarla recurriendo al
principio de la conservacin de la energa y al mtodo de los trabajos virtuales.
La adopcin de esta metodologa tiene la ventaja de la generalidad, pues abarca tanto
a las fuerzas electrodinmicas como a las magnticas y porque tambin es aplicable al
campo elctrico.
El precio de la generalidad es la prdida de la interpretacin fsica del origen de las
fuerzas, lo que en algunos casos puede conducir a equvocos. Por ello es conveniente
el uso complementario del mtodo energtico y de los procedimientos de la teora de
campos.
Para
favorecer la creacin de imgenes personales, tan necesarias para la
comprensin de fenmenos complejos, los desarrollos de los prrafos siguientes se
limitan a sistemas simples con un solo grado de libertad mecnico. Esto no les quita
generalidad, ya que un movimiento tridimensional siempre puede pensarse generado a
partir de tres movimientos unidimensionales, para cada uno de los cuales vale la
conclusin, que slo habr una fuerza resultante en direccin de una coordenada
determinada si un movimiento virtual en la direccin de esa coordenada produce
variacin de la energa acumulada en el campo.

5-90

captulo 5 : fuerzas electromagnticas

5.2

Fuerza y energa, una visin sistmica

Para la determinacin de las fuerzas de origen electromagntico se supone que el


sistema electromecnico (transductor, mquina elctrica) est formado por un
subsistema elctrico y por un subsistema mecnico, acoplados mediante un campo
magntico conservativo.
Como se trata de sistemas en los cuales las frecuencias de la variables elctricas y las
velocidades mecnicas son relativamente bajas, su descripcin energtica slo
requiere de cuatro formas de energa:

Energa elctrica (Wel ), suministrada por las fuentes al subsistema elctrico.


Energa mecnica (W mec), asociada al subsistema mecnico.
Energa magntica (W mgn), asociada al campo magntico de acoplamiento.
Energa calrica (Wcal), asociada a los fenmenos disipativos en los subsistemas
elctrico y mecnico (prdidas en el cobre, prdidas en el fierro, prdidas por roce).

i1
v1
f,x
CAMPO
CONSERVATIVO

in
vn

Puertas elctricas

Wmgn

Puerta mecnica

Figura 5.2.1.Sistema electromecnico de n puertas


elctricas y una puerta mecnica

Estas cuatro formas de energa


estn relacionadas a travs del
principio de la conservacin de
la energa, conocido tambin
como primera ley de la
termodinmica, (formulado por
K.Mohr en 1837 como la
energa no se crea ni se
destruye, slo se transforma).
Considrese ahora el sistema
electromecnico conservativo
con n puertas elctricas y una
puerta o grado de libertad
mecnico (x) de la figura 5.2.1,
donde
para
las
puertas
elctricas rige la convencin
carga y para la puerta
mecnica la convencin fuente.

Pinsese un desplazamiento virtual positivo x del nodo mecnico, sobre el cual acta
la fuerza electromagntica fe, cuya referencia positiva coincide con la de x .
Durante el desplazamiento virtual el campo realiza sobre el terminal mecnico el
trabajo virtual Wmec =fex , se produce un aumento virtual de la energa acumulada en

5-91

captulo 5 : fuerzas electromagnticas

el campo magntico Wmgn y se absorbe la energa elctrica virtual Wel de las fuentes
que alimentan el subsistema elctrico.
En virtud del principio de conservacin de la energa, en un sistema conservativo (sin
prdidas) la energa que entra menos la energa que sale debe ser igual al incremento
de la energa en el sistema:
Wel W mec = W mgn

(5.2.1 )

Reemplazando en (5.2.1) la expresin para Wmec se obtiene


fe =

W el W mgn
.
x

(5.2.2)

Si se imagina el desplazamiento virtual realizado de manera tal que los enlaces de flujo
permanezcan constantes (i = 0), la energa elctrica absorbida no vara durante el
desplazamiento virtual
W el =

i =0

(5.2.3)

y (5.2.2) se reduce a
fe =

W mgn
x

(5.2.4)

i = cte

lo que se expresa matemticamente como


fe =

W mgn ( i ,x )
.
x

(5.2.5)

Alternativamente, si el desplazamiento virtual se imagina realizado manteniendo las


corrientes constantes ( ii = 0), la expresin (5.2.2) toma la forma
fe =

W el ( i i ,x ) W mgn ( i i ,x )

.
x
x

(5.2.6)

Mediante la relacin (transformacin de Legendre)


W mgn ( i i ,x )=

i W mgn ( i i ,x )

(5.2.7)

se define la funcin coenerga magntica W mgn y su uso reduce la relacin (5.2.6) a

5-92

captulo 5 : fuerzas electromagnticas

fe =

W mgn (i i ,x )
x

ya que

(5.2.8)

i W el (i i ,x )
=
.
x
x

(5.2.9)

En el caso de sistemas magnticamente lineales rige


i =

i j

i j

W mgn ( i i ,x )=

1
2

, por lo que

i j

(5.2.10)

(x ) i i i j

(5.2.11)

i, j

y, de acuerdo con (5.2.7), energa magntica y coenerga magntica son ahora


numricamente iguales. Sin embargo, debe respetarse escrupulosamente las variables
independientes usadas en (5.2.5) y (5.2.8) so pena de obtener un signo errneo para la
fuerza de origen electromagntico.

(i)

Wmgn

W mgn
0

i
I

Figura 5.2.2. Interpretacin de la energa y la


coenerga como reas en el plano -i.

Para sistemas con un grado de


libertad elctrico y un grado de
libertad mecnico la relacin
expresada por (5.2.7) tiene una
interpretacin geomtrica simple
en trminos de reas en el plano
(,i) , como puede apreciarse en
la figura 5.2.2, donde se observa
tambin que tanto el valor de la
energa magntica como el de la
coenerga
magntica
slo
dependen de los valores finales
del enlace de flujo () y de la
corriente
(I)
y
son
independientes de la forma en
que se alcanza esos valores
finales.

De (5.2.5) y (5.2.8) se desprende que slo se desarrollan fuerzas electromagnticas


si la energa (coenerga) asociada al campo vara con el desplazamiento.
Esta es una conclusin fundamental y perfectamente general.
Las ideas generales hasta aqu expuestas se comprenden mejor si se aplican a
situaciones concretas que permiten hacer asociaciones con la experiencia subjetiva.

5-93

captulo 5 : fuerzas electromagnticas

Para evitar dificultades matemticas innecesarias se supondr que los sistemas


electromecnicos poseen caractersticas magnticas (i) lineales y que los campos son
unidimensionales.

5.3

Transductores de movimiento limitado

La mayor parte de las mquinas y aparatos electromagnticos basan su accin en


campos confinados espacialmente mediante circuitos magnticos de material
ferromagntico de alta permeabilidad, interrumpidos por entrehierros relativamente
estrechos.
De la fsica experimental se sabe que sobre las superficies limtrofes entre materiales
de diferente permeabilidad (permitividad) se desarrollan fuerzas magnticas (elctricas)
que la teora de campos interpreta a travs de las propiedades elsticas que asocia a
las lneas de fuerza: las lneas de fuerza tienden a acortarse y a separarse.
La
fuerza magntica por unidad de rea se conoce como tensin de Maxwell y su
direccin, normal a la superficie limtrofe, es desde el medio de mayor permeabilidad al
de menor permeabilidad.
x

Seccin q

armadura
F

Considrese ahora el dispositivo


elemental de la figura 5.3.1. Su ncleo
est formado por chapas silicosas de
permeabilidad infinita en las que las
corrientes parsitas sean despreciables.
El campo en el entrehierro sea
homogneo.

Sobre la superficie de la armadura que


enfrenta el entrehierro el campo
magntico desarrolla fuerzas - se deben
i
a la tendencia de las lneas de fuerza a
Figura 5.3.1. Dispositivo elemental para
acortarse - que tienden a disminuir el
demostrar la fuerza de Maxwell. entrehierro.
La determinacin de la fuerza resultante sobre la armadura mediante la relacin (5.2.8)
requiere de la formulacin de una expresin para la coenerga asociada al campo
magntico en trminos de la corriente. Para ello se puede suponer que la energa
magntica slo se acumula en el volumen correspondiente al entrehierro V=qx, ya que
con un entrehierro finito el flujo y la induccin tambin sern finitos y la permeabilidad
infinita del ncleo implica que, es decir, que la densidad de energa magntica en el
ncleo es cero.
Como el campo en el entrehierro se supuso homogneo, la energa se expresa
simplemente como producto de la densidad de energa por el volumen:

5-94

captulo 5 : fuerzas electromagnticas

W mgn = 21 BH qx = 12 0 H 2 qx

(5.3.1)

La relacin entre H e i se obtiene aplicando la ley de Ampere a lo largo del camino de


integracin indicado en la figura 5.3.1, recordando que Hfe=0.
H=

iN
x

(5.3.2)

Reemplazando (5.3.2) en (5.3.1) se logra


W mgn ( i,x )= 12 0 N 2

q 2
i
x

(5.3.3)

Por otro lado, en trminos de la inductancia asociada a la bobina y la corriente rige la


expresin general
W mgn ( i,x )= 12 L( x ) i 2 ,

(5.3.4)

y por comparacin de coeficientes se tiene que en este caso la inductancia vale


L( x )= 0

q 2
N .
x

(5.3.5)

Como se trata de un sistema magnticamente lineal, la energa y la coenerga son


iguales y la expresin (5.2.8) toma la forma
fe =

W mgn (i ,x ) 1 2 dL
qN 2
= 2i
= 12 i 2 0 2
x
dx
x

(5.3.6)

o, en trminos de variables de campo,


f e = 12 0 H 2 q
(5.3.7)
Se aprecia que la fuerza es negativa en relacin con las referencias elegidas en la
figura 5.3.1, es decir, su sentido es del medio de mayor permeabilidad al de menor
permeabilidad. Su valor por unidad de superficie es
e = 12 0 H 2 =

B2
2 0

(5.3.8)

y corresponde a la tensin de Maxwell para campos homogneos.


La frmula (5.3.8) constituye el punto de partida para el dimensionamiento de
electroimanes.

5-95

captulo 5 : fuerzas electromagnticas

5.3.1 Torque de reluctancia


Te

Considrese ahora el dispositivo


ilustrado esquemticamente en la
figura 5.3.2, formado por un
ncleo fijo, provisto de una bobina
de N vueltas, y una armadura
rotatoria,
separados,
cuando
estn
alineados,
por
un
entrehierro cilndrico de ancho d.
El ncleo y la armadura sean
ideales.

Al desplazar la armadura desde la


posicin de simetra en un ngulo
, las lneas de fuerza del campo
Figura 5.3.2. Dispositivo elemental para demostrar se deforman con lo que aparece
el momento de reluctancia.
una distribucin de fuerzas sobre
las caras planas del rotor que da
lugar a un torque resultante, salvo para las posiciones =0, =90, =180 y =270,
para las cuales la distribucin de las lneas de fuerza es simtrica respecto al eje de
simetra principal del motor, por lo que el momento resultante desaparece. En el
intervalo 0<<90 el momento es negativo en relacin con la referencia positiva de la
figura 5.3.2, mientras que en el intervalo 90<<180 el momento es positivo. Se
aprecia que el momento siempre es tal que tiende a alinear el rotor con el eje de
simetra del estator.
i

La determinacin cuantitativa del torque resulta compleja, ya que requiere del


conocimiento de la distribucin espacial del campo magntico como funcin del ngulo
.
Frente a la imposibilidad de poder contar con una solucin analtica para el problema
de campo planteado se hace necesario bajar las exigencias y conformarse con una
solucin aproximada de validez limitada.
Considerando que para valores relativamente pequeos del ngulo la mayor parte de
la energa del campo se encuentra asociada al volumen del entrehierro limitado por las
caras cilndricas, se puede formular el siguiente modelo que a primera vista parece algo
burdo: toda la energa est en el volumen entre las superficies cilndricas.
Con este modelo y las denominaciones de la figura 5.3.2 se obtiene las siguientes
expresiones para la energa magntica en el entrehierro de ancho radial d y largo axial
l:
Wmgn = 2r ( + )dl 12 0H2

para -<<0

(5.3.9)

5-96

captulo 5 : fuerzas electromagnticas

Wmgn = 2r ( )dl 12 0H2

para 0<< .

(5.3.10)

Aplicando la ley de Ampere a lo largo del camino de integracin cerrado indicado en la


figura 5.3.2 se establece que
H=

iN
2d

(5.3.11)

por lo que (5.3.9) y (5.3.10) pueden ser reescritos como


Wmgn (i, ) = 21 0

rl
( + )N2 i2 = 12 L 1 ( ) i 2
2d

(5.3.12)

y
Wmgn (i, ) = 21 0

rl
( )N2 i2 = 12 L 2 ( ) i2
2d

(5.3.13)

obtenindose el momento a partir de la relacin (5.2.8), adaptada convenientemente al


desplazamiento giratorio, como
Te =

Wmgn (i, )

1
2

dL( ) 2
i
d

(5.3.14)

expresin que para los respectivos rangos de toma las formas explcitas
Te = 0

rl 2 2
N i
4d

para -<<0

(5.3.15)

para 0<<

(5.3.16)

y
Te = 0

rl 2 2
N i
4d

La dependencia del torque del cuadrado de la corriente implica que, an en sistemas


magnticamente lineales, la conversin electromecnica de energa es un fenmeno
nolineal.
La figura 5.3.3 ilustra la variacin de la inductancia propia y del torque (con i=cte) con el
desplazamiento angular del rotor (). Se puede observar que, coincidentemente con el
anlisis cualitativo, el torque es nulo para =0, =90, =180 y =270, es positivo para
-<<0 y es negativo para 0<<.
A primera vista este resultado puede parecer paradjico, ya que al despreciar el campo
en el aire fuera del entrehierro tambin desaparecen las fuerzas de traccin sobre las
caras planas del rotor, causantes del momento.

5-97

captulo 5 : fuerzas electromagnticas

Sin embargo, la contradiccin es slo aparente, ya que sobre las caras curvas tambin
actan fuerzas de presin tangenciales debidas a la tendencia de las lneas de fuerza a
separarse, aspecto que no se haba mencionado explcitamente en la discusin
anterior.
De manera que no hay contradiccin entre los resultados obtenidos a travs de la
aplicacin del criterio energtico y los esperables a partir de los conceptos de la teora
de campos.

T
L

()

(+)

Figura 5.3.3. Variacin de la inductancia y del torque para


el momento idealizado de la figura 5.3.2.

Los resultados cuantitativos de la figura 5.3.3 pueden ser mejorados, si se considera


que el valor mnimo de la inductancia propia no es cero sino un valor finito, estimado de
alguna manera. Las curvas segmentadas de la figura 5.3.3 ilustran el efecto de la
correccin.
De acuerdo con lo expuesto, el dispositivo slo desarrolla momento si el rotor es
anisotrpico, es decir, si no posee las mismas propiedades magnticas en todas las
direcciones radiales, pues solamente de esa manera se produce variacin de la
energa magntica con el desplazamiento angular.
Expresado en la terminologa de los circuitos magnticos, debe variar la reluctancia de
ste para que haya momento. Por esta razn el momento generado en esas
condiciones se conoce como momento de reluctancia.
Un rotor cilndrico no dara lugar a la formacin de momento.

5-98

captulo 5 : fuerzas electromagnticas

5.3.2 Torque de excitacin


Para que un dispositivo de
x1
simetra cilndrica, como el de la

Longitud axial : l
figura
5.3.4,
desarrolle
momento, el rotor debe estar
d
equipado con un devanado por
Te
i1
el cual circule corriente. Este
x2
N2
devanado, ubicado en ranuras,
N1
permite que haya variacin de la
i2
energa
magntica
o
deformacin de las lneas de
fuerza - con el desplazamiento
angular del rotor ().
Figura 5.3.4. Dispositivo elemental para demostrar el
momento de excitacin.
La determinacin del momento
desarrollado por el dispositivo de la figura 5.3.4 pasa por la determinacin de la energa
magntica asociada a l.
Para ello se recurre convenientemente a la distribucin de fmm resultante (figura 5.3.5),
obtenida a partir de la superposicin de las distribuciones de fmm rectangulares
correspondientes a las bobinas concentradas del estator y del rotor respectivamente.
De ella se puede apreciar que la intensidad del campo resultante y, con ella, la
densidad de energa magntica en el entrehierro, toma slo dos valores:
wms = 21 o (H1 + H2 )
y

(5.3.17)

w md = 21 o (H1 H2 ) .
2

(5.3.18)

La energa magntica total en el entrehierro se obtiene al multiplicar las densidades de


energa por los correspondientes volmenes y sumar luego las energas parciales as
obtenidas
Wmgn = 2 w ms ( )rdl + 2w md rdl

[
(
= rdl[H + 2( 2 )H H

Wmgn = 0 rdl ( ) H12 + 2H1H2 + H22 + H12 2H1H2 + H22


Wmgn

Wmgn = 12 0

2
1

+ H22

)]

rl 2 2
rl
rl 2 2
N1 i1 + 0
N1N2 ( 2 ) i1i2 + 12 0
N2 i2
2d
2d
2d

(5.3.19)

5-99

captulo 5 : fuerzas electromagnticas

F1+F2
F1

F2

/2

/2

Figura 5.3.5. Distribucin de la fmm a lo largo del


entrehierro del dispositivo de la figura 5.3.4.
Por otra parte, la expresin general para la energa asociada a un sistema de n bobinas
dada en (5.2.11), para el caso de 2 bobinas, se reduce a
W mgn = 21 L1 i12 + L12 i1i 2 + 21 L2 i22

(5.3.20)

y por comparacin de coeficientes con (5.3.19) se establece que las inductancias


propias y mutuas valen respectivamente:
L1 = 0

rl 2
N
2d 1

(5.3.21)

L2 = 0

rl 2
N
2d 2

(5.3.22)

L 12 = 0

( 2 )rl N N
1

2d

para

(5.3.23)

Se aprecia que slo la inductancia mutua L12 es funcin de la posicin angular del
rotor, por lo que la expresin para el momento se reduce a
Te =

Wmgn (i, )

dL12 ( )
i1 i2
d

(5.3.24)

y considerando a (5.3.23) toma la siguiente forma explcita:


Te =

rl
0 N1N2 i1i 2

para

(5.3.25)

5-100

captulo 5 : fuerzas electromagnticas

cuya representacin grfica como funcin de , para i1 e i2 constantes, muestra la figura


5.3.6.
Se puede observar que,
Te
L12
para un desplazamiento
entre los ejes magnticos
del estator y del rotor
dado,
el
momento

0
/2
/2
electromagntico
que

acta sobre el rotor tiene


un sentido tal que tiende
a
alinear
los
ejes
magnticos del estator y
Figura 5.3.6. Variacin de la inductancia y del torque para del rotor.
el modelo idealizado de la figura 5.3.4.
Alternativamente tambin
podra decirse que el momento nace de la tendencia de los campos del estator y del
rotor a alinearse.
Si se considera que induccin en el entrehierro producida por el devanado del estator
vale
B1 = 0

i1N1
2d

para

<x<
2
2

y
i N
B1 = 0 1 1
2d

(5.3.26)
para

2 < x

x <

la ecuacin (5.3.25) puede reescribirse como


Te = 2 r B1 l N2 i 2 = 2r f e
donde

f e = B1 l N2 i 2

(5.3.27)
(5.3.28)

tiene la estructura de la frmula de Lorentz para la fuerza sobre un conductor de


longitud l y corriente i2N2 que se encuentra en un campo magntico homogneo de
induccin B1 de orientacin normal al conductor.
Debido a la equivalencia entre las relaciones (5.3.27) y (5.3.24) es legtimo considerar
al momento, formalmente, como si se debiera a fuerzas electrodinmicas sobre los
conductores del rotor, aunque, en rigor, el momento se debe a fuerzas magnticas que
actan sobre las paredes de las ranuras.

5-101

captulo 5 : fuerzas electromagnticas

Sobre el conductor en la ranura prcticamente no actan fuerzas tangenciales, ya que


la alta permeabilidad del fierro hace que la mayor parte del flujo que cruza el
entrehierro siga por el fierro, por lo que el flujo en las ranuras corresponde
fundamentalmente al flujo de dispersin, causado por la propia corriente en la ranura.
Este flujo es paralelo al fondo de la ranura y determina fuerzas sobre el conductor
dirigidas hacia el fondo de la ranura, que por lo tanto no producen momento.

5.4

Mquinas rotatorias, conversin continua de energa

La mantencin de un proceso continuo de conversin de energa elctrica a energa


mecnica, o viceversa, requiere que el valor medio del trabajo mecnico realizado en
cada revolucin del rotor debe ser distinto de cero:
1 2
T d 0
2 0 e

(5.4.1)

Esta condicin no es satisfecha por los dispositivos giratorios analizados en el prrafo


anterior, si las corrientes en el devanado del estator y en el devanado del rotor son
corrientes continuas. Una mirada a los grficos de las figuras 5.3.3 y 5.3.6 permite
corroborar esta afirmacin. En ambos casos el momento medio para una revolucin es
cero.
Supngase ahora que la bobina del dispositivo de la figura 5.3.2 est alimentada por
pulsos de corriente cuya duracin corresponda a un cuarto de revolucin, como ilustra
la figura 5.4.1.
En esas condiciones el momento electromagntico sera positivo en los intervalos
2 < < 0 y 2 < < y nulo en los intervalos 0 < < 2 y < < 3 2 , por lo
que quedara satisfecha la condicin (5.4.1) y, gracias a la inercia, el movimiento del
rotor sera prcticamente continuo.
Te , i
i
Te

Figura 5.4.1. Relativo a la conversin de energa en


dispositivos de reluctancia.

La alimentacin de la
bobina con pulsos de
corriente
positivos
y
negativos
alternados
(figura 5.4.2) no altera la
forma del momento, ya que
ste, de acuerdo con
(5.3.14),
depende
del
cuadrado de la corriente de
excitacin.
Ntese que la frecuencia
de
la
componente

5-102

captulo 5 : fuerzas electromagnticas

fundamental de la onda de corriente de la figura 5.4.2 es igual a la frecuencia de giro


del rotor.
Esto lleva a la conclusin que el dispositivo alimentado con corriente alterna de
frecuencia angular 1 e impulsado a velocidad angular m=1 debe desarrollar un
momento medio.
Te , i

i
Te

Figura 5.4.2. Relativo a la conversin de energia


en dispositivos de reluctancia.
Para comprobar esto formalmente, supngase que el rotor gira con velocidad angular
constante m y que la inductancia, que es una funcin peridica del ngulo = mt, est
expresada mediante la serie de Fourier
L( ) = L 0 + L cos( ) .

(5.4.2)

Reemplazando esta expresin en (5.3.14), aquella se convierte en


Te = 21 L sen( ) i2 .

(5.4.3)

Supngase ahora que la corriente sea sinusoidal


i = 2I cos (1t + ) ,

(5.4.4)

con lo que i 2 = I 2 1+ cos(2 1t +2) y el valor medio del momento toma la forma
1 2
I2 2
T
d

cos(21t + 2) L sen( )d
2 0 e
4 0

2
21
I2
=
cos 2 cos
8
m
0

2
L v sen( )d sen 2 sen 1

m
0

L sen( )d

5-103

captulo 5 : fuerzas electromagnticas

Invocando las relaciones


2

cos mx sen nx dx = 0

(5.4.5)

y
2

0 sen mx sen nx dx =
2

se establece que

para m n
para m = n
21

T d 0 si
e

(5.4.6)

= ,

(5.4.7)

(5.4.8)
2 m
En el caso especfico del dispositivo elemental de la figura 5.3.2, cuya inductancia L()
est representada en la figura 5.3.3, la armnica dominante en el correspondiente
desarrollo en serie de Fourier es la segunda (=2), lo que implica que en ese caso se
produce conversin continua de energa si el rotor gira a velocidad sincrnica , es decir,
es decir, que hay momento medio si

si

m = 1

1 =

(5.4.9)

tal como lo haba previsto el anlisis cualitativo previo.


De acuerdo con la relacin (5.4.8) tambin habra otras velocidades a las cuales el
dispositivo desarrolla un momento medio distinto de cero, por ejemplo, para la cuarta
armnica (=4) resulta m = 1 2 . Sin embargo, el momento desarrollado a esa
velocidad es mucho ms dbil, por lo que carece de significacin prctica.
Considrese ahora la posibilidad de conversin continua de energa para el dispositivo
doblemente excitado de la figura 5.3.4.
Las condiciones necesarias se pueden visualizar elementalmente recurriendo a la idea
del campo giratorio.
Un devanado del estator de p pares de polos, excitado con una corriente alterna de
frecuencia angular 1 , produce un campo alterno cuya fundamental
b1( x1 ,t ) = B1 cos( px 1 ) cos( 1t + 1 ) ,

(5.4.10)

segn lo visto en el prrafo 4.3 del captulo sobre devanados, puede interpretarse como
resultante de la superposicin de dos campos giratorios que giran en sentidos opuestos
y cuya amplitud es igual a la mitad de la amplitud del campo alterno:
b1( x1 ,t ) =

B1
B
cos( px 1 1t 1 ) + 1 cos( px 1 + 1t + 1 )
2
2

(5.4.11)

5-104

captulo 5 : fuerzas electromagnticas

En forma anloga se obtiene para el campo producido por el devanado del rotor
b2 ( x 2 ,t )=

B2
B
cos( px 2 2 t 2 ) + 2 cos( px 2 + 2 t + 2 )
2
2

(5.4.12)

Los ejes magnticos de los devanados del estator (fijo) y del rotor (mvil), que
coinciden respectivamente con los orgenes de las coordenadas x1 y x2, estn
desplazados en el ngulo = m t (figura 5.3.4), por lo que rige:
x1 = x 2 + = x 2 + m t

(5.4.13)

Reemplazando (5.4.13) en (5.4.12) se obtiene


b2 ( x 1,t )=

B2
B
cos px1 ( 2 + p m ) t 2 + 2 cos px 1 + ( 2 p m ) t + 2 , (5.4.14)
2
2

que representa al campo del rotor referido al sistema de coordenadas del estator o, en
otras palabras, al campo del rotor visto por un observador ubicado en el estator.
Se aprecia que un campo giratorio del rotor gira a la misma velocidad que uno del
estator si :
1 = 2 + p m
o si
1 = 2 p m

(5.4.15)
(5.4.16)

Como el momento nace de la tendencia de los campos del estator y del rotor a
alinearse el uno con el otro, la velocidad relativa entre esos campos debe ser
necesariamente nula para que esta situacin se mantenga en el tiempo y pueda
desarrollarse un momento con valor medio distinto de cero. Si la velocidad relativa no
es cero, se desarrolla un momento oscilatorio cuyo valor medio es cero.
Las relaciones (5.4.14) y (5.4.15) son las restricciones que deben satisfacer las
frecuencias de las corrientes del estator y del rotor para que haya conversin continua
de energa en un dispositivo doblemente excitado.
Se aprecia que para que haya conversin continua de energa al menos uno de los
devanados tiene que estar excitado con una corriente alterna.
La comprobacin formal de estas conclusiones sigue la lnea del desarrollo hecho para
el momento de reluctancia, donde debe considerarse que ahora el momento depende
de la variacin de la inductancia mutua L 12 = L12 cos( ) .

captulo 5 : fuerzas electromagnticas

5.5

5-105

Resumen

En los prrafos precedentes se obtuvo criterios para la aparicin de fuerzas y


momentos
electromagnticos
a
partir
de
consideraciones
energticas,
yuxtaponindolos a los criterios que tienen su origen en la teora de campos.
Desde el punto de vista del balance energtico se concluye que slo se desarrollan
fuerzas o momentos electromagnticos si la energa asociada al campo vara como
funcin de la coordenada mecnica [(5.25) y (5.2.8)] y que el sentido de la fuerza es tal
que esta - con corriente constante - tiende a producir un desplazamiento que determina
un aumento de la coenerga del campo magntico [(5.2.8)].
El momento desaparece cuando la coenerga alcanza un valor extremo [(5.2.8)],
correspondiendo un mximo a una condicin de equilibrio estable y un mnimo a una
inestable.
De aqu se desprenden afirmaciones como: polos opuestos se atraen y polos iguales
se rechazan, un rotor anisotrpico tiende a alinearse con el campo o, los campos
producidos por las corrientes del estator y del rotor tienden a alinearse, que slo son
otras maneras de expresar la condicin de equilibrio en sistemas electromecnicos.
La comprobacin que la frmula de Lorentz tambin es aplicable a situaciones en que
los conductores estn alojados en ranuras, rodeados de fierro de alta permeabilidad,
permite inferir que esa frmula fundamental es equivalente a (5.2.8), lo que tiene
importantes consecuencias prcticas.
La relacin que deben satisfacer las frecuencias de las corrientes del estator y del rotor
y la frecuencia de giro del rotor ( p m = 1 2 ) , traducida a trminos del campo
giratorio, implica que para conversin continua de energa la velocidad relativa entre un
campo giratorio del estator y uno del rotor debe ser nula.
Esta condicin, si bien necesaria, no es suficiente. Para que exista momento medio
distinto de cero debe haber adems un desplazamiento espacial entre los ejes
magnticos de los campos giratorios del estator y del rotor, lo que implica un
desfasamiento apropiado de las corrientes del estator (1) y del rotor (2)
( 5.4.11)(, 5.4.14) .

En las diferentes mquinas elctricas que se analizan en los captulos siguientes:


mquina de corriente continua, mquina sincrnica y mquina asincrnica, estas
condiciones se cumplen de diferentes maneras, tratndose satisfacer en forma ptima
las exigencias especficas de cada aplicacin.

6. Mquina de corriente continua


6.1

Introduccin

La disponibilidad de una fuente de corriente continua, a travs de la pila de Volta,


determin que el desarrollo inicial de la electrotecnia girara al rededor de esa forma de
energa elctrica.
Consecuentemente, la primera mquina elctrica rotatoria, que aparece a mediados del
siglo XIX, es la mquina de corriente continua, la que a partir del descubrimiento del
principio dinamoelctrico por Siemens en 1866 experimenta un rpido desarrollo,
incorporando los detalles constructivos que la caracterizan hasta el presente.
La mquina de corriente continua, en su uso como motor, desplaz a las mquinas de
vapor de las fbricas y, en su uso como generador, posibilit los primeros sistemas de
distribucin elctrica, sentando as las bases para el desarrollo de la industria elctrica.
Con la introduccin de la corriente alterna, las mquinas de corriente continua perdieron
su posicin hegemnica inicial, mantenindose s en muchas aplicaciones en las que
sus caractersticas especficas las hacan irreemplazables.
El desarrollo ms reciente de los semiconductores de potencia ha tenido un efecto
doble sobre el empleo de la mquina de corriente continua.
Por una parte, la disponibilidad de rectificadores controlados, de costo muy inferior al de
una mquina de corriente continua, ha desplazado a esta de sus funciones tradicionales
como generador o como amplificador de potencia, mquinas que ya casi no se
construyen.
En cambio esos mismos rectificadores han ampliado considerablemente sus
posibilidades de uso como motor, al permitir la alimentacin de estos desde las redes
de corriente alterna.
En consecuencia, en la actualidad la principal aplicacin de la mquina de corriente
continua es como motor en accionamientos de velocidad variable. El rango de potencias
va desde una fraccin de kW hasta potencias del orden de 10.000kW, usadas en trenes
laminadores de la industria siderrgica y en motores que impulsan las hlices de
rompehielos y de submarinos.
En el futuro prximo el abaratamiento de nuevos semiconductores de potencia y de los
circuitos integrados a gran escala har posible la construccin de inversores que
permitirn darle a un motor asincrnico trifsico las caractersticas de un motor de
corriente continua y a un costo menor que el de ste y el correspondiente rectificador.

Captulo 6 : mquina de corriente continua.

6-107

Sin embargo, este desarrollo, al basarse en una estrategia de control que se inspira en
la mquina de corriente continua, tambin requiere de la comprensin del principio de
funcionamiento bsico de esta mquina, que, como idea, sigue tan vigente como
siempre.
En este captulo se pretende sentar las bases para la comprensin de la teora de la
mquina de corriente continua a partir de los conceptos generales desarrollados en los
captulos precedentes.
6.2

Caractersticas constructivas

En la figura 6.2.1 se muestra una vista frontal y una vista axial, ambas con cortes, de
una mquina de corriente continua convencional de cuatro polos (p=2). En ella se
identifican las partes ms importantes y se les asigna un nmero [ ].
El corte en la vista frontal permite apreciar la mitad del circuito magntico principal, que
est formado por las piezas polares[4](sobre las cuales estn montadas las bobinas
concentradas del devanado de campo), el yugo del estator, los entrehierros, los dientes
de la armadura (rotor) y el yugo de la armadura.
El yugo del estator cumple la doble funcin de elemento estructural y parte del circuito
magntico. En las mquinas convencionales es de acero fundido y en mquinas
modernas de construccin compacta est armado con chapas silicosas.
Las piezas polares, fijadas con pernos al yugo del estator, son de chapas magnticas,
para reducir las prdidas adicionales por corrientes parsitas, cuyo origen est en las
fluctuaciones locales de induccin producidas por los dientes y las ranuras del rotor en
movimiento.
El rotor[3], que gira en el campo magntico continuo creado por los polos principales,
est armado de chapas silicosas para reducir las prdidas por corrientes parsitas y por
histresis causadas por el giro. En las chapas se estampan las ranuras destinadas a
alojar el devanado de armadura (o inducido).
El devanado de armadura es un devanado de corriente continua cuya accin fue
discutida en el prrafo 4.3.4 del captulo sobre devanados. Cuando la mquina tiene
ms de dos polos, el devanado puede ejecutarse como imbricado (figura 6.2.2) u
ondulado (figura 6.2.3). Cada bobina est conectada [6] a dos segmentos (delgas,
lminas) [7] del conmutador (colector), formando as un devanado cerrado sin principio
y sin fin.
La alimentacin del devanado se efecta a travs de escobillas (carbones)[8], que
rozan sobre la superficie cilndrica del colector y que estn sujetas al estator mediante
portaescobillas y un collar [10].

Captulo 6 : mquina de corriente continua.

6-108

Figura 6.2.1 Vista frontal y axial de una mquina de corriente continua.


Las escobillas estn hechas de un aglomerado de grafito y metal en polvo y su
resistencia elctrica influye en forma importante sobre las caractersticas de la
conmutacin .
Una conmutacin correcta, sin chisporroteo, requiere de polos auxiliares (interpolos)[5],
ubicados simtricamente entre los polos principales, provistos de enrollados conectados
en serie con el devanado de armadura.

6-109

Captulo 6 : mquina de corriente continua.

1 2 3 4 5 6 7 8 9 10 11 12 13 14 15 16 17 18 19 20 21 22 23 24 25 26 27 28 29 30 31 32 33 34 35 36
1 2 3 4 5 6 7 8 9 10 11 12 13 14 15 16 17 18 19 20 21 22 23 24 25 26 27 28 29 30 31 32 33 34 35 36

33 34 35 36

1 2 3 4 5 6 7 8 9 10 11 12 13 14 15 16 17 18 19 20 21 22 23 24 25 26 27 28 29 30 31 32

Figura 6.2.1. Devanado imbricado.Nr=N d=36 , p=2 , a=2.

10 11 12 13 14 15 16 17 18 19 20 21 22 23 24 25 26 27

10 11 12 13 14 15 16 17 18 19 20 21 22 23 24 25 26 27

10 11 12 13 14 15 16 17 18 19 20 21 22 23 24 25 26 27

Figura 6.2.2. Devanado ondulado. N r =Nd=27 , p=2, a=1.


Fuera de las as llamadas partes activas, como el devanado y el circuito magntico, las
mquinas elctricas estn compuestas por partes pasivas (eje [1],tapas frontales,
descansos, ventilador[2], etc...), que no intervienen directamente en el proceso de
conversin de energa, pero cuyo adecuado diseo y ejecucin determina la calidad
mecnica de la mquina.

6.3

Principio de funcionamiento

La figura 6.3.1 muestra esquemticamente un corte transversal de la mquina de la


figura 6.2.1 en los que se destaca el circuito magntico principal y los devanados de
excitacin y de armadura.

6-110

Captulo 6 : mquina de corriente continua.

I/2
I

I/2
I/4

Figura 6.3.1. Dibujo esquemtico de una mquina de


corriente continua de 4 polos.
Si en la figura 6.2.2 se asume que las escobillas estn conectadas a una fuente de
corriente y se determina la distribucin de la corriente por las bobinas del devanado de
armadura para dos posiciones diferentes de la armadura, se comprueba que la
alimentacin del devanado de armadura a travs del mecanismo conmutador-escobillas
crea una distribucin espacial de corriente seudoestacionaria, caracterizada en la figura
6.3.1 mediante una secuencia alternada de puntos y cruces (corrientes entrando y
saliendo del plano del dibujo) que mantiene su posicin respecto a las piezas polares,
independientemente del giro de los conductores que forman el devanado de armadura.
La interpretacin formal de las fuerzas tangenciales sobre la superficie del rotor como
fuerzas electrodinmicas (captulo 5) permite apreciar que las fuerzas sobre las
corrientes bajo cada polo tienen el mismo sentido y que dan lugar a un momento
resultante. Con las referencias positivas para las corrientes de la figura 6.3.1 el sentido
positivo para el momento coincide con el sentido positivo para la velocidad angular, lo
que implica que la potencia mecnica que sale de la mquina es positiva.
Bajo la accin del momento electromagntico el rotor se pone en movimiento y se
inducen tensiones en las bobinas del devanado de armadura, que se suman en la forma
vista en el captulo 4, apareciendo una tensin resultante entre las escobillas cuya
polaridad, de acuerdo con la regla de Lenz, es tal que tiende a oponerse a la causa que
la produce. Como la causa de la tensin es el movimiento de la armadura y ste se
debe a la circulacin de corriente por esta, la polaridad de la tensin inducida es
opuesta a la de la tensin aplicada y tiende a disminuir la corriente absorbida por la
armadura.
El proceso de aceleracin termina cuando el momento resultante es nulo, vale decir,
cuando la tensin inducida es tal que la corriente de armadura es justo la necesaria
para desarrollar un momento electromagntico igual al momento externo aplicado al eje.

6-111

Captulo 6 : mquina de corriente continua.

Para invertir el sentido del flujo de energa y convertir la mquina de corriente continua
en un generador se debe reducir el momento aplicado al eje a cero, establecindose la
velocidad de vaco. Luego se invierte el momento y se lleva el rotor a una velocidad
superior a la de vaco. De esa manera la tensin inducida se hace mayor que la tensin
aplicada, con lo que se invierte el sentido de la corriente de armadura y con ella se
invierte el flujo de energa y cambia el sentido del momento electromagntico, que pasa
a ser frenante.
El razonamiento anterior pretende explicar la accin de la mquina de corriente
continua en trminos de sus variables externas o de terminales, como lo son la tensin
y la corriente y el torque y la velocidad. En trminos de esas variables elctricas
externas la mquina es una mquina de corriente continua.
Al derivar las condiciones generales para conversin continua de energa se haba
establecido que en al menos un devanado deba haber corriente alterna. Esta condicin
tambin se cumple en el caso de la mquina de corriente continua, donde, gracias a la
accin del conmutador, las corrientes en las bobinas del devanado de armadura son
corrientes alternas de forma de onda rectangular y frecuencia f = p n (con n velocidad
de giro en rps). Esto se hace aparente si se sigue el giro de un conductor a travs de
las zonas de puntos y cruces en el esquema de la figura 6.3.1.
De manera que la mquina de corriente continua, con 1 = 0 y 2 = p m , satisface la
condicin para la conversin continua de energa, 1 2 = p m , para cualquier
velocidad de giro. Esta caracterstica constituye la fortaleza del motor de corriente
continua.
6.4

Ecuaciones de equilibrio elctricas

Las mquinas de corriente continua poseen bsicamente dos circuitos: el circuito de


campo y el circuito de armadura, representados esquemticamente en la figura 6.4.1.
La aplicacin de la ley de Faraday a cada uno de estos circuitos da lugar a la
correspondiente ecuacin de equilibrio elctrica.
Con las denominaciones y sentidos de referencia de la figura 6.4.1 rige para el circuito
de armadura :
i a Ra v a =

,
dt t

(6.4.1)

donde el primer trmino del segundo miembro corresponde a la tensin inducida por la
rotacin de la armadura en el campo creado por la corriente de excitacin if, problema
ya analizado en el prrafo 4.3.4, y el segundo trmino corresponde a la tensin inducida
por la variacin temporal del flujo enlazado por el devanado de armadura, variacin que,

6-112

Captulo 6 : mquina de corriente continua.

If

Vf

Ia
Va
Figura 6.4.1. Referencias positivas para los circuitos de
armadura y de campo.
por la ortogonalidad de los ejes de los devanados de campo y de armadura, slo es
causada por la corriente de armadura.
En consecuencia (6.4.1) puede ser reescrita como
i a Ra v a = Vrot La

di a
dt

v a = i a Ra + La

dia
+ Vrot
dt

(6.4.2)

En el prrafo 4.3.4 se deriv la siguiente expresin para la tensin inducida entre las
escobillas de una armadura de corriente continua de un par de polos (p=1) y un par de
circuitos en paralelo (a=1):
Vrot = z n p

(6.4.3)

donde z es el nmero total de conductores de la armadura, n es la frecuencia de giro de


la armadura en revoluciones por segundo y p es el flujo por polo en Weber.

6-113

Captulo 6 : mquina de corriente continua.

Como ya se mencionara en el prrafo 6.2, para ms de dos polos existen bsicamente


dos posibilidades de ejecucin para el devanado de armadura, que puede ser imbricado
u ondulado.
En el devanado imbricado simple (figura 6.2.2) los extremos de una bobina se conectan
a dos delgas consecutivas, en las cuales se interconectan con las bobinas adyacentes,
formando un devanado cerrado que es dividido por las escobillas en tantos circuitos en
paralelo como polos haya (2a=2p).
En el devanado ondulado (figura 6.2.3) los
extremos de una bobina se conectan a dos
delgas separadas en aproximadamente un
Vrot
doble paso polar, en las cuales se
f
interconectan con otra bobina desplazada
respecto a la primera tambin en
aproximadamente un paso polar. De esa
manera se forman slo dos circuitos en
paralelo (2a=2) al ubicar un par de
escobillas desplazado relativamente en un
arctg d
paso polar. En la prctica se utiliza tantas
escobillas como polos, ubicando escobillas
Ff(I f)
adicionales en puntos equipotenciales con
0
las escobillas iniciales y unindolas a estas.
Figura 6.4.2.
De esa manera se logra un mejor
Caracterstica de magnetizacin p (Fp) y aprovechamiento de la superficie del
caracterstica de vaco V rot (I f).
conmutador, que redunda en conmutadores
ms cortos.

La generalizacin de (6.4.3) para estos dos tipos de devanados se desprende del


desarrollo realizado en el prrafo 4.3.4 si se considera que ahora cada rama en paralelo
est formada por Nd/2a bobinas y que el rea correspondiente a un paso polar es Rl/p.
Resulta
Vrot =

p
z n p
a

(6.4.4)

La relacin entre flujo y corriente de excitacin es nolineal y est determinada por la


caracterstica de magnetizacin del circuito magntico principal (figura 6.4.2). Slo en la
zona lineal la caracterstica p (if) se puede reemplazar por la funcin
p = d

Nf
if
p

(6.4.5)

donde d corresponde a la permeancia del circuito magntico principal o en el eje


directo y Nf corresponde al nmero de vueltas en serie del devanado de campo.

6-114

Captulo 6 : mquina de corriente continua.

Reemplazando (6.4.5) en (6.4.4) se obtiene


Vrot =

N
p

z m d f if
a
2
p

(6.4.6)

relacin que puede reescribirse convenientemente como


2
N N i
(6.4.7)
q f d m f
z
con Nq =
, el nmero de vueltas de cada rama en paralelo,
4a
2
y
= f z , el factor de zona correspondiente a un ancho de zona de 180 elctricos.

Vrot =

Definiendo la inductancia rotacional como


G fq = f z Nq Nf d

(6.4.8)

se logra finalmente una expresin para la tensin


inducida por rotacin
en trminos de un
parmetro concentrado constante que tiene la
estructura de una inductancia mutua:
Vrot = Gfq m i f .

(6.4.9)

di a
+ G fq m i f
dt

Ra

La

va

En consecuencia, para circuitos magnticos


lineales la expresin (6.4.2) toma la forma:
v a = R a i a + La

ia

vrot

Figura 6.4.3 Circuito equivalente


(6.4.10)

ecuacin que es satisfecha por el circuito equivalente de la figura 6.4.3, donde la


tensin rotacional est representada por una fuente de tensin controlada.
Por otra parte, la aplicacin de la ley de Faraday al circuito de campo de la figura 6.4.1
permite anotar:
d f
i f Rf v f =
,
(6.4.11)
dt
donde, en circuitos magnticos lineales, f = Lf i f , por lo que en ese caso
v f = R f i f + Lf

di f
dt

(6.4.12)

6-115

Captulo 6 : mquina de corriente continua.

En estado estacionario las corrientes de armadura y de campo son constantes, por lo


que para esa condicin las ecuaciones (6.4.10) y (6.4.12) se reducen respectivamente
a
Va = Ra Ia + Vrot

(6.4.13)

Vf = R f I f .

(6.4.14)

6.5

Ecuacin de equilibrio mecnica

Las ecuaciones de equilibrio elctricas, obtenidas a partir de la ley de Faraday, son


equivalentes a la segunda ley de Kirchhoff (LVK), que exige que en una malla la suma
de las tensiones es igual a cero.
Anlogamente, la ecuacin de equilibrio mecnica, obtenida a partir de la aplicacin de
la segunda ley de Newton al rotor:
J

d m
=
dt

(6.5.1)

exige que la suma de los momentos sobre el rotor (considerado como un cuerpo rgido)
sea igual al momento de inercia J por la aceleracin angular.
Sobre el rotor actan dos momentos : el momento mecnico externo aplicado al eje Tm
y el momento electromagntico Te .
El momento electromagntico
que acta sobre el rotor puede considerarse
formalmente como debido a fuerzas electrodinmicas sobre los conductores de la
armadura donde, de acuerdo con el desarrollo del prrafo 5.3, cada ranura aporta con
Te i = R Bi l i i

(6.5.2)

al momento resultante
Nr
Ni N r
Te = Te i = R l a Bi
a i =1
i =1

(6.5.3)

Ni
Te = R l a Nr Bm
a

(6.5.4)

donde N es el nmero de vueltas de cada bobina del devanado de armadura, Nr el


nmero de ranuras del rotor, a el nmero de pares de circuitos en paralelo de la
armadura, ia la corriente de armadura, Bi la induccin en el entrehierro sobre la i-sima
ranura y Bm la induccin media en el entrehierro.

6-116

Captulo 6 : mquina de corriente continua.

Si en la relacin (6.5.4) se reemplaza la induccin media en trminos del flujo por polo
mediante
p
(6.5.5)
Rl
p
y se considera que el nmero de conductores de la armadura es z=2NNr,
Bm =

Te =

queda

1 p
v i
zia p = rot a
2 a
m

(6.5.6)

expresin que explicita el balance de potencia, o sea, que la potencia electromagntica


convertida v rot ia es igual a la potencia mecnica Te m .
Para el caso de un circuito magntico lineal se puede reemplazar vrot en trminos de
(6.4.9), con lo que se obtiene
Te = Gfq if ia

(6.5.7)

En estado estacionario la aceleracin es nula y el momento en el eje se obtiene


directamente de (6.5.6) como
Tm =

Vrot I a
m

(6.5.8)

relacin que se usar ms adelante al analizar las caractersticas de funcionamiento


estacionarias de la mquina de corriente continua.

6.6

Funcionamiento estacionario

Para la determinacin de la tensin de rotacin o del momento electromagntico no fue


necesario el conocimiento detallado de la distribucin de la induccin a lo largo del
entrehierro, bastando el conocimiento del flujo por polo. Sin embargo, hay una serie de
aspectos asociados al correcto funcionamiento de la mquina de corriente continua real,
como la conmutacin, las prdidas de fierro adicionales, la tensin entre delgas o la
estabilidad estacionaria, que requieren del conocimiento de la distribucin espacial del
campo en el entrehierro.

6-117

Captulo 6 : mquina de corriente continua.

6.6.1 Distribucin del campo en el entrehierro


En los prrafos siguientes se determinar la distribucin espacial idealizada para la
induccin en el entrehierro a partir de principios bsicos. Las idealizaciones se refieren
a asumir la permeabilidad del fierro como infinita, la permeancia del entrehierro bajo los
polos como constante y la permeancia del espacio interpolar como nula.
En la parte superior de la figura 6.6.1
se muestra esquemticamente un
corte
transversal
desarrollado,
correspondiente a un doble paso
polar, y en la parte inferior est
representada la distribucin de fmm
y de induccin en el entrehierro
producida en vaco por la corriente
en el devanado de campo, el que se
ha supuesto muy delgado.

2p

Ff

p
B ,Ff

En la figura 6.6.2 se muestra la


distribucin
de
fmm
(lnea

2
0
segmentada) y de induccin (lnea
llena) para el caso en que slo
circula corriente por el circuito de
Figura 6.6.1 Distribucin de fuerza
armadura, que adems del devanado
e induccin en vaco
magnetomotriz
de armadura incluye los interpolos.
(Ia=0).
Como la permeancia en los espacios
interpolares es supuestamente nula, la induccin en esas regiones tambin lo es,
independientemente del valor de la fmm. La intensidad de la fmm de los interpolos se
elige siempre algo mayor que
la amplitud de la onda
triangular producida por el
devanado de armadura, para
satisfacer
las
exigencias
planteadas
por
una
B Fa
conmutacin correcta.
x

En la figura 6.6.3 se ilustra la


superposicin de las dos
situaciones anteriores. Se
aprecia
que
como
consecuencia de la reaccin
de armadura la distribucin
de induccin bajo los polos
deja de ser constante,
aumentando bajo una mitad
del polo en relacin con el

Induccin debido a la armadura e interpolos.


Fmm debido a la armadura e interpolos.

Figura 6.6.2. Distribucin de fuerza magnetomotriz


e induccin debida a la corriente de
armadura solamente.

6-118

Captulo 6 : mquina de corriente continua.

valor de vaco y disminuyendo respecto a ese valor bajo la otra mitad.


2p

Motor

Como por razones de seguridad


(peligro de arco elctrico en el
conmutador) la tensin mxima
entre delgas no debe exceder a
valores del orden de 30V, la
distribucin dispareja de la
induccin
lleva
a
un
subaprovechamiento
de
la
mquina y a una disminucin de
su capacidad de sobrecarga.
Adems, la distribucin dispareja
de la induccin implica un
aumento en las prdidas de fierro
en los dientes y en el yugo de la
armadura en relacin con el valor
que tienen en vaco.

Induccin debida al campo


Induccin debida a la armadura
Induccin resultante.

Para contrarrestar estos efectos


negativos se puede recurrir a un
devanado de compensacin,
Figura 6.6.3 Distribucin de induccin resultante,
alojado en ranuras practicadas
distorsionada por la reaccin de
en las zapatas polares
y
armadura
conectado elctricamente en
serie con la armadura. En la
figura 6.6.4 se aprecia que la fmm del devanado de compensacin anula a la fmm
debida a la reaccin de armadura a lo largo de la zapata polar, restituyendo as la
distribucin de induccin a la forma que tiene en vaco.
Los devanados de compensacin son caros, por lo que su uso se limita a mquinas de
gran potencia, a mquinas con fuertes sobrecargas momentneas o a mquinas cuya
velocidad se regula mediante el debilitamiento del campo.

6.6.2 Efecto desmagnetizante de la reaccin de armadura


En ausencia de saturacin la fmm de reaccin de armadura distorsiona la distribucin
espacial de induccin (figura 6.6.3) sin alterar el flujo neto por polo, que es proporcional
al rea bajo la curva de induccin B(x). El aumento de flujo bajo una mitad del polo es
igual a la disminucin de flujo bajo la otra mitad.
Considrese ahora que el fierro se satura. Invocando la ley de Ampere, se puede
determinar la caracterstica de magnetizacin equivalente B(F) para el camino de

6-119

Captulo 6 : mquina de corriente continua.

integracin que pasa por los ejes de simetra de los polos, indicado en la figura 6.6.5.
Esta caracterstica tambin vale para un camino de integracin que cruza el entrehierro
a una distancia x del eje de simetra del polo, ya que a lo largo de los tramos de longitud
x en el fierro del polo y de la armadura el camino de integracin es perpendicular a las
lneas de fuerza, por lo que los correspondientes aportes a la integral de Ampere son
nulos.
2p

p
B
Fa

x
Fc

Fmm debido a la armadura e interpolos.


Fmm debido a devanado de compensacin
Induccin resultante.

Figura 6.6.4. Distribucin de induccin resultante con


devanado de compensacin.
Para determinar la distribucin B(x) a lo largo de la zapata polar basta entonces
determinar el valor de la fmm disponible para cada punto x (igual a la corriente
abrazada por el camino de integracin) y entrar con estos valores a la caracterstica de
magnetizacin equivalente B(F). El procedimiento est ilustrado en la parte inferior de
la figura 6.6.5 para los puntos extremos de la zapata polar.
Concretamente, en los extremos del arco polar, es decir, para x =

p
, la fmm total
2

abrazada por el camino de integracin vale respectivamente


F = Ff F
con
z
I

F =
a
2p 2a 2

(6.6.1)
(6.6.2)

6-120

Captulo 6 : mquina de corriente continua.

2p

Ff

B
Bd

Bo
Bi

0
p

Ff

Figura 6.6.5. Determinacin del campo resultante en presencia de


saturacin. Efecto desmagnetizante de la reaccin de
armadura.

En la figura 6.6.5 se aprecia que en presencia de saturacin el aumento de la induccin


en el extremo derecho de la zapata Bd, donde la fmm tiene el valor Ff + F , es menor
que la disminucin de la induccin en el extremo izquierdo Bi , donde la fmm tiene el
valor Ff F , lo que implica que el aumento del flujo bajo el semipolo derecho es
menor que la disminucin del flujo bajo el semipolo izquierdo . En consecuencia, el flujo
con carga, proporcional al rea bajo la curva B (x), disminuye en relacin con el flujo en
vaco, proporcional al rea del rectngulo de base p y altura B0, en
1
= ((B0 Bi ) (Bd B0 ))p l , si se aplica la regla de Simpson.
6

En mquinas saturadas la reaccin de armadura tiene un efecto desmagnetizante !

6-121

Captulo 6 : mquina de corriente continua.

interpolos

+
shunt

serie

armadura

compensacin

Figura 6.6.6. Esquema de coneccin de una


mquina de corriente contnua.

Para evitar esta disminucin


indeseada del flujo, que en
motores conectados a redes de
tensin
constante
puede
provocar un comportamiento
inestable, se usa
(en
aquellas mquinas en las que
todava no se justifica un
devanado de compensacin) un
devanado adicional de pocas
espiras montado sobre los polos
principales
y
conectado
elctricamente en serie con la
armadura de manera que su
fmm refuerce la del campo
principal.

La figura 6.6.6 resume en forma esquemtica los diferentes devanados de una mquina
de corriente continua y su interconexin elctrica.
6.6.3 Autoexcitacin
Una de las caractersticas sobresalientes de los generadores de corriente continua es
su capacidad de procurarse su propia corriente de excitacin a travs del as llamado
principio dinamoelctrico descubierto por Siemens en 1866.
Al discutir el lazo de histresis se vio que, despus de anulada la excitacin (H=0), la
induccin no bajaba a cero, sino slo a su valor de remanencia Br . Esta induccin de
remanencia es normal en mquinas elctricas que han sido magnetizadas alguna vez y
es una condicin bsica para la autoexcitacin.
Considrese ahora una mquina de corriente
continua cuyo devanado de campo haya sido
conectado en paralelo con la armadura en la forma
ilustrada en la figura 6.6.7 y que es impulsada a
velocidad constante.

f
-

if

Va Vrot

Rf , L f

Figura 6.6.7.
La conexin fuerza que la tensin de armadura, que
Relativo a la autoexcitacin del
en vaco es aproximadamente igual a la tensin
generador de corriente continua.
rotacional, sea igual a la tensin de campo
v rot = Rf i f + Lf

di f
dt

(6.6.3)

6-122

Captulo 6 : mquina de corriente continua.

En la figura 6.6.8 estn representadas las


V
caractersticas vrot e ifRf como funcin de la
corriente de campo if . Se aprecia que la
diferencia entre las ordenadas para un Vrot0
determinado valor de la corriente de campo
corresponde a la tensin de autoinduccin
en el devanado de campo
Lf

di f
= v rot Rf if
dt

if Rcrit

ifR f

Vrot
Lf

di f
dt

(6.6.4)

y mientras esta, proporcional a la rapidez


de crecimiento de la corriente de campo,
sea positiva la corriente de campo crecer.

Rf if

Vr
0

if
if

If0

Figura 6.6.8.
Por lo tanto la tensin de armadura
Relativo a la autoexitacin del
aumentar hasta que se establezca un
generador
de corriente continua.
valor estable para la corriente de campo, lo
que ocurre para el punto de interseccin de las dos caractersticas, para el cual la
tensin de autoinduccin desaparece, con lo que termina la fase transitoria del proceso
de autoexcitacin.
De lo anterior se desprende que para que haya autoexcitacin deben cumplirse las
siguientes condiciones:
1.2.3.4.-

Debe haber flujo remanente suficiente.


La conexin del devanado de campo debe ser tal que la corriente de campo
refuerce el flujo remanente.
El circuito magntico debe exhibir saturacin.
La resistencia del circuito de campo debe ser menor que cierto valor crtico,
determinado por la pendiente inicial de la caracterstica de saturacin.

6.6.4 Conmutacin
El hecho que, por un lado, la distribucin espacial de la corriente sea seudo estacionaria
y que, por otro lado, los conductores de la armadura giren con el rotor, implica que la
corriente a travs de los conductores invierte su sentido cada vez que estos se
desplazan en un paso polar.
De la observacin de los devanados de corriente continua de las figuras 6.2.2 y 6.2.3 se
desprende que las escobillas cortocircuitan transitoriamente al menos dos bobinas por
cada par de polos.

6-123

Captulo 6 : mquina de corriente continua.

Ia/2
Ia /2

Ia /2

Ia/2

2 1

Ia/2

Ia/2

2 1

2 1

Ia

Ia

Ia

a)

b)

c)

Figura 6.6.9. Las tres fases de la conmutacin de la corriente


en una bobina de la armadura.
Durante el lapso en que una bobina permanece cortocircuitada se produce la inversin
de la corriente que circula por ella. La figura 6.6.9 ilustra las tres fases de este proceso
conocido como conmutacin.
Hasta antes del cortocircuito la bobina est incluida en el grupo de bobinas que forman
la rama derecha de las dos ramas en paralelo creadas por la escobilla y por ella
circula la corriente Ia /2 . Esta situacin corresponde a la figura 6.6.9 a).
Cuando la escobilla establece el cortocircuito de la bobina se inicia la segunda fase,
durante la cual la bobina est excluida de ambas ramas. Durante esta fase la corriente
debera variar idealmente desde el valor inicial Ia /2 al valor final -Ia /2. Esta situacin
corresponde a la figura 6.6.9 b).
i

Ia/2
0

Ia/2
Tc
T=1/pn

Al abrirse el cortocircuito de la bobina


despus del tiempo de conmutacin Tc ,
la bobina queda incorporada a la rama
izquierda de las dos ramas en paralelo
creadas por la escobilla. Se ha
completado la conmutacin. Esta
situacin corresponde a la figura 6.6.9 c).
La figura 6.6.10 ilustra un perodo de la
forma de onda de la corriente en la
bobina bajo el supuesto de una
conmutacin ideal.

Figura 6.6.10. Forma de onda de la corriente La conmutacin ideal, o lineal, no se da


en forma natural, como se puede
en una bobina de armadura.
apreciar en el caso extremo de una

6-124

Captulo 6 : mquina de corriente continua.

bobina sin resistencia.


En este caso, en la malla formada por la bobina cortocircuitada por la escobilla debe
cumplirse

d b
di
= Lb
= 0
dt
dt

R=0

+Ia/2

(6.6.5)

donde Lb es la inductancia asociada a la


bobina cortocircuitada. Esto implica que
0
Tc t
la corriente permanece constante en el
valor inicial i=Ia /2 hasta el instante Tc en
que la escobilla abandona la delga 2
-Ia/2
(figura 6.6.11), forzando la interrupcin
Figura 6.6.11. La funcin i(t) en la bobina
brusca de la corriente que sale de la
delga 2 (figura 6.6.9), que salta de i2=Ia
conmutante para conmutacin
lineal y en ausencia de resistencia. a i2=0. La correspondiente tensin de
autoinduccin es tan elevada, que
ioniza el aire y establece un arco elctrico entre el borde de fuga de la escobilla y la
delga abandonada por esta.

Lm()

mT c

Figura 6.6.12. Variacin de la inductancia mutua


durante la conmutacin.
Para evitar este fenmeno indeseado es necesario modificar las consecuencias de la
restriccin (6.6.5), para lo cual se completa la mquina con polos auxiliares (interpolos o
polos de conmutacin) ubicados en la zona neutra magntica entre los polos principales

6-125

Captulo 6 : mquina de corriente continua.

(figura 6.2.1), provistos de bobinas de pocas vueltas por las cuales circula la corriente
de armadura Ia . De esta manera la restriccin enlace de flujo constante, impuesta por
el cortocircuito de la bobina que conmuta, ya no implica corriente constante, sino
b( 0 + ) = b(t)

para (0< t < Tc ) ,

(6.6.6)

es decir,
Lp

Ia
+ Lm( 0) I a = Lp i + Lm() I a
2

(6.6.7)

donde Lp es la inductancia propia (constante) de la bobina que conmuta y Lm es la


inductancia mutua (variable) entre esa bobina y los interpolos.
La figura 6.6.12 ilustra la variacin de la inductancia mutua durante la conmutacin,
cuando la bobina conmutante avanza de la posicin a) de la figura 6.6.9 a la posicin c).
Se puede apreciar que en primera aproximacin

2t
Lm( ) = Lm(0) 1

Tc

(0< t < Tc )

(6.6.8)

(0< t< Tc ).

(6.6.9)

por lo que se logra de (6.6.7)


i =

Ia
2Lm ( 0) 2t
1

2
Lp
Tc

Si el nmero de vueltas de los interpolos fuese tal que 2Lm(0)=Lp , la conmutacin sera
lineal y la corriente i alcanzara justamente el valor -Ia /2 cuando t=Tc (figura 6.6.11).
Un anlisis ms riguroso del problema de la conmutacin, que incluya el efecto de las
resistencias de bobinas y escobillas, conduce a un sistema de ecuaciones diferenciales
y escapa del objetivo de este prrafo.
6.6.5 Caractersticas estacionarias como generador
El anlisis del comportamiento estacionario de los generadores de corriente continua
requiere del uso de procedimientos grficos, ya que, debido a la saturacin del circuito
magntico principal, la relacin entre las variables de terminales es nolineal.
En este contexto interesan especialmente la caracterstica interior, que representa a
Vrot(If) con la corriente de armadura Ia como parmetro, y la caracterstica exterior, que
representa a Va(Ia) con la corriente de campo If como parmetro.

6-126

Captulo 6 : mquina de corriente continua.

Vrot
Vrot 0

I a=0

Vrot I

Ia=I nom
I aRa
n = cte.

I f

Va

La caracterstica interior se
puede
obtener
en
forma
experimental,
impulsando
la
mquina
con
velocidad
constante y variando la corriente
de campo entre cero y un valor
algo superior al nominal. Este
ltimo corresponde a la corriente
de campo para la cual la tensin
inducida en la armadura es igual
a la tensin nominal.

If

En vaco, es decir, para Ia=0, la


caracterstica interior coincide
la
caracterstica
de
Figura 6.6.13. Caracterstica interior del generador con
magnetizacin.
En
cambio
con
de corriente continua.
carga se hace sentir el efecto
desmagnetizante de la reaccin de armadura, lo que implica que para igual corriente de
excitacin que en vaco la tensin inducida ser menor en la zona saturada de la
caracterstica.
0

If0

If

La tensin inducida Vrot slo es medible en vaco. Con carga debe calcularse a partir de
la relacin Vrot=V a+IaRa y de los valores medibles Va e Ia .
En la figura 6.6.13 est representada la caracterstica de vaco y la caracterstica
interior para corriente nominal. Se aprecia que para generar la misma tensin Vrot1 que
en vaco, o sea, para mantener el mismo flujo por polo, la corriente de campo tiene que
ser incrementada en If, que representa el efecto desmagnetizante de la reaccin de
armadura, expresada en trminos de la corriente de campo.
Las caractersticas interna y externa del generador estn relacionadas. La figura 6.6.14
muestra esta relacin para el caso de un generador autoexcitado en derivacin, tambin
conocido como generador shunt.
En vaco la corriente de excitacin es If0 y la tensin inducida es Vrot=V a=R f If .
Supngase ahora que por la armadura circule la corriente Ia1 . La caracterstica Va(If )
para esa corriente se logra desplazando la caracterstica interna correspondiente a Ia1
paralelamente hacia abajo en una distancia igual a la cada hmica en la armadura IaRa.
Debido a la conexin shunt, la tensin aplicada al campo debe ser igual a la tensin de
armadura, Vf = V a, condicin que se cumple para la interseccin de la recta del campo
con la caracterstica Va(If) y determina Va1 . En consecuencia, la corriente de campo
disminuye de If0 a If1 . Con Va1 e Ia1 queda determinado un punto en la caracterstica
externa.

6-127

Captulo 6 : mquina de corriente continua.

Vrot

n=cte

Va

Va

Ia =0

0
Va1

Ia1

Va2
Ia1R a

Va3

Va1

Ia2 R a

1
Va2
a2

If2

If1

If0

If

Icoci

Ia1

Ia2

Ia

Figura 6.6.14. Caracterstica interior y exterior para un generador shunt.

Al relacionar las caractersticas externa e interna, se aprecia que la variacin de la


tensin de armadura para una determinada corriente de armadura Ia1 se debe a la
disminucin del flujo debida a la disminucin de la corriente de campo de If0 a If1 (Va1),
a la disminucin del flujo debida al efecto desmagnetizante de la reaccin de armadura
(Va2) y a la cada de tensin en la resistencia de armadura (Va3).
De la relacin entre caracterstica externa e interna se desprende adems que el
generador shunt slo funciona establemente hasta una cierta corriente mxima. Cuando
la corriente de armadura alcanza ese valor, la caracterstica Va(If ) correspondiente se
hace tangente a la recta del campo. Para corrientes mayores el equilibrio estacionario
no es posible, ya no se produce la necesaria interseccin con la recta del campo, por lo
que rige
v a = v f = R f i f + Lf

di f
dt

(6.6.10)

Como ahora v a < Rf i f , el trmino con la derivada es negativo, por lo que la


corriente de campo disminuye y con ella la tensin rotacional.
La corriente de armadura se reduce (lnea segmentada en la caracterstica externa de la
figura 6.6.14) hasta la corriente de cortocircuito, mantenida por la tensin de
remanencia. Se aprecia que la prdida de la capacidad de autoexcitarse del generador
shunt lo proteje en caso de un cortocircuito en sus terminales.

6-128

Captulo 6 : mquina de corriente continua.

6.6.6 Caractersticas estacionarias como motor


La caracterstica torque-velocidad de un motor de corriente continua depende en
primera instancia de la conexin del devanado de campo, donde debe distinguirse entre
excitacin independiente, excitacin shunt, excitacin serie y excitacin compound.
En el caso de la excitacin independiente el devanado de campo est alimentado por
una fuente independiente de la que alimenta a la armadura. Es la conexin tpica para
motores alimentados mediante rectificadores controlados. Haciendo abstraccin del
efecto desmagnetizante de la reaccin de armadura, el flujo es esencialmente
constante.
La excitacin shunt implica que el campo y la armadura estn conectados en paralelo a
la misma fuente. La tensin nominal del campo es por lo tanto igual a la tensin nominal
de la armadura. Si la tensin de armadura es constante, el motor shunt se comporta en
forma similar al motor con excitacin independiente.
En el motor con excitacin serie el devanado de campo est conectado en serie con la
armadura, por lo que el flujo vara fuertemente con la carga, lo que determina la
caracterstica momento-velocidad tpica para este motor, que lo haca particularmente
apto para aplicaciones de traccin elctrica.(locomotoras, tranvas). Actualmente el
principal uso del motor serie es como motor universal en el accionamiento de aparatos
electrodomsticos.
El motor compound es un hbrido, pues est provisto de un devanado shunt y de un
devanado serie. El devanado serie, normalmente de pocas vueltas, se usa para
compensar el efecto desmagnetizante de la reaccin de armadura, pero tambin para
darle al motor una caracterstica torque-velocidad intermedia entre la del motor shunt y
la del motor serie.
En lo que sigue, el anlisis se limitar al motor de excitacin independiente como
representante de la categora flujo constante y al motor serie, para ilustrar la obtencin
de la caracterstica cuando el flujo es variable.
De acuerdo con (6.4.13) , para el circuito de armadura de un motor con excitacin
independiente que funciona en estado estacionario alimentado desde una fuente de
tensin V debe cumplirse que
V = Vrot + Ra I a

(6.6.11)

Si en esta ecuacin se reemplaza a Vrot por la expresin (6.4.4) y se expresa Ia en


trminos del torque electromagntico Te mediante la relacin (6.5.6) se logra la
siguiente relacin entre frecuencia de giro n y el torque Te :

6-129

Captulo 6 : mquina de corriente continua.

V
Ra
n = p
Te
1 p 2
z p
z
a
2 a

(6.6.12)

2
p

Como la tensin de armadura y el flujo (si se hace abstraccin del efecto


desmagnetizante de la reaccin de armadura) son constantes, la ecuacin (6.6.12)
representa a una recta con pendiente negativa en el plano n-T e de la figura 6.6.15.
Cuando el torque es cero, el motor desarrolla la velocidad de vaco
n0 =

V
p
z p
a

(6.6.13)

ajustable mediante la tensin de armadura o el flujo. La velocidad de vaco con tensin


nominal y flujo nominal se denomina la velocidad natural de la mquina.
Una caracterstica distintiva del motor de excitacin independiente es la posibilidad de
ajustar su velocidad en un amplio rango.
n

+
n03

n02
n01

p
Ra

Te

0
Figura 6.6.15. Caracterstica n(Te) para un motor shunt.
Para aumentar la velocidad por sobre la natural se recurre al debilitamiento del campo
a travs de la reduccin de la corriente de excitacin. La velocidad mxima permisible
est limitada por los valores mximos admisibles para la fuerza centrfuga y para la
tensin entre delgas.

6-130

Captulo 6 : mquina de corriente continua.

Para disminuir la velocidad por debajo de la natural se recurre a la disminucin de la


tensin de armadura, por ejemplo, a travs del aumento del ngulo de disparo del
rectificador controlado.
Con flujo dado, la pendiente de la caracterstica slo depende del valor de la resistencia
de armadura Ra . Como esta es pequea, la velocidad del motor con excitacin
independiente vara poco con la carga.
Contrariamente al caso del motor con excitacin independiente, el motor serie funciona
con flujo variable, por lo que es necesario incluir el efecto de la saturacin mediante la
caracterstica de magnetizacin (obtenida como generador con excitacin
independiente). Para destacar lo esencial, aqu tambin se ignorar el efecto
desmagnetizante de la reaccin de armadura.
Para determinar la caracterstica n(T e) se procede punto por punto, asumiendo
sucesivos valores para la corriente de armadura Ia.
Dado que Ia =If , con el valor asumido para Ia se entra a la caracterstica de
magnetizacin (figura 6.6.16 a) ) y se determina (Vrot/n), valor que permite calcular el
momento mediante la relacin (6.5.8) como
Te =

1
2

Vrot

I
n a

(6.6.14)

Por otro lado se calcula


Vrot = V Ra Ia

(6.6.15)

para determinar con el valor para (Vrot/n) obtenido anteriormente


n =

Vrot
,
Vrot

(6.6.16)

y obtener as un punto de la caracterstica n(T e) de la figura 6.6.16 b).


Al observar la caracterstica n(T e) del motor serie se puede apreciar que la velocidad
vara fuertemente con la carga, lo que es caracterstico para este motor y fue
determinante para su aplicacin en traccin elctrica. Tambin se aprecia que la
velocidad de vaco tiende a crecer sin lmite, por lo que debe tomarse medidas para que
este motor nunca pueda funcionar en vaco.

6-131

Captulo 6 : mquina de corriente continua.

Vrot
n

Ra
I a=I f

0
a)

Te

0
b)

Figura 6.6.16. Caracterctica p(I a) y n(Te) para un motor serie.

7. Mquina sincrnica
(de rotor cilndrico)

7.1

Introduccin

A partir de la introduccin de la generacin y transmisin trifsica de energa, con


motivo de la Feria Internacional de Francfort de 1881, la electrotecnia experiment una
rpida expansin. En el breve plazo de una dcada surgi todo lo que hasta el presente
es parte substancial de un sistema elctrico de potencia.
El desarrollo posterior se caracteriz por la concentracin del proceso de conversin de
energa - realizado originalmente en pequeas centrales de propiedad municipal - en
centrales cada vez mayores. Calderas , turbinas y generadores grandes tienen mejor
rendimiento que las versiones ms pequeas.
Actualmente estn en servicio turbogeneradores en centrales nucleares con potencias
del orden de los 2000MVA e hidrogeneradores con potencias que alcanzan a 860MVA
(Itaip), cifras de las que se desprende que la mquina sincrnica es la mquina
elctrica de mayor tamao que se construye.
Como generador, la mquina sincrnica es la fuente de energa de los sistemas
elctricos de potencia. Sus caractersticas electromagnticas y electromecnicas son
determinantes en el comportamiento, tanto estacionario como dinmico, de estos
sistemas.
Como motor, la mquina sincrnica, gracias a su mejor rendimiento y a la capacidad de
suministrar potencia reactiva inductiva a la red, es la mquina motriz preferida para los
accionamientos de gran potencia.
En este captulo se desarrolla la teora del funcionamiento estacionario simtrico de la
mquina sincrnica sobre la base de los principios e ideas desarrollados en los
primeros cinco captulos, privilegiando el uso de la idea del campo giratorio y
enfatizando la relacin entre los desplazamientos espaciales entre ondas de induccin y
el desfasamiento temporal entre las tensiones inducidas por estas, para lo cual se hace
uso amplio de los diagramas fasoriales.
Los parmetros usados en los circuitos equivalentes se obtienen a partir de la
integracin de las variables de campo y se expresan en trminos de las dimensiones
geomtricas de la mquina, permitiendo as relacionar sus valores numricos con el
tamao de esta.
La visualizacin de los procesos a travs del uso de imgenes adecuadas permite una
mejor comprensin del trasfondo fsico y sienta las bases para el posterior estudio del
comportamiento de la mquina sincrnica en rgimen transitorio.

Captulo 7 : mquina sincrnica.

7.2

7-133

Caractersticas constructivas

La figura 7.2.1 muestra una mquina sincrnica en corte. El estator est armado de
segmentos de chapa silicosa en los que estn estampadas ranuras en las que se aloja
el devanado trifsico, conectado en estrella sin neutro. En mquinas de baja velocidad,
caracterizadas por un nmero de los polos elevado, el devanado se ejecuta con un
nmero de ranuras por polo y por fase (q) fraccionario, para mejorar la forma de onda
de la tensin inducida.

Figura 7.2.1 Vista en perspectiva, con corte longitudinal de


una mquina sincrnica de rotor cilndrico.

Para el rotor existen dos formas constructivas. En mquinas de baja velocidad se usa
la forma polos salientes, provistas de devanados concentrados (figura 7.2.2), mientras
que en mquinas de alta velocidad (turbogeneradores) se recurre a la forma rotor
cilndrico, donde el devanado est alojado en ranuras fresadas en el cuerpo cilndrico
de acero forjado del rotor (figura 7.2.3). Las bobinas de los polos del rotor
habitualmente estn conectadas en serie. La alimentacin del devanado de campo
realiza a travs de anillos rozantes, aunque en mquinas modernas de gran potencia es
cada vez ms comn el uso de la excitacin sin escobillas, que obtiene la corriente
continua a partir de una excitatriz alterna y un puente de diodos que gira con el rotor.

Captulo 7 : mquina sincrnica.

7-134

p=2

Figura 7.2.2. Corte transversal a travs


de un rotor de polos salientes.

p=1

Figura 7.2.3. Corte transversal a travs


de un rotor cilndrico.

En mquinas mayores (> 40 MVA) el medio refrigerante gaseoso es hidrgeno, por su


mayor calor especfico (4 veces el del aire) y su menor densidad ( de la del aire). Esta
ltima caracterstica disminuye notablemente las prdidas de roce. Para evitar el
peligro de explosin por mezcla con el oxgeno atmosfrico, el hidrgeno debe estar
sobrepresin. La carcaza debe ser hermtica y tiene que resistir presiones internas de
10 bar sin deformarse.
El rotor de turbogeneradores de dos polos puede alcanzar un dimetro de 1,2m (lmite
impuesto por los esfuerzos centrfugos) y 6 a 8 m de longitud (lmite impuesto por la
tranquilidad de marcha, vibraciones admisibles). Para 50 Hz la velocidad tangencial
alcanza 190 m/s, lo que implica fuerzas centrfugas muy elevadas sobre las cabezas de
las bobinas, que deben ser protegidas mediante sendos anillos de acero no magntico.
Las ranuras del rotor se cierran con cuas metlicas (normalmente de bronce) que por
regla general forman la jaula de amortiguacin, cuya funcin ser discutida ms
adelante.
El entrehierro de mquinas sincrnicas es relativamente grande, si se le compara con el
de mquinas asincrnicas o el de mquinas de corriente continua, y puede alcanzar a
varios centmetros.
7.3

Principio de funcionamiento

Supngase que la mquina es impulsada a velocidad nominal y que por el devanado


del rotor circule corriente continua. El devanado trifsico del estator est abierto.
La corriente continua determina una distribucin espacial de fmm fija respecto al rotor,
que, debido al movimiento de ste, gira respecto al estator con velocidad sincrnica. La
fundamental de la onda de induccin correspondiente induce en las fases del estator un
sistema de tensiones simtrico de frecuencia

Captulo 7 : mquina sincrnica.

f1 = p n

7-135

(7.3.1)

donde p es el nmero de pares de polos de la mquina y n es la frecuencia mecnica


de giro en revoluciones por segundo. El momento aplicado al eje en esas condiciones
es cero, si se hace abstraccin de las prdidas ( de roce y ventilacin y en el fierro del
estator).
Considrese ahora que se conecta una carga simtrica a las fases del estator. Las
tensiones inducidas determinarn un sistema de corrientes simtrico, desfasado
respecto al de las tensiones en un ngulo determinado por las impedancias de carga.
Estas corrientes dan lugar a un segundo campo giratorio en el entrehierro de la
mquina, que gira a la misma velocidad que el rotor, pero est desplazado respecto al
eje magntico de ste en un ngulo que depende del desfasamiento de la corriente de
armadura respecto a la tensin de terminales. De la tendencia a alinearse de estos dos
campos nace el momento electromagntico frenante, el que debe ser igualado por el
momento de la mquina motriz para que la velocidad permanezca constante. En
estado estacionario la potencia mecnica suministrada por la mquina motriz es igual a
la potencia elctrica absorbida por la carga ms las prdidas asociadas al proceso de
conversin de energa.
7.4

Ecuaciones de equilibrio elctricas

La figura 7.4.1 muestra un corte transversal esquemtico de una mquina sincrnica de


rotor cilndrico. El devanado de campo de Nf vueltas en serie, ubicado en el rotor,
ocupa 2/3 de la periferia y da lugar a una onda de fmm trapezoidal y, con entrehierro
constante, a una onda de induccin trapezoidal de valor mximo.
Bf =

0 Nf i f
" 2p

(7.4.1)

Del desarrollo en serie de Fourier de la distribucin trapezoidal slo se considera la


fundamental
bf ( x2 ) = B$ f cos( px 2 )

(7.4.2)

de amplitud
$ = 4 f B ,
B
(7.4.3)
f
df f
ya que el efecto de la tercera armnica sobre la tensin es contrarrestado por la
conexin estrella sin neutro del devanado del estator y el efecto de las armnicas
superiores es atenuado mediante un acortamiento y distribucin apropiados.

Captulo 7 : mquina sincrnica.

7-136

eje fase a
eje d

x1
b'

c
p=1

eje q

a'

x2

c'

Figura 7.4.1. Corte esquemtico a travs de la


mquina sincrnica de rotor cilndrico.

La coordenada fija al estator x1 , cuyo origen coincide con el eje magntico de la fase a,
y la coordenada fija al rotor x2, cuyo origen coincide con el eje de simetra (eje d) del
rotor, estn relacionadas por (figura 7.4.1).
x1 =

+ x2
p

(7.4.4)

donde en estado estacionario


= 1t 0

(7.4.5)

es el ngulo entre los ejes magnticos de la fase a y del rotor.


En trminos de la coordenada x1 la distribucin espacial de induccin (7.4.2) toma la
forma
$ cos( px t + )
bf ( x1, t ) = B
f
1
1
0

(7.4.6)

y el flujo enlazado por el devanado concentrado equivalente de la fase a, se calcula


como (figura 7.4.1).

Captulo 7 : mquina sincrnica.

7-137

/2p

b (x , t)lRdx

af =

(7.4.7)

/2p

af =

2lR )
Bf cos(1 t 0 )
p

(7.4.8)

La tensin inducida en la fase a por el flujo de la rueda polar vale


v p = N1 f d 1

d af
dt

(7.4.9)

Al introducir el flujo por polo del rotor p =

v p = p N1 fd 1 1 j e j( 1t 0 )

vp =

2 Vp e j 1t

2Rl $
Bf y la notacin compleja, queda
p

(7.4.10)
(7.4.11)

con
Vp = j 1

p
2

N1 f d 1e

j 0

(7.4.12)

fasor de la tensin inducida en la fase de referencia a por el flujo debido a la corriente


de campo. Considerando (7.4.1) y (7.4.3), la ecuacin (7.4.12) puede reescribirse como
Vp = j1 L1f

I f j 0
e
2

(7.4.13)

donde
L1f =

p N1fd 1 4 0 R l
=
(N f )(N1fd 1 )
If
" p 2 f df

(7.4.14)

es la inductancia mutua entre el devanado de campo y la fase a del estator cuando


ambos devanados estn alineados. L1f slo es constante en ausencia de saturacin, la
que en (7.4.14) se manifiesta a travs del entrehierro equivalente .
Por su lado, las corrientes en las tres fases del estator

Captulo 7 : mquina sincrnica.

ia =

7-138

2I 1 cos( 1t + 1 )

2
)
3
2
i c = 2I1 cos( 1t + 1 +
)
3
ib =

2I 1 cos( 1t + 1

(7.4.15)

crean una distribucin espacial de fmm, cuya fundamental


f1 =

3 4 N1 fd 1
2 2p

2I 1 cos( px 1 1t 1 )

(7.4.16)

determina la distribucin giratoria de induccin


b1 ( x 1 ,t ) =

3 4 0 N1 f d 1
2 " 2p

2I1 cos( px 1 1 t 1 ) .

(7.4.17)

Si bien la superposicin de fmms siempre es admisible, no vale lo mismo para las


inducciones, que slo pueden ser superpuestas si el circuito magntico es lineal.
Suponiendo que el principio de superposicin sea aplicable, el flujo creado por las
corrientes del estator y enlazado por el devanado concentrado equivalente de la fase a
se calcula como
/2p

b (x , t) l R dx

a1 =

(7.4.18)

/ 2p

a1 =

3 4 o lR
N1 fd 1 2I 1 cos( 1t + 1 ) .
2 p 2

(7.4.19)

La tensin inducida por esta componente del flujo en la fase es


v a = N1 f d 1

d a 1

(7.4.20)

dt

34
lR N1 f d 1
va =
o

2
p

} {

2 I1 1 sen( 1t + 1 )

(7.4.21)

(7.4.22)

v a = 2 j1Lm 1I1e j 1t = 2Va e j 1t


donde

Captulo 7 : mquina sincrnica.

Lm 1

3 4
lR N1 f d1
=
0

2
p

7-139

(7.4.23)

es la inductancia propia de campo giratorio del devanado del estator.


Adems del flujo fundamental en el entrehierro, las corrientes en los devanados del
estator producen campos armnicos, flujo de dispersin de ranuras y flujo de dispersin
frontal, los que inducen en las fases del estator tensiones de frecuencia angular 1 . El
efecto inductivo de estos flujos se expresa mediante la inductancia de dispersin del
devanado del estator L1. De manera que
v =

d
d
=
L 2I 1 cos(1t + 1 )
dt
dt 1

v = 1 L 1 2I 1 sen( 1 t + 1 ) =

(7.4.24)

2 j 1 L 1 I 1 e j 1 t .

(7.4.25)

Al aplicar la Ley de Faraday a la fase de referencia a del estator se tiene que


r
r
d
E
ds = dt
se convierte en
v 1 i1R1 =

d
= v p + va + v ,
dt

(7.4.26)

de donde se logra, al reemplazar respectivamente las expresiones fasoriales (7.4.22) y


(7.4.25) para va y v,
V1 = R1I1 + jX m 1I1 + jX 1I1 + Vp .

(7.4.27)

En estado estacionario los campos giratorios se mueven sincrnicamente con el rotor y


por lo tanto no inducen tensiones en el devanado de campo. Para este devanado rige
en consecuencia
Vf = R f I f

(7.4.28)

Captulo 7 : mquina sincrnica.

7-140

7.4.1 Circuito equivalente por fase


Del examen de la ecuacin (7.4.27) se desprende que esta es satisfecha por el circuito
equivalente de la 7.4.2. En este circuito las tensiones Vp , Va, y V representan los
flujos p , a y 1 de la mquina fsica.
resultante en el entrehierro.
I1

R1

representa el flujo

Xm1

X1

V1

La tensin Vi

Vi

Vp

Figura 7.4.2. Circuito equivalente de mquina sincrnica.


El circuito equivalente de la figura 7.4.3 se logra formalmente a partir del de la figura
7.4.2 aplicando el teorema de transformacin de fuentes. Las corrientes I1 , If e I m
representan las fmms de la mquina original.
I1

R1

I f'

X1
Im

V1

Vi

Xm1

Vp
jXm1

Figura 7.4.3. Circuito equivalente de mquina


sincrnica.
Concretamente,
If =

Vp
jX m 1

L1f I f
ej 0
Lm 1 2

(7.4.29)

Captulo 7 : mquina sincrnica.

7-141

es el valor efectivo de una corriente alterna ficticia, que, circulando en el devanado


trifsico del estator, produce el mismo efecto magntico que la corriente continua If al
circular por el devanado de campo. De (7.4.29) se desprende que mediante la relacin
L1f

I f =

2Lm 1

If =

1
I
g f

(7.4.30)

con
g =

3 N1f d 1
2 Nf fdf

(7.4.31)

como factor de reaccin de armadura, es posible expresar una corriente de armadura


mediante una corriente de campo equivalente y viceversa. De esta manera se puede
trabajar con corrientes en el lugar de fmms, lo que probar ser muy prctico al
considerar el efecto de la saturacin.

7.4.2 Efecto de la saturacin


En mquinas modernas, altamente aprovechadas, la induccin alcanza valores
elevados, lo que implica la saturacin del fierro, especialmente en las regiones
correspondientes a los dientes y yugo del estator.
d

La
figura
7.4.4
ilustra
esquemticamente el circuito
magntico del flujo principal en
vaco. Haciendo abstraccin del
flujo de dispersin del devanado
de campo, se trata de un circuito
magntico serie, donde el flujo
es el mismo en el entrehierro, en
el yugo del estator y en el yugo
del rotor. En consecuencia, el
valor del flujo en el entrehierro
fija la induccin en cada tramo
del circuito magntico y por ende
fija el valor de la fmm resultante.

Como en vaco el flujo en el


entrehierro es proporcional a la
tensin inducida Vp (7.4.12) y la
fmm resultante es proporcional a la corriente de campo If , la caracterstica de vaco
Vp(If) es proporcional a la caracterstica de magnetizacin p(Ff) de la mquina
sincrnica.
Figura 7.4.4. Circuito magntico principal en vaco.

Captulo 7 : mquina sincrnica.

7-142

La fmm correspondiente a un valor de flujo determinado se puede pensar


descompuesta en dos sumandos, correspondientes respectivamente a los tramos en
aire y en fierro del camino de integracin a lo largo del cual se aplica la Ley de Ampere
(figura 7.4.5).
'

Vp
p

''

r
r
H ds = Ff

2H + Hfe l fe = Ff

Ffe

(7.4.32)

F + Ffe = Ff
La prolongacin de la parte recta inicial
de la caracterstica de vaco se conoce
como caracterstica del entrehierro.
Segn ese modelo, Ffe es una medida
del grado de saturacin del circuito
magntico. En ausencia de saturacin
toda la fmm se gasta en el entrehierro.
If

Podra sospecharse que la saturacin y


la consiguiente nolinealidad de la
0
Ff
Ff
funcin p (If)
complica el anlisis,
obligando a recurrir a mtodos
Figura 7.4.5. Caracterstica de magnetizacin. grficos, como en el caso de las
mquinas de corriente continua.
Sin embargo, esto por regla general no es as, porque la mquina sincrnica funciona
normalmente conectada a redes de tensin y frecuencia constantes, lo que determina
un flujo y por lo tanto un grado de saturacin que tambin son constantes. En este
caso, se puede considerar que la fmm resultante se gasta en un entrehierro ficticio y
que el fierro es ideal (fe ), lo que equivale al reemplazo de la caracterstica de
magnetizacin por una caracterstica lineal (figura 7.4.5), que por supuesto slo vale
para esa condicin de saturacin.
7.4.3 Diagrama fasorial
Las tensiones y corrientes asociadas a los circuitos equivalentes de las figuras 7.4.2 y
7.4.3 se representan convenientemente en forma de un diagrama fasorial. Para
destacar lo esencial se supondr que la resistencia del estator es despreciable, o sea,
R1 = 0, lo que siempre ser admisible en mquinas mayores.
Supngase conocidas la tensin en los terminales V1, la corriente de armadura I1 , el
factor de potencia cos 1, el factor de reaccin de armadura g, la reactancia de
dispersin X1 y la caracterstica de vaco.

Captulo 7 : mquina sincrnica.

7-143

Vp
Va

Vp'

Vi
R1=0

V1

Im'
1
I f'
I1
Figura 7.4.6. Diagrama fasorial en presencia de saturacin.
La construccin del diagrama fasorial de la figura 7.4.6 comienza con la ubicacin de I1
en relacin a V1 . Restando de V1 la cada en la reactancia de dispersin V = jX 1I1
se logra la tensin Vi , que es proporcional al flujo resultante en el entrehierro y que por
lo tanto fija el grado de saturacin.
Con el mdulo de Vi se entra a la caracterstica de vaco y se fija la pendiente de la
caracterstica del entrehierro equivalente (figura 7.4.7).
En seguida, se calcula I 1 = g I1, una corriente de campo ficticia que produce el mismo
efecto magntico que la corriente armadura I1, y se determina sobre la caracterstica del
entrehierro equivalente la tensin inducida por el campo de reaccin de armadura en el
devanado de armadura Va. Recordando que Va est adelantado en /2 respecto a I1 ,
se resta Va de Vi para obtener Vp .
Con el mdulo de Vp se entra a la caracterstica del entrehierro equivalente y se
determina la corriente de campo If .
Finalmente se calcula I f = If /g y se la dibuja atrasada en /2 respecto a Vp. La suma de
If e I1 determina la corriente magnetizante I m.

Captulo 7 : mquina sincrnica.

7-144

Vp
Vp

Vp'
Vi

Va

If
I1' Im'
If
Figura 7.4.7. Construccin del diagrama fasorial en presencia de
saturacin.
0

Al construir e interpretar el diagrama fasorial debe tenerse presente que slo V1, I1 e If
corresponden a magnitudes medibles y que las dems tensiones y corrientes son
magnitudes ficticias. As, al desconectar la mquina de la red, la tensin en terminales
no es Vp sino Vp, ya que con la desconexin cambia el grado de saturacin y con l el
entrehierro ficticio que determina la pendiente de la caracterstica de magnetizacin
equivalente.
Durante la construccin del diagrama fasorial se tiene la libertad de elegir un fasor como
fasor de referencia. Normalmente elige V1 = V 1 /0 , con lo que
Vp = jVp e j ( / 2 0 ) = Vp e j

(7.4.33)

est desfasada en el ngulo de carga


=

0
2

(7.4.34)

respecto a V1. El ngulo de carga corresponde al desplazamiento del fasor Vp respecto


a V1 cuando la carga de la mquina vara entre vaco (T = 0, = 0) y un valor
cualquiera.

Captulo 7 : mquina sincrnica.

7-145

La interpretacin del diagrama fasorial se enriquece si se considera que el ngulo de


desfasamiento entre dos tensiones es igual al ngulo de desplazamiento espacial (en
radianes elctricos) entre las correspondientes distribuciones de induccin y que el
ngulo de desfasamiento entre dos corrientes es igual al ngulo de desplazamiento
espacial entre las correspondientes distribuciones de fmm.
Eje q

Re

Vp

Eje d

V1
- 0

Im

t=0
V1=V 1 0
-0= - /2

Figura 7.4.8. Relacin entre flujos y tensiones.


Desde esta perspectiva, la eleccin V1 = V1 /0 , que implica que v a = 2V1 cos( 1t ) ,
considera como t = 0 al instante en que la tensin inducida en la fase de referencia a
es mxima. Pero como la tensin es mxima cuando el flujo abrazado por la bobina
pasa por cero, en t = 0 el flujo resultante 1 debe estar desplazado en /2 radianes
respecto al eje magntico de la fase a. El flujo p est desplazado en el ngulo
respecto a 1, ya que Vp est desfasado respecto a V1 en (figura 7.4.8).

Vp

Va
V1

Vi

1
I1

- 0

m
Im

I f'

Figura 7.4.9. Diagrama fasorial completo como


generador.

Con estos antecedentes se puede dibujar el diagrama fasorial completo de las figura
7.4.9, donde el tringulo de los flujos es semejante con el tringulo de las tensiones.

Captulo 7 : mquina sincrnica.

7.5

7-146

Potencia y momento

En funcionamiento estacionario como generador, la potencia absorbida en el eje es


igual a la potencia entregada en los terminales ms las prdidas
Pmec = P1 + Pperd

(7.5.1)

Si en primera aproximacin se desprecia las prdidas elctricas y mecnicas se tiene


que

1
T = 3 { V1I*1 }
p

(7.5.2)

de donde se logra
*
j
3 p V1 Vp
3 p V1Vp e
T =
V1
=
j

1 jX1
1
X 1

T=

3pV1Vp
1 X 1

sen .

(7.5.3)

Se aprecia que el momento de origen electromagntico tiene un valor mximo


TM =

3 pVV
1 p

(7.5.4)

1X 1

directamente proporcional a la excitacin


Vp e inversamente proporcional a la
reactancia sincrnica X1.
Con las referencias supuestas, el
momento es positivo como motor ( < 0)
y negativo como generador (figura
7.5.1). En el rango - /2 < < /2 los
puntos de trabajo estacionarios son
estables, lo que se expresa mediante la
desigualdad
T
Tc
<

(7.5.5)

Motor

T
TM
TC

Generador
Figura 7.5.1. Caracterstica torque-ngulo

Captulo 7 : mquina sincrnica.

7-147

Para
= 2 la mquina desarrolla el mximo momento posible para una
determinada condicin de excitacin y se dice que alcanza el lmite de estabilidad
estacionario. Para momentos de carga mayores el equilibrio de momentos (2a. Ley de
Newton) slo puede ser satisfecho mediante un cambio de velocidad, por lo que la
mquina pierde el sincronismo.
Resulta conveniente derivar una expresin alternativa a (7.5.3) para el momento.
T

Vp

-T

p
I 1X1
V1

I1X1senT=V1sen

T
I1

Figura 7.5.2. Relacin entre ngulo de carga


y ngulo de torque.
Del diagrama fasorial de la figura 7.5.2 se desprende la relacin
I1 X 1 sen( T ) = V1 sen
o
V
I1 sen T = 1 sen ,
X1
que reemplazada en (7.5.3) permite escribir
T=

3p
V I sen T ,
1 p 1

(7.5.6)

expresin que al considerar (7.4.13) toma la forma


T = p

3
L I I 2 sen T = pLf 1 I f I 1 2 sen T ,
2 1f f 1

(7.5.7)

donde el ngulo de torque T corresponde al ngulo entre las distribuciones de


induccin del estator y del rotor, como se ilustra en la figura 7.5.3. La expresin para el

Captulo 7 : mquina sincrnica.

7-148

momento en trminos de las corrientes y el ngulo de torque es fsicamente ms


significativa, pero, como Vp y V1 habitualmente son constantes, la relacin (7.5.3)
resulta ms simple de evaluar y por eso se la prefiere a (7.5.7).

d
q

p
Vp

Va
Vi

V1

I1
Figura 7.5.3. Relacin entre los desplazamientos espaciales de distribuciones
de flujo y desfasamientos de tensiones inducidas.
En lo que a la dependencia del momento del seno del ngulo se refiere, la mquina
sincrnica es anloga al pndulo y comparte con ste la capacidad de oscilar. La figura
7.5.4 ilustra la analoga.
Supngase ahora que la mquina en vaco (Tc = 0) sufre una perturbacin. La ecuacin
de equilibrio dinmico establece que con posterioridad a la perturbacin rige
J d
= T
p dt
Como = 1 +
J d 2
= T
p dt 2

(7.5.8)
d
dt

1 = cte , la ecuacin toma la forma


(7.5.9)

Si la oscilacin es lo suficiente lenta para que pueda suponerse que las variables
elctricas permanecen en estado estacionario, se puede escribir

Captulo 7 : mquina sincrnica.

7-149

3 pV1V p
J d 2
=
sen
2
p dt
1 X 1

(7.5.10)

y para ngulos pequeos (sen )


3p 2V1Vp
d 2
+
= 0
dt 2
J 1 X 1

(7.5.11)

aprecindose que la frecuencia angular de oscilacin vale


=

3 p 2V1Vp
J 1 X 1

ce
.
J

(7.5.12)

d 2
= Tm sen
dt 2

Tm sen

Tm
Figura 7.5.4. Analoga mquina sincrnica-pndulo.
Al evocar la expresin para la frecuencia natural de un sistema masa-resorte, se
aprecia que la mquina sincrnica acta como un resorte electromagntico de rigidez
ce. La frecuencia de oscilacin natural /2 normalmente es del orden de 1 a 2 Hz.
Para atenuar estas oscilaciones la mquina sincrnica suele estar equipada en el rotor
con un devanado especial, conocido como jaula de amortiguacin, cuyo movimiento
relativo respecto al campo giratorio induce corrientes en l. La energa disipada por
estas corrientes en la jaula de amortiguacin proviene del movimiento oscilatorio, por lo
que la amplitud de ste es atenuada rpidamente.

Captulo 7 : mquina sincrnica.

7.6

7-150

Condiciones de funcionamiento especiales

7.6.1 Cortocircuito estacionario


Cuando las tres impedancias de carga son cero, las tensiones en los terminales de la
mquina tambin valen cero y se dice que la mquina est en cortocircuito trifsico
simtrico. Es un estado de funcionamiento en el que los flujos en la mquina estn
relacionados en forma especialmente simple si se desprecia el efecto de R1.
eje d

m
p

eje q

Figura 7.6.1. Esquema de flujos generados en


cortocircuito trifsico estacionario.
La condicin V1 = 0 exige que en condiciones estacionarias el flujo enlazado por cada
fase del estator debe ser cero. Esto slo puede ser satisfecho si en el devanado del
estator circula un sistema de corrientes trifsicas de amplitud y fase tales que produzca
un campo giratorio que anule el enlace de flujo producido por el rotor y el enlace de flujo
de dispersin. La figura 7.6.1 ilustra esquemticamente esta situacin, descrita en
trminos de tensiones y corrientes por los circuitos equivalentes por fase de la figura
7.6.2 y el diagrama fasorial correspondiente a estos ( figura 7.6.3). El uso consistente
de la convencin carga permite asociar los fasores de corriente en oposicin de fase
con distribuciones espaciales de fmm desplazadas relativamente en radianes. Se
puede apreciar que la reaccin de armadura acta en oposicin al campo y la fmm
resultante, proporcional a I m, determina un flujo resultante en el entrehierro igual y
opuesto al flujo de dispersin, de manera que el flujo total enlazado por cada fase del
estator es nulo, como lo exige la condicin del cortocircuito V1 = 0.

Captulo 7 : mquina sincrnica.

X1

7-151

X1

Xm1

I1

I1
Vi

Xm1

Vp

I f'

Vi
Im

Figura 7.6.2. Cortocircuito trifsico estacionario.


Vp

Vp

jI1Xm1

Tringulo de Potier.

Vi

I1

If'
Im

I1

Figura 7.6.3
Diagrama fasorial en cortocircuito

Vi
I' m

I1 =gI1

Ifc

If

Figura 7.6.4.
Cortocircuito simtrico estacionario,
el tringulo de Potier.

Como el flujo de dispersin del estator con corriente nominal es del orden de un 20%
del flujo nominal, en cortocircuito el circuito magntico principal no est saturado.
De
I1 =

Vp
X1

1L1f I f
1L1 2

= If

(7.6.1)

se aprecia que la relacin entre corriente de campo y corriente de armadura es lineal e


independiente de la frecuencia angular 1 , comportndose la mquina como un
transformador de corriente.
Las relaciones entre Im, I1 e I f representadas en la figura 7.6.2 pueden ser traducidas a
relaciones entre corrientes de campo equivalentes y, como tales, ser representadas en
el plano Vp(If) de la caracterstica de magnetizacin, donde determinan el as llamado
tringulo de Potier (figura 7.6.4), cuyo cateto horizontal representa el efecto

Captulo 7 : mquina sincrnica.

7-152

desmagnetizante de la reaccin de armadura y cuyo cateto vertical representa la


tensin inducida por el flujo resultante en el entrehierro, que es igual a la cada de
tensin en la reactancia de dispersin. Como Vi no es medible, la determinacin
experimental del tringulo de Potier requiere de la realizacin de un ensayo con tensin
nominal y carga reactiva pura.
7.6.2

Carga reactiva inductiva pura

Considrese que el cortocircuito haya sido reemplazado por una carga reactiva
inductiva simtrica de manera que circule corriente nominal y la tensin en bornes sea
la nominal. Las referencias sean las de la figura 7.6.5. Con ellas se construye el
diagrama fasorial de la figura 7.6.6. Se puede apreciar que la relacin entre las
corrientes y, por ende, entre las fmms, es similar a la existente en cortocircuito. Este
hecho permite determinar el tringulo de Potier en la forma ilustrada en la figura 7.6.7,
donde, al utilizar la caracterstica de vaco como equivalente a la caracterstica de
magnetizacin, se hace abstraccin del flujo de dispersin del devanado del inductor
(aproximacin a un circuito magntico serie).
I1

Vi

Xm1

X1

jI1X1
V1

XL

V1

Vi

Vp

Figura 7.6.5. Carga reactiva inductiva.


If'
I1
Supngase por un momento que la reactancia de
Im
I1
dispersin (de Potier) fuese conocida, entonces
se podra determinar Vi a partir de V1 y con ello Figura 7.6.6.
I m. Sumando a I m el efecto desmagnetizante de Carga reactiva inductiva,
la reaccin de armadura I 1 se lograra la corriente determinacin del tringulo de Potier.
de excitacin necesaria If . El tringulo ABD
corresponde al tringulo de Potier. En la prctica se desconoce X1 e I 1, por lo que se
procede fijando en punto A(V1, If) para copiar luego el trazo AC de longitud Ifc
(obtenida del ensayo en cortocircuito). Por C se traza una paralela a la caracterstica
del entrehierro, cuya interseccin con la caracterstica de vaco (obtenida del ensayo en
vaco) determina el punto D(Vi , I m). Una perpendicular desde D a la recta AC
determina el punto B(V1,I m), con el que se completa el tringulo de Potier.

Captulo 7 : mquina sincrnica.

7-153

Vi
V1

de Potier

C B

A
I1'
I fc

Vicc
0

Ifc

I m'

If

Figura 7.6.7. Determinacin del tringulo de


Potier.
La inductancia de Potier se determina como
X pot X 1 =

7.7

Vi V1
I1

(7.6.2)

Determinacin experimental de la reactancia sincrnica

Del circuito equivalente de la figura 7.6.2, correspondiente al cortocircuito estacionario,


se desprende que
X1 =

Vp
I cc

(7.7.1)

Sin embargo, esta expresin no se puede evaluar directamente, ya que no es posible


medir simultneamente Vp e Icc .
Esta dificultad se puede superar tomando en cuenta que las fmms resultantes, en
cortocircuito como en vaco, actan a lo largo del mismo circuito magntico, lo que
permite invocar en ambos casos la misma caracterstica de magnetizacin.
Considrese las caractersticas de vaco y de cortocircuito de las figuras 7.7.1. y 7.7.2

Captulo 7 : mquina sincrnica.

7-154

Vp

I1

V1n

I1n

I cc
I cc*
0

If0 *

Ifo

If

Figura 7.7.1. Determinacin de la


reactancia sincrnica.

I f0*

I fo

I fc

If

Figura 7.7.2. Determinacin de la reactancia


sincrnica.

En vaco la corriente de campo Ifo determina una tensin inducida igual a la tensin
nominal. Al cortocircuitar la armadura se desarrolla la corriente de cortocircuito Icc de
magnitud tal que Icc X1 = Vn. Entrando con Ifo en el eje de abscisas de las
caractersticas de vaco y de cortocircuito se determina los valores de tensin y de
corriente correspondientes a esa corriente de campo, que permiten evaluar (7.7.1)
aproximadamente (en cortocircuito no hay saturacin).
Para obtener parmetros relativamente independientes del tamao de la mquina es
usual el empleo de valores relativos (o en por unidad). Los valores base, o de
referencia, son los valores nominales para tensin y corriente, que a su vez determinan
los valores base para impedancias Zb = Vn / In y para potencias Pb = 3Vn In .
La reactancia sincrnica en [pu] vale
X 1[ pu ] =

X 1 [ ]
V
I
1
= n n =
,
Z b [ ]
I cc Vn
I cc [ pu ]

(7.7.2)

es decir, en [pu] la reactancia sincrnica es numricamente igual al valor recproco de


la corriente de cortocircuito estacionaria.
De la figura 7.7.2 se desprende la proporcionalidad
I
In
= fc
Icc
If 0

(7.7.3)

Captulo 7 : mquina sincrnica.

7-155

de la que se concluye, al considerar (7.7.2), que la reactancia sincrnica en [pu]


tambin est dada por el cuociente entre la corriente de excitacin para la cual la
corriente de cortocircuito es igual a la corriente nominal (Ifc ) y la corriente de excitacin
para la cual la tensin de vaco es igual a la tensin nominal (If0 ). El valor recproco de
este cuociente se conoce como razn de cortocircuito SCR.
SCR =

If 0
1
=
I fc
X 1[ pu ]

(7.7.4)

Si en lugar de la caracterstica de magnetizacin se utiliza la caracterstica del


entrehierro (figura 7.7.1), se logra el valor no saturado de la reactancia sincrnica:
X1 =

I
Vn I n
1
= *
= *fc
*
Icc Vn
I cc [ pu ]
If 0

(7.7.5)

Para mquinas modernas, provistas de reguladores de tensin rpidos, la razn de


cortocircuito vara entre 0,5 y 0,9, lo que implica que la reactancia sincrnica vara entre
2 y 1,1 [pu].
7.8

Funcionamiento en red infinita

La red infinita es una idealizacin de una red real. Puede absorber o entregar cualquier
potencia sin variar su tensin en barras. Su representacin circuital es una fuente ideal
de tensin.

Red

L1
L2
L3

Vlvula

Turb.

W 1 V1

U1

Gen.

F1 F2

Excitacin

Figura 7.8.1. Relativo a la sincronizacin.

Para que una mquina sincrnica pueda


intercambiar energa con una red debe ser
sincronizada previamente con esta. Una
sincronizacin ideal consiste en la conexin
de la mquina a la red sin que se produzca
perturbacin alguna. Esto implica que debe
satisfacerse cuatro condiciones:
la secuencia de fases de la mquina y de
la red debe ser la misma,
la frecuencia de las tensiones de mquina
y red debe ser la misma,
la magnitud de las tensiones de mquina y
red debe ser la misma, y
la fase de las tensiones de mquina y red
debe ser la misma.

Captulo 7 : mquina sincrnica.

7-156

Satisfechas estas condiciones, las estrellas


de tensiones de la figura 7.8.2 coinciden y
el interruptor de la figura 7.8.1, al cerrarse,
une puntos equipotenciales. No circularn
corrientes ni habr momento. Se dice que
la mquina flota en la red (V1 = Vp ; I1 = 0)
y el diagrama fasorial correspondiente es el
de la figura 7.8.4.

L1

U1

r
m
W1
L2

L3
I1

V1
X1
Figura 7.8.2. Relativo a la sincronizacin.
Vp

V1

Vp

V1
=0

I 1=0
Figura 7.8.3. Circuito equivalente de la
mquina conectada a la red
infinita.

Figura 7.8.4. Diagrama fasorial de la


mquina flotante.

Para la mquina conectada a la red infinita rige el circuito equivalente de la figura 7.8.3.
Con las referencias indicadas en ese circuito se tiene que la potencia aparente
absorbida por la mquina desde la red est dada por
S = P + jQ = 3 V1I*1

(7.8.1)

Pero
V1 Vp
V1 Vp e j
I1 =
=
jX 1
jX1

(7.8.2)

Reemplazando (7.8.2) en (7.8.1) y separando luego parte real e imaginaria se logra


P=

Q=

3V1Vp
X1

sen

3V12 3V1Vp

cos
X1
X1

(7.8.3)

(7.8.4)

Captulo 7 : mquina sincrnica.

7-157

Como la tensin de la red infinita es por definicin constante, el monto de la potencia


activa o reactiva intercambiada entre mquina y red queda determinado por el ngulo
de carga y la corriente de excitacin If , que fija el valor de Vp. Interesa entonces
determinar el efecto especfico de estas variables.
7.8.1

Variacin de la excitacin

Supngase ahora que la potencia activa sea cero (nada conectado al eje). Entonces
segn (7.8.3) el ngulo de carga es cero y la mquina, vista desde la red, se
convierte en un inductor o un condensador sincrnico, dependiendo el signo de la
potencia reactiva de si est subexcitada (Vp < V 1) o sobreexcitada (Vp > V 1):
Q =

3V1
V Vp .
X1 1

(7.8.5)

Los diagramas fasoriales de la figura 7.8.5 ilustran esta situacin.

Mediante la variacin de la corriente de excitacin es posible regular la potencia


reactiva intercambiada entre la mquina y la red.
inductor

capacitor

jI1X1

V1

Vp

jI1X1

V1

Vp
I1

I1
a)

b)

Figura 7.8.5 Relacin de fase entre tensin y corriente en los


terminales de una mquina
a) sobreexcitada
b) subexcitada

7.8.2 Variacin del momento


Si se aplica un momento motriz externo al eje de la mquina que flota en la red, se
produce la aceleracin del rotor, la que implica un aumento del ngulo con el
consiguiente desfasamiento de Vp respecto a V1 que causa la aparicin de la corriente

Captulo 7 : mquina sincrnica.

7-158

de armadura I1(7.8.2) y la entrega de potencia activa a la red (7.8.3). Cuando el


momento electromagntico desarrollado por la mquina iguala al momento aplicado al
eje, la aceleracin desaparece y el ngulo permanece constante. Se establece el
equilibrio estacionario. El diagrama fasorial de la figura 7.8.6 ilustra esta situacin.

L.G. P=cte.

L.G. I=cte.
L.G. Vp=cte.

jI1X1
L.G. Q=cte.
Vp

V1
p

1
Lmite de estabilidad
terico.

T
I1

Figura 7.8.6.
Diagrama fasorial para exitacin 100 %
y T < 0 (generador).

7.8.3

Se aprecia que al variar el momento


aplicado al eje, manteniendo la excitacin
constante, el extremo del fasor Vp recorre
un arco de circunferencia al igual que la
corriente de armadura I1 , lo que implica la
variacin del factor de potencia.
La variacin de la potencia activa tambin
afecta a la potencia reactiva. Si esta ha
de permanecer constante, es necesario
actuar sobre la corriente de campo y
modificar el mdulo de Vp , como se
desprende del lugar geomtrico de Vp
para Q=cte de la figura 7.8.6.
Mediante la accin combinada sobre el
momento aplicado al eje y sobre la
corriente de excitacin se puede ajustar
cualquier condicin de funcionamiento de
la mquina.

Lugar geomtrico de la corriente

Del circuito equivalente de la figura 7.8.3 se desprende que:


V1 Vp = jX 1I1

(7.8.6)

de donde se despeja
I1 = j

Vp j
V1
+ j
e
X1
X1

(7.8.7)

que, para V1 y Vp constantes, corresponde a una circunferencia en el plano complejo,


cuyo centro se encuentra sobre el eje imaginario negativo y cuyo radio est dado por
Vp / X1, como puede apreciarse en la figura 7.8.7.
En cambio los lugares geomtricos de la corriente I1 para potencia activa constante,

Captulo 7 : mquina sincrnica.

7-159

{ V1I *1 } = V1{ I1} = cte. ,

(7.8.8)

son rectas paralelas al eje imaginario.


De manera anloga se establece que los lugares geomtricos de la corriente para
potencia reactiva constante son rectas paralelas al eje real.
Los puntos del semiplano superior describen estados de funcionamiento como motor
(< 0) y los del semiplano inferior como generador ( > 0).
En lo que a la potencia reactiva se refiere, para puntos en el semiplano izquierdo, la
mquina absorbe potencia reactiva capacitiva y para puntos en el semiplano derecho,
absorbe potencia reactiva inductiva.
Xc

XL

Im
L.G. I 1 =cte .

V1
X1

Motor

L.G. P=cte.

Diagrama circular o
L.G. Vp=cte.

L.G. Q=cte.

Re
V1

Vp j
e
X1

Generador

I1

Figura 7.8.7. Diagrama circular de la corriente I1


y otros lugares geomtricos.
Si bien cada punto del plano describe unvocamente un estado de funcionamiento
caracterizado por sendos valores para la potencia activa y la potencia reactiva, debe
tenerse en cuenta que hay lmites impuestos por los valores mximos admisibles para
la potencia activa, limitada por la potencia mxima de la mquina motriz, la corriente de
armadura, limitada por el calentamiento mximo admisible para el devanado, la
corriente de excitacin, limitada por el calentamiento admisible para el devanado de
campo y el ngulo (lmite de estabilidad estacionario), de manera que no todos los
puntos del plano corresponden a condiciones de operacin segura.

Captulo 7 : mquina sincrnica.

I fmax

cos nom

7-160

I 1nom
Pmax

Limite de
estabilidad
terico.

prctico
.

Para orientar al operador de la mquina,


se ha concebido la carta de operacin,
que representa el rea de operacin
segura en el plano P-Q , que est limitado
por
los
lugares
geomtricos
correspondientes a los valores mximos
admisibles para las diferentes variables.
Se puede apreciar la correspondencia de
la carta de operacin de la figura 7.8.8
con los lugares geomtricos de la figura
7.8.7, si se tiene en cuenta que los
correspondientes ejes coordenados estn
girados en 90.

As, de la carta de operacin para un


generador de la figura 7.8.8 se desprende
que el funcionamiento con corriente
Figura 7.8.8. Carta de operacin
nominal no es posible con factores de
como generador.
potencia inferiores al nominal (usualmente
0,8 o 0,9), ya que la corriente de campo
necesaria excedera a la corriente de campo mxima admisible, o que el funcionamiento
subexcitado est limitado por el lmite de estabilidad prctico (potencia mxima sin
riesgo de prdida del sincronismo), que considera una reserva de un 10% de la
potencia mxima en relacin con el correspondiente lmite de estabilidad terico para un
determinado grado de excitacin.
10 % Pmax

8. Mquina asincrnica
8.1

Introduccin

En 1987 la mquina asincrnica cumpli 100 aos. Durante ese tiempo se ha


convertido en el motor elctrico ms difundido y se puede afirmar sin temor a
exageraciones que es el motor del desarrollo industrial del siglo XX.
Durante su ms que centenaria existencia el progreso tecnolgico, que se manifiesta a
travs de una teora cada vez ms completa y a travs de la disponibilidad de
materiales magnticos y dielctricos de caractersticas mejoradas, ha permitido reducir
en forma muy importante el peso por unidad de potencia de la mquina asincrnica, lo
que, junto con el mejoramiento del proceso de fabricacin, la ha convertido en el motor
elctrico ms barato.
La mquina asincrnica se construye en un rango de potencias que cubre todo el
amplio espectro de las necesidades industriales, desde unos pocos Watt de los
aparatos electrodomsticos, hasta algunos Megawatt de las bombas de alimentacin de
calderas en centrales nucleares.
La gran mayora de las mquinas asincrnicas funcionan conectadas a redes de tensin
y frecuencia fijas, lo que les da el carcter de mquinas de velocidad prcticamente
constante.
La disponibilidad de convertidores de frecuencia sobre la base de semiconductores
controlados ha permitido el levantamiento de la restriccin histrica velocidad
constante y ha abierto a la mquina asincrnica campos de aplicacin que
anteriormente le estaban vedados, en un momento en que, por razones econmicas y
ecolgicas, aumenta la demanda por un uso ms racional de la energa.
As, el uso de motores asincrnicos con velocidad ajustable mediante un convertidor de
frecuencia en el accionamiento de bombas y ventiladores ha permitido importantes
ahorros de energa en relacin con accionamientos de velocidad constante. Las
locomotoras de ltima generacin alcanzan sus caractersticas superiores porque
incorporan motores asincrnicos de velocidad variable como motores de traccin y
muchos otros accionamientos han encontrado en el conjunto convertidor de frecuencia
y motor asincrnico la mejor solucin tcnico-econmica.
Estos usos nuevos de la mquina asincrnica requieren de esquemas de control que
utilizan modelos dinmicos de la mquina, lo que obliga a un cambio de nfasis en el
tratamiento de la teora, cuya formulacin debe permitir su posterior ampliacin al
estado de funcionamiento transitorio.

Captulo 8 : mquina asincrnica

8.2

8-162

Caractersticas constructivas

La figura 8.2.1 muestra una representacin en explosin de un motor asincrnico tpico


para el rango de tensin entre 2 y 6,6 kV y el rango de potencia entre 315 y 3550kW a
3000r.p.m.

Figura 8.2.1 Vista en explosin de un motor asincrnico con rotor


de jaula, tpico para el rango de tensin 2-6,6 kV y el
rango de potencia 315-3550 kW a 3000 r.p.m.

La parte activa del estator y del rotor est armada de chapas silicosas de 0,5 mm de
espesor en las que se ha estampado ranuras para el alojamiento del devanado (figura
8.3.1).
En mquinas grandes se utiliza ranuras abiertas, en las que se monta bobinas
prefabricadas, mientras que en mquinas pequeas se utiliza ranuras semicerradas, en
las que se forma la bobina directamente, usualmente con mquinas bobinadoras.

8-163

Captulo 8 : mquina asincrnica

El devanado del estator es normalmente trifsico, conexin estrella sin neutro (figura
8.2.2).

W2

U1

U2

V1

V2

W1

Figura 8.2.2. Devanado trifsico, p=2 , q=2.


Para el rotor se puede elegir entre dos tipos de devanados. En el caso de condiciones
de arranque muy severas, o cuando se opta por regulacin de velocidad con la
mquina conectada a una red de frecuencia fija, se utiliza en el rotor un devanado
trifsico conectado en estrella y unido a tres anillos rozantes. En ausencia de
condiciones especiales, se prefiere, por su menor costo y mayor robustez, una forma de
devanado conocida como jaula de ardilla. Hasta potencias del orden de 250 kW la jaula
se realiza en aluminio fundido y sobre esta potencia con barras de cobre y bronce en
las versiones de jaula doble o de jaula de barras profundas.
Contrariamente al caso de la mquina sincrnica, caracterizada por un entrehierro
relativamente grande, el entrehierro de la mquina asincrnica debe ser lo ms
pequeo posible, variando entre 0,3 mm para mquinas pequeas y 2,7 mm para
motores de 3000 kW. Esto impone estrechas tolerancias de fabricacin y montaje.
A medida que aumenta la potencia del motor cobra mayor importancia la disipacin del
calor causado por las prdidas. Esto se traduce en un cuidadoso diseo de los circuitos
de refrigeracin, tratando de llegar con el fluido refrigerante (normalmente aire)
directamente a los lugares donde se producen las prdidas.

8-164

Captulo 8 : mquina asincrnica

8.3

Principio de funcionamiento

Considrese que por el devanado trifsico del estator, conectado a una red de tensin y
frecuencia constantes, circulen corrientes trifsicas simtricas. El devanado del rotor
est inicialmente abierto.
Las corrientes en el devanado del estator dan lugar a una distribucin espacial de fmm,
que determina una distribucin de flujo representada, para un cierto instante y a lo
largo de un paso polar, en la figura 8.3.1, que se desplaza a lo largo del entrehierro con
velocidad angular mecnica igual a 1 /p (velocidad sincrnica).
1 paso polar

p /2

p /2

Figura 8.3.1

p /2

p / 2

Chapas del estator y rotor, circuito magntico


principal y distribucin del flujo en vaco.

Este campo giratorio induce en las fases del devanado del rotor un sistema de
tensiones simtrico.
Supngase ahora que se ha cerrado el devanado del rotor. Las tensiones inducidas
dan lugar a corrientes, cuya interaccin con la onda de induccin giratoria produce un

8-165

Captulo 8 : mquina asincrnica

momento que impulsa al rotor en el sentido de giro de campo, ya que, de acuerdo con
la regla de Lenz, el efecto es siempre tal que tiende a oponerse a la causa que lo
produce, y en este caso la causa de la corriente es el movimiento relativo entre campo y
los conductores del devanado del rotor.
Cuando el rotor ha alcanzado la velocidad angular m /p, la velocidad del campo
giratorio en relacin con el devanado del rotor se ha reducido a
2 1 m
=

p
p
p

(8.3.1)

y la frecuencia angular de las tensiones inducidas en el rotor se ha reducido a 2.


Se aprecia que (8.3.1) es equivalente a la condicin para conversin continua de
energa desarrollada en el captulo 5, lo que implica que el motor asincrnico produce
un momento medio distinto de cero para cualquier velocidad distinta a la sincrnica.
La diferencia entre la velocidad sincrnica y la velocidad del rotor, referida a la
velocidad sincrnica, se denomina deslizamiento
s=

1 m 2
=
1
1

(8.3.2)

Cuando el rotor alcanza la velocidad sincrnica, desaparece el movimiento relativo


entre rotor y campo giratorio y con l las tensiones inducidas en el rotor, la corriente en
el devanado del rotor se hace cero y con ella desaparece el momento. El motor no
puede girar estacionariamente con velocidad sincrnica, pues, para vencer los
inevitables momentos debidos al roce en los descansos, el motor debe desarrollar un
momento electromagntico, lo que slo es posible si gira a una velocidad inferior a la
sincrnica. De ah la denominacin de motor asincrnico.
La accin de la jaula se puede interpretar formalmente en trminos de un devanado
equivalente de tantas fases como barras posee la jaula, donde cada fase posee media
vuelta.
Para apreciar esta equivalencia, considrese un devanado anular de m 2 espiras,
distribuidas en forma regular sobre el yugo del rotor (figura 8.3.2a). Las tensiones
inducidas en las espiras, debidas a la variacin peridica del flujo en las secciones del
yugo enlazadas por estas, dan lugar a corrientes que forman un sistema de corrientes
simtrico de m2 fases, cuya suma en todo instante es cero. Por lo tanto, nada cambia
desde el punto de vista de la distribucin de las corrientes si las espiras se
interconectan en estrella en la forma indicada en la figura 8.3.2b y se suprime el
neutro. El posterior reemplazo de la estrella de la figura 8.3.2b por un polgono
equivalente en la figura 8.3.2c tampoco altera la distribucin de corrientes y deja de
manifiesto la equivalencia entre las situaciones de las figuras 8.3.2c y 8.3.2a.

8-166

Captulo 8 : mquina asincrnica

a)

b)

c)

Figura 8.3.2. Equivalencia entre un devanado anular y


un devanado jaula de ardilla.
Para la tensin inducida en una espira de la figura 8.3.2a vale
$
V = 4,44 f
y

(8.3.3)

y como el valor mximo del flujo en el yugo es igual a la mitad del valor mximo del flujo
por polo, segn se puede apreciar en la figura 8.3.1, esta relacin puede reescribirse
en trminos del flujo por polo como
$
V = 4 ,44 f 12
p

(8.3.4)

y reinterpretarse formalmente como la tensin inducida en una bobina ficticia de paso


completo de vuelta, perteneciente a un devanado de m 2 fases, donde m 2 es igual al
nmero de barras de la jaula.
8.4

Ecuaciones de equilibrio elctricas

Las ecuaciones de equilibrio elctricas en trminos de las variables de terminales se


logran aplicando la ley de Faraday al devanado en cuestin.
Para determinar la tensin inducida por el flujo enlazado por una fase resulta
conveniente descomponer el campo resultante en un campo debido a las corrientes del
estator y un campo debido a las corrientes del rotor y aplicar el principio de
superposicin.
Considrese primeramente las corrientes en las tres fases simtricamente desplazadas
del estator, que en estado sinusoidal estacionario simtrico tienen la forma

8-167

Captulo 8 : mquina asincrnica

i 1a = 2I 1 cos( 1t 1 )

i 1b = 2I 1 cos( 1t 1

2
3

i 1c = 2I1 cos( 1t 1 + 23

)
)

(8.4.1)

y determinan en el entrehierro una distribucin de fmm giratoria


f1 =

f d,1
3 2I1N1 4
cos( px 1 1t + 1 )
2 2 p =6 g +1

(8.4.2)

cuya componente fundamental, de acuerdo con lo visto en el captulo 4, determina la


onda de induccin giratoria
B1 ( x 1, t ) =

3 4 0
2

2I 1N1f d 1,1
2p

cos( px1 1t + 1 ) ,

(8.4.3)

donde es el entrehierro efectivo, que incluye el efecto de la saturacin y de las


ranuras.
El flujo enlazado por la fase a del estator, cuyo eje magntico coincide con el origen de
la coordenada x1 , se calcula como
1a

Rl
= N1f d 1
p

1a =

+/ 2

B ( x ,t ) d ( px )
1

(8.4.5)

/ 2

2
34
Rl
0
N
f
2I1 cos( 1t 1 )
(
)
1
d
1
,
1
2
p 2

(8.4.6)

y la tensin inducida en esa fase vale


v 1a =

2
d 1a
3
4 R l
= 0
Nf
1 2I1 sen( 1t 1 ) .
2 ( 1 d 1,1 )
dt
2
p

(8.4.7)

Pasando a notacin compleja queda:


2

3
4 Rl

v 1a = j 1 0
Nf
I 2e j 1t
2 ( 1 d1,1 ) 1
2
p

v 1a = V11 2e j 1t ,

(8.4.8)

(8.4.9)

8-168

Captulo 8 : mquina asincrnica

donde

V11 = j 1Lm 1I1

(8.4.10)

es el fasor de la tensin inducida por el campo giratorio y


2
3
4 Rl
0
N
f
(8.4.11)
(
)
1
d
1
,
1
2
p 2
es la inductancia propia de campo giratorio del devanado del estator.

Lm 1 =

La inductancia de campo giratorio engloba el efecto inductivo de las tres corrientes del
estator y reduce la situacin trifsica a una monofsica equivalente.
Por otra parte, el campo giratorio tambin enlaza al devanado del rotor (figura 8.3.1),
que se mueve respecto al estator con velocidad angular m [rad.el./s]. La coordenada
fija al rotor est relacionada con la coordenada fija al estator a travs de la relacin
(figura 8.4.1)
px1 = m t + px 2 = (1 s ) 1t + px 2

(8.4.12)

px 1

mt

px 2

a'

a
x 1=0

px
a'

a
x 2=0

Figura 8.4.1. Relacin entre la coordenada del estator x1


y la coordenada del rotor x2.
En trminos de la coordenada fija al rotor la onda de induccin B1 queda descrita por
3 4 0 2I1N1fd 1,1
B1 ( x 2 , t ) =
cos( px 2 s 1t + 1 )
(8.4.13)
2
2p
por lo que el flujo enlazado por la fase a del rotor se calcula como
2 a = (N2 f d1, 2 )

R l
p

+ /2

B ( x ,t )d ( px )
1

(8.4.14)

/2

y la tensin inducida en la fase de referencia a del rotor vale


v 2a =

d 2a
3
4 Rl
= 0
(N1fd 1,1 )(N2 fd 1,2 )s 1 2I1 sen( s 1t 1 )
dt
2
p 2

(8.4.15)

8-169

Captulo 8 : mquina asincrnica

v 2a = V21 2e j s 1t ,

(8.4.16)

donde V21 = js 1L21I 1

(8.4.17)

es el fasor de la tensin inducida por el campo giratorio del estator en una fase del rotor
y
3
4 Rl
L21 = 0
(8.4.18)
(N1fd 1,1 )(N2 fd 1,2 ) ,
2
p 2
es la inductancia mutua de campo giratorio entre el devanado del estator y una fase del
rotor. Para un rotor con devanado del tipo jaula N2=1/2 y fd1,2 =1.
Considrese ahora las m 2 corrientes del rotor

2
i 2 i = 2I 2 cos s 1t 2
i 1)
(
m2

con

i = 1, 2, ..., m2 ,

(8.4.19)

que dan lugar a una distribucin de fmm giratoria, cuya componente fundamental est
dada por
f2 ( x 2 ,t ) =

m 2 4 2I 2 (N2 fd 1, 2 )
cos( px 2 s 1t + 2 )
2
2p

(8.4.20)

y que, con entrehierro constante, determina la onda de induccin giratoria


B2 ( x 2 , t ) =

2I 2 (N2 f d 1, 2 )

m 2 4 0
2

2p

cos( px 2 s 1t + 2 )

(8.4.21)

cuyo flujo induce en la fase a del rotor la tensin


2

m
4 Rl

v 2a = js 1 2 0
N f
2I 2e j s 1t
2 ( 2 d 1, 2 )
2
p

v 2a = V22 2e j s 1t ,
donde

V22 = js 1Lm 2 I 2

(8.4.22)

(8.4.23)
,

es el fasor de la tensin inducida por el campo giratorio y

(8.4.24)

8-170

Captulo 8 : mquina asincrnica

Lm 2 =

m2
4 Rl
2
0
N f
2 ( 2 d 1, 2 )
2
p

(8.4.25)

es la inductancia propia de campo giratorio del devanado del rotor.


En forma anloga se determina la tensin inducida por el campo giratorio del rotor en
una fase del estator (a), logrando

(8.4.26)

con

V12 = j 1L12 I 2

(8.4.27)

L12 =

v 1a = V12 2e j 1t

m2
4 Rl
0
(N1fd 1,1 )(N2 fd 1,2 )
2
p 2

(8.4.28)

inductancia mutua de campo giratorio entre el devanado del rotor y una fase del estator.
Cuando el nmero de fases del estator no es igual al nmero de fases del rotor las
inductancias mutuas de campo giratorio no son recprocas, pues de (8.4.18) y (8.4.23)
se desprende que
3L12 = m 2 L21

(8.4.29)

De (8.4.11), (8.4.18), (8.4.25) y (8.4.28) se establece la siguiente relacin general entre


inductancias de campo giratorio:
L12 L21 = Lm 1Lm 2 .

(8.4.30)

Adems de los campos fundamentales en el entrehierro, las corrientes en los


devanados del estator y del rotor crean campos de dispersin en las ranuras y en el
espacio frontal y campos armnicos en el entrehierro, asociados a sus respectivos
devanados, que tambin se consideran como campos de dispersin de esos
devanados. El efecto inductivo de todos estos campos se engloba en sendas
inductancias de dispersin (L1 y L2).
La aplicacin de la ley de Faraday a las fases representativas del estator y del rotor
equivale a hacer un inventario de las tensiones inducidas por los diferentes campos en
esas fases, por lo que se puede anotar para cada una de ellas en trminos de fasores:
V1 = R1I1 + j 1 (L 1 + Lm 1 ) I1 + j 1L12 I 2

V2 = R2 I2 + js 1 (L 2 + Lm 2 ) I 2 + js 1L21I1

(8.4.31)

El devanado del rotor est normalmente cortocircuitado, lo que implica V2 = 0.

8-171

Captulo 8 : mquina asincrnica

8.5

Circuito equivalente y diagrama fasorial

Los devanados descritos por el sistema de ecuaciones (8.4.31) estn en movimiento


relativo, por lo que las frecuencias de las corrientes en ellos es 1 y s1
respectivamente.
La divisin formal por el deslizamiento s de la ecuacin del rotor cortocircuitado en
(8.4.31) resulta en
0=

R2
I + j 1 (L 2 + Lm 2 ) I2 + j 1L21 I1
s 2

(8.5.1)

ecuacin que puede reinterpretarse fsicamente como correspondiente a un rotor


detenido, ya que ahora la frecuencia angular en ese circuito parece ser 1 en lugar de
s1 .
Con la intencin de avanzar hacia la obtencin de un circuito equivalente por fase,
considrese ahora el reemplazo del devanado m 2-fsico del rotor por uno trifsico,
similar al del estator, excitado con corrientes trifsicas simtricas I 2 tales que
produzcan la misma distribucin de induccin fundamental en el entrehierro que el
devanado original. Esto equivale a exigir que el flujo enlazado por una fase del estator
no debe ser alterado con la substitucin del devanado del rotor, o sea que
Lm1 I 2 = L12 I 2

(8.5.2)

m
3
N1 f d 1,1 I 2 = 2 N2 f d1. 2 I 2 ,
2
2
expresin que explicita la igualdad de las respectivas distribuciones de fmm.
Adems, en el devanado equivalente deben producirse las mismas prdidas y la
energa acumulada en sus campos de dispersin debe ser la misma que en el
devanado original. Esto implica exigir que
m 2 I 22R2 = 3I 2 2 R2
y
m 2 I 22 L 2 = 3I 2 2 L 2

(8.5.3)
(8.5.4)

de donde se logra con (8.5.2) la resistencia y la inductancia de dispersin del devanado


trifsico equivalente
m L
R2 = R2 2 m 1
3 L12

(8.5.5)

8-172

Captulo 8 : mquina asincrnica

y
2

L 2 = L 2

m 2 Lm 1

,
3 L12

(8.5.6)

tambin conocidos como valores reducidos al primario de los respectivos parmetros R2


y L2.
Reemplazando I2, R2 y L2 en (8.4.31) y (8.5.1) en trminos de los correspondientes
valores reducidos al primario y considerando (8.4.29) y (8.4.30), se logra las ecuaciones
V1 = R1I1 + j 1 (L 1 + Lm 1 ) I1 + j 1Lm 1 I 2
0=

(8.5.7)

R2
I + j 1( L 2 + Lm1 ) I 2 + j 1Lm1 I1 ,
s 2

(8.5.8)

que reordenadas toman la forma


V1 = (R1 + j 1L 1 ) I1 + j 1Lm 1 I m

(8.5.9)

0 = 2 + j 1L 2 I2 + j 1Lm 1 Im ,
s

donde se ha introducido la corriente magnetizante


Im = I1 + I2

(8.5.10)

(8.5.11)

una corriente ficticia, que, circulando en el devanado del estator, produce el mismo
efecto magntico en el entrehierro que las corrientes I1 e I2 en conjunto.
I1

I' 2
R1

V1

X1

X'2

Im
Xm1

R2/s

Vi

Figura 8.5.1. Circuito equivalente por fase de la mquina asincrnica.

8-173

Captulo 8 : mquina asincrnica

Las ecuaciones (8.5.9) y (8.5.10) corresponden ahora a una fase de una mquina
trifsica, tanto en el estator como en el rotor, donde ambos devanados tienen el mismo
nmero de vueltas efectivo por fase.
I1
I' 2
R1

X1

V1

X'2

Im
Xm1

Vi

R2
1s
R2' ( )
s

Figura 8.5.2. Circuito equivalente por fase de la mquina asincrnica.


La observacin ms detenida de las ecuaciones (8.5.9) y (8.5.10) permite apreciar que
satisfacen el circuito equivalente de la figura 8.5.1, que est repetido en la figura 8.5.2
con la resistencia asociada a la malla del rotor descompuesta en
R2
1 s
= R2 + R2

s
s

(8.5.12)

V1

Vi
I1
1
I'2

2
Im
I'2

Figura 8.5.3 Diagrama fasorial de la mquina


asincrnica

para rescatar la similitud con el


transformador con carga resistiva,
una imagen ampliamente usada en
la teora clsica de la mquina
asincrnica. Sin embargo, debe
tenerse en cuenta que, debido a la
existencia del entrehierro, la
reactancia de magnetizacin Xm1 de
la mquina asincrnica es mucho
menor que la de un transformador
(1:30).
El circuito equivalente est formado
por dos mallas, respectivamente
representativas del estator y del
rotor.
Ntese la similitud de la malla
representativa del estator con la
malla correspondiente en el circuito
equivalente
de
la
mquina
sincrnica, representado en la figura

8-174

Captulo 8 : mquina asincrnica

7.4.3 del captulo 7. En ambos casos la tensin en la reactancia Xm1 corresponde a la


tensin inducida en una fase del devanado del estator por el campo resultante en el
entrehierro.
Con las referencias de la figura 8.5.1, se puede construir el diagrama fasorial de la
figura 8.5.3.
Si bien las magnitudes representadas en el diagrama fasorial son las tensiones y
corrientes del circuito equivalente, no debe olvidarse la relacin de estas magnitudes
con los flujos y las fmms de la mquina real, por lo que la interpretacin de este
diagrama se enriquece, si se tiene en cuenta que los ngulos entre los fasores de
corriente son iguales a los ngulos de desplazamiento espacial entre las
correspondientes distribuciones de fmm (induccin), segn se desprende de las
relaciones (8.4.13) y (8.4.21). Interpretado en esta forma, el diagrama fasorial es
tambin la representacin simblica de las distribuciones giratorias de fmm y de
induccin.
Vi
R 2=0

F2

T =180

Fm

F1

I1

I'2
R1

X 1

Im

X' 2

R 2'/s

Vi

V1
X m1

V1
Vi

I2'

R 2=0

Im
I1
T =180

Figura 8.5.4. Disposicin de fuerzas magnetomotrices


y diagrama fasorial para el caso R 2=0.
Esta equivalencia es de gran valor para entender el funcionamiento de la mquina en
trminos de los conceptos desarrollados en el captulo 5, donde se estableci que el

8-175

Captulo 8 : mquina asincrnica

momento electromagntico naca de la tendencia a alinearse de las distribuciones de


induccin del estator y del rotor y que cesaba cuando estas distribuciones lograban
alinearse. As se aprecia que en una mquina asincrnica hipottica cuya resistencia
del rotor fuese nula, el ngulo de torque sera de 180, como muestra el diagrama
fasorial de la figura 8.5.4. En consecuencia, tal mquina no podra desarrollar momento.

8.6

Potencia y momento

En estado sinusoidal estacionario la potencia media suministrada al campo magntico


es cero, por lo que rige el siguiente balance de potencia:
Pmec = P1 Pcu 1 Pcu 2

(8.6.1)

a partir del cual se puede obtener una expresin para la potencia mecnica en trminos
de las variables elctricas.
De (8.5.9) y (8.5.10) se tiene que

P1 = 3 V1 I1 = 3R1I12 + 3 j 1Lm 1Im I1


0=3

R2 2
I + 3 j 1Lm 1Im I 2
s 2

(8.6.2)

(8.6.3)

En (8.6.2), la expresin

3 j 1Lm 1I m I1 = P1 Pcu1 = PCG1

(8.6.4)

se identifica como la potencia asociada al campo giratorio del estator PCG1 .


Considerando que segn (8.5.11) I2 = I m I 1 , se establece que

j 1Lm 1Im I 2 = j 1Lm 1I m I1 ,

(8.6.5)

lo que, reemplazado en (8.6.3), permite escribir


Pcu 2
= PCG1 .
s

(8.6.6)

Con (8.6.4) y (8.6.6) el balance de potencia de (8.6.1) toma la forma


Pmec = PCG 1 Pcu 2

(8.6.7)

8-176

Captulo 8 : mquina asincrnica

Pmec = (1 s )PCG 1
1 s
Pmec =
P
s cu 2

(8.6.8)

(8.6.9)

De esta ltima relacin se desprende que la potencia disipada en la resistencia


(R2(1-s)/s) de la figura 8.5.2 corresponde a la potencia mecnica por fase.
Cabe destacar que de acuerdo con (8.6.7) la potencia del campo giratorio PCG1 se
divide en el entrehierro en potencia mecnica Pmec y potencia transferida al circuito del
rotor Pcu2 de acuerdo con una clave dada por el deslizamiento, segn lo establecen las
relaciones (8.6.6) y (8.6.8).
Desde el punto de vista mecnico se tiene la siguiente relacin entre la potencia
mecnica y el torque:
Pmec = T

m
T
= (1 s ) 1
p
p

(8.6.10)

por lo que, al igualar esta expresin con (8.6.9), se logra


T=

p Pcu 2
p R2 2
=
3
I
1 s
1 s 2

(8.6.11)

Esta relacin confirma desde otra perspectiva la afirmacin hecha al final del prrafo
8.5, que un hipottico motor asincrnico sin resistencias en el rotor no desarrollara
momento.
La corriente I 2 se puede determinar aplicando el Teorema de Thvenin a la malla del
estator del circuito equivalente de la figura 8.5.1, el que de esta manera se reduce al
circuito serie de la figura 8.6.1 .
I2

X'2
VTh

ZTh

Vi

R2'/s

Figura 8.6.1. Equivalente Thevenin del circuito de la figura 8.5.1.

8-177

Captulo 8 : mquina asincrnica

La tensin de Thvenin corresponde a la tensin Vi con I 2 = 0, es decir,


VTh = j X m 1

V1
R1 + j ( X 1 + X m 1 )

(8.6.12)

y la impedancia de Thvenin vale


ZTh =

(R

+ j X 1 ) j X m1

R1 + j ( X 1 + X m 1 )

= RTh + j XTh

(8.6.13)

Tanto VTh como ZTh son constantes para una determinada tensin y frecuencia de la
red de alimentacin. Con (8.6.12) y (8.6.13) se determina
I2 =

VTh
R

Z Th + 2 + jX 2
s

(8.6.14)

que reemplazada en (8.6.11) permite obtener la siguiente expresin para el momento:


T=

VTh2 R2
3p

.
2
1

R2
2
s RTh +
+ ( X Th + X 2 )
s

(8.6.15)

Derivando el denominador de esta expresin respecto al deslizamiento, e igualando


esta derivada a cero, se obtiene el deslizamiento para el cual el momento es mximo
sM =

R2
2
RTh
+ ( X Th + X 2 )

(8.6.16)

que reemplazado en (8.6.15) permite determinar el momento mximo:


TM =

VTh2
3p
2 1 R R 2 + ( X + X ) 2
Th
Th
Th
2

(8.6.17)

El signo positivo corresponde al funcionamiento como motor y el signo negativo al


funcionamiento como generador.
Ntese que el momento mximo TM es independiente de la resistencia del rotor R2,
mientras que el deslizamiento sM para el cual se produce el momento mximo es
proporcional a R2.

8-178

Captulo 8 : mquina asincrnica

En mquinas de potencia superior a 10 kW el efecto de las prdidas en el devanado del


estator sobre las caractersticas de funcionamiento con frecuencia nominal no es
significativo. Asumiendo R1 = 0 , las expresiones (8.6.16) y (8.6.17) se simplifican, ya
que ahora
X 1
RTh = 0
y
X Th =
,
(8.6.18)
1 + 1
X
con 1 = 1 , coeficiente de dispersin del estator,
(8.6.19)
X m1
y quedan como
sM =

R2
X 1
+ X 2
1 + 1

R2
X e

(8.6.20)

y
TM =

2
VTh
V12
3p
3p
=
21 X 1
21 X 1 + X 2 (1 + 1 )
+ X 2
1 + 1

(8.6.21)

En trminos de (8.6.20) y (8.6.21) la expresin para el momento (8.6.15) toma la forma


simtrica:
T=

2TM
s
s
+ M
sM
s

(8.6.22)

donde el momento mximo TM ahora es el mismo, tanto para funcionamiento como


motor o como generador.
Para deslizamientos pequeos (s << sM ) esta expresin es aproximada por
2TM
s
(recta)
sM
y para deslizamientos grandes (s>>sM) por
T=

T = 2TM sM

1
s

(hiprbola)

(8.6.23)

(8.6.24)

Estas tres relaciones estn representadas grficamente en la figura 8.6.2 con sM =0,2
para el rango 0 s 1 en el que la mquina funciona como motor.
Para motores normalizados el momento mximo es de 2 a 2,5 veces el momento
nominal, por lo que en el rango de funcionamiento normal como motor (0 T Tn) la

8-179

Captulo 8 : mquina asincrnica

aproximacin (8.6.23), que reemplaza la curva por una recta, es perfectamente lcita y
origina la comparacin con la correspondiente caracterstica del motor de corriente
continua en conexin shunt.
Cuando el rotor de la mquina es impulsado a velocidades superiores a la sincrnica,
aplicndole un momento externo (negativo) a su eje, el deslizamiento se hace negativo
(8.3.2) y el momento electromagntico cambia de signo (8.6.22). Se invierte el sentido
del flujo de potencia (PCG1<0). La mquina pasa a ser un generador asincrnico
T
TM

Caracterstica T(s)
para R1=0 y sM=0.2

2.00

1.80

1.60

1.40

1.20

1.00

.80

.60

.40

.20

.00

s
.00

.20

.40

.60

.80

1.00

Figura 8.6.2 Caracterstica torque-deslizamiento y sus


aproximaciones.
Por otra parte, si con el rotor en movimiento se intercambia la conexin de dos fases a
la red, se cambia la secuencia de las corrientes y con ella cambia el sentido de giro del
campo giratorio, que ahora gira en sentido opuesto al sentido de giro del rotor. En esa
condicin el deslizamiento es mayor que uno (s>1) y la potencia mecnica se hace
negativa (8.6.9), mientras que la potencia del campo giratorio del estator permanece
positiva (8.6.6). La mquina pasa a ser un freno de contracorriente. Las potencias
mecnica y elctrica absorbidas por la mquina se convierten en calor en las
resistencias del rotor.

Escucho y olvido
Veo y recuerdo
Hago y comprendo
(Proverbio chino)

9. EJERCICIOS
Problema 1.1
En el circuito magntico de la figura se emplea chapa silicosa y fierro fundido, cuyas
caractersticas magnticas estn indicadas en la figura 1.3.3 de los apuntes.
chapa
silicosa
q=4cm2
l=30cm

N=300

fierro fundido
q=6cm2
l=6cm
entrehierro
1mm

a)
Determine la corriente continua necesaria para que en el entrehierro de 1mm se
establezca un campo cuya induccin sea igual a 0,9T.
Determine la induccin en el entrehierro si la corriente que excita el campo vale 10A.
Resolucin
a)
Anlisis preliminar
Se supone que el campo en el circuito magntico es homogneo y que la dispersin
magntica en el entrehierro es despreciable. De esa manera se conoce el flujo en el
entrehierro y en las otras secciones del circuito magntico y se puede determinar el
valor de la induccin B en cada seccin. Entrando con estos valores a la caracterstica
de magnetizacin del material correspondiente, se determina el valor de la intensidad
de campo H asociado. Con H conocido se aplica la ley de Ampere al circuito magntico,
lo que permite determinar la corriente buscada.
Desarrollo
0,9 6
La induccin en la chapa silicosa vale B1 =
= 1,35T , valor al que corresponde
4
segn la caracterstica H1=0,14.104A/m. En el tramo de fierro fundido la induccin es la
misma que en el entrehierro y de la caracterstica se obtiene el valor de la intensidad de
campo H2=0,85.104A/m. Para el entrehierro se logra H3=0,9/(4.10-7)=71,6.104A/m. La
fmm resultante vale
IN = Hi li = 10 4 (0,14 0,3 + 0,85 0,06 + 71,6 0,001) = 420 + 510 + 716 = 1646 A
i

En consecuencia I=1646/300=5,5A.

9-181

Ejercicios y Problemas

b)
Anlisis preliminar
Como slo se conoce la fmm total disponible IN=3000A, pero no su distribucin entre
los diferentes tramos del circuito magntico, es necesario resolver el problema
iterativamente: suponer un flujo inicial, calcular la fmm necesaria para el flujo supuesto y
compararla con la fmm disponible. La iteracin termina cuando la diferencia entre las
fmms es menor que una toterancia dada (p.ej.1%).
Escriba un programa de acuerdo con el siguiente esquema de iteracin:

Bi=/qi
Hi
Hi li

=+

=-

Hi li : IN

Problema 1.2
Sea el circuito magntico de la figura adjunta, con las dimensiones en cm indicadas en
la figura adjunta, sobre cuya columna central est dispuesto un devanado de 736
vueltas. La caracterstica magntica de las 171 chapas de 0,35 mm de espesor que
forman el ncleo est dada en la siguiente tabla:
Bmax T
1,2
Hef A/cm 1,8

1,3
2,3

1,4
3,8

1,5
7,6

1,6
14,8

1,7
26,0
18

a)
Si la rama derecha incluye un entrehierro de 0,1mm,
determine los flujos en las diferentes partes del circuito
magntico, si la induccin en la rama izquierda es de 1,6T.
Determine la corriente magnetizante para las condiciones de
saturacin del punto anterior.
Qu fraccin de la fmm se gasta en el entrehierro?

3
3

15

Problema 1.3
Sea el dispositivo cilndrico de la figura adjunta. La bobina diametral montada en el
cilindro interior puede pensarse de seccin transversal despreciable. La longitud axial
es de 6cm.
a) Determine la corriente necesaria para establecer en el entrehierro una induccin
media de 0,5T, considerando que la permeabilidad sea infinita.

9-182

Ejercicios y Problemas

b) Repita el punto a) considerando que el material es chapa silicosa. Haga


aproximaciones razonables.
3cm
0,5mm
10cm

200 vueltas

Problema 2.1
Para filtrar la corriente de salida de un rectificador monofsico de onda completa
alimentado desde una red de 50Hz se desea utilizar un reactor procedente de un avin
(110V, 400Hz). Se ha medido las prdidas en el fierro con induccin constante e igual a
la nominal, obteniendo: 60W a 400Hz y 4,5W a 50Hz.
Determine el valor efectivo que podra alcanzar la componente fundamental de la
tensin rectificada, sin que sean sobrepasadas las prdidas de fierro admisibles.
Problema 2.2
Sea un ncleo de permeabilidad infinita provisto de un entrehierro y de dos bobinas
caracterizadas por los parmetros R1=R2<<L1 y L1=L2 =L12.
Las dos bobinas estn conectadas en serie y la bobina 2 puede ser cortocircuitada
mediante un interruptor.
Si I es la corriente a travs de la bobina 1, I2 es la corriente a travs de la bobina 2 e I3
es la corriente a travs del interruptor, determine I2 e I3 en trminos de I cuando
a)
el interruptor est cerrado e I es una corriente alterna,
b)
el interruptor est cerrado e I es un corriente continua.
Justifique sus respuestas.
Problema 2.3
Sea un ncleo de chapas silicosas con permeabilidad fe = 500 0 cuya seccin
cuadrada mide 20cm2 y cuya longitud es de 50cm. El ncleo posee un entrehierro de
2mm y sobre l estn enrolladas dos bobinas de 280 y 40 vueltas respectivamente. A
los terminales de la bobina de 280 vueltas est conectado un condensador C y la
bobina de 40 vueltas est conectada, en serie con una resistencia de 5 a la fuente de
alimentacin.
a)
Determine el circuito equivalente para el dispositivo descrito, suponiendo que la
dispersin sea despreciable.
b)
Si la tensin de la fuente es v=35sen(1883 t) y el valor de la capacidad del
condensador es tal que la impedancia de entrada del circuito sea infinita, determine el
valor mximo de la induccin en el entrehierro.
c)
Determine el valor del condensador usado en b)

Ejercicios y Problemas

9-183

Problema 2.4
Sea un material de imn permanente con la caracterstica adjunta.
B/T
0
1,45 1,54 1,44 1,35 1,30 1,25 1,20 1,10 0,8 0
H/kA/m 60
70
80
40
0
-20 -40 -45 -50 -56 -60
a)
Para magnetizar a un imn de 5cm de longitud y seccin cuadrada de 10cm2 se
lo incorpora a un circuito magntico cerrado formado por fierro ideal, provisto de una
bobina de 100 vueltas. Si el flujo inicial es cero, determine el valor mximo de la
corriente, necesario para producir una induccin remanente de 1,35T.
b)
Determine en forma aproximada la energa requerida para magnetizar el imn en
las condiciones indicadas en a). La resistencia de la bobina sea despreciable.
c)
Determine el valor de la induccin para la cual el producto BH es mximo.
d)
Determine la seccin ptima de un imn de 50cm3 , incorporado a un circuito
magntico formado por fierro ideal y un entrehierro de 5cm2 y 2mm de longitud. Cul
es la induccin en el entrehierro?
Resolucin
a)
Anlisis preliminar
Para que el imn quede con la induccin de remanencia de 1,35T de la caracterstica,
es necesario magnetizarlo hasta el vrtice (en el primer cuadrante de la caracterstica
BH ), es decir, la fuerza magnetomotriz debe ser tal que H sea igual a 80000A/m.
Desarrollo
Aplicando la ley de Ampere a lo largo del circuito magntico, H i l i = i N , se tiene que
80000 0,05
i =
= 40 A .
100
b)
Anlisis preliminar
Como la curva virgen no est indicada, puede suponerse que la energa suministrada
por unidad de volumen corresponde aproximadamente al rea encerrada por el lazo de
histresis en el primer cuadrante. El valor as determinado es algo mayor que el valor
real. Como se trata de una primera aproximacin, el lazo de histresis puede ser
aproximado por un rectngulo del mismo rea.
Desarrollo
El rea es aproximadamente wm = 80000 14
, = 112000J / m 3 , y como el volumen es de
V = 0,1 0,05 = 0,005m 3 , la energa requerida, que en ausencia de prdidas es igual a la
energa acumulada, es W m = 112000 0,005 = 560J .
c)
Anlisis preliminar
Una primera aproximacin se logra al determinar el punto de interseccin de la
caracterstica en el segundo cuadrante con la diagonal del rectngulo BrHc y a partir de
este valor (1,2x45000=54000Ws) habra que tantear en forma sistemtica.
Alternativamente se puede proceder en forma grfica trazando y evaluando BH(B).
d)
Anlisis preliminar

9-184

Ejercicios y Problemas

Se sabe que para volmenes de imn y de entrehierro dados la induccin en el


entrehierro es mxima cuando el producto Bi Hi es mximo, por lo tanto, se determina Ba
a partir del producto determinado en c) y se calcula la seccin del imn a partir de la
condicin de continuidad del flujo:
B
qi = qa a .
Bi
Desarrollo
A partir de los valores de primera aproximacin del punto c) se tiene que
qi l i
50
Ba =
0 Bi H i =
4 10 7 54000 = 18
, T
qa l a
1
por lo que
18
,
qi = 5
= 7,5cm 2
12
,
Debido a que la seccin del entrehierro es menor que la del imn, la induccin en el
entrehierro es mayor que en el imn.
El clculo no incluye el efecto de la dispersin magntica en el entrehierro.
Problema 2.5
Sea un transformador con ncleo acorazado, con las dimensiones en cm
indicadas en la figura adjunta, sobre cuya columna central estn dispuestos dos
devanados de 172 y de 736 vueltas respectivamente. La caracterstica magntica a
corriente alterna de las 171 chapas de 0,35 mm de espesor est dada en la siguiente
tabla:
Bmax T
1,2
Hef A/cm 1,8
a)
b)

1,3
2,3

1,4
3,8

1,5
7,6

1,6
14,8

1,7
26,0

Si al devanado de172 vueltas se aplica una tensin


de 220V, 50Hz, determine la corriente magnetizante.
Determine la inductancia vista desde los terminales
de la bobina de 736 vueltas.

Problema 3.1
Un transformador monofsico de 220V, 50Hz posee un ncleo toroidal con un dimetro
interior de 10cm, un dimetro exterior de 18cm y una seccin cuadrada.18El ncleo est
armado de chapas silicosas de las siguientes caractersticas :
3
Bm
AVef /cm
Pfe

0,8
0,40
0,71

1,0
0,48
1,05

1,2
0,67
1,50

1,4
1,50
2,10

1,5
4,0
2,45

1,6
10,5
2,85

T
A/cm
W/kg

15

fe=7,6kg/dm3
El devanado de 220V es de 1 capa de 420 vueltas uniformemente enrolladas sobre el
ncleo.

Ejercicios y Problemas

9-185

a)
Determine los parmetros de la rama de magnetizacin del circuito equivalente
(Xm, Rfe)
b)
Determine la constante de tiempo de cortocircuito del transformador si con el
secundario cortocircuitado se midi en el primario 2,1A y 26,6W al aplicar una tensin
alterna de 14V y 50Hz.
Resolucin
a)
Anlisis preliminar
Se trata de un transformador toroidal, del tipo usado en el Laboratorio de Mediciones
Elctricas. La geometra y las caractersticas del ncleo estn dadas. La tensin alterna
impone el flujo por lo que se puede determinar la induccin mxima y con ella las
prdidas en el fierro y la corriente magnetizante, a partir de las cuales se determina los
parmetros de la rama de magnetizacin.
Desarrollo
Debido a que el enrollado encierra al flujo en el volumen del toroide, todas las vueltas
del devanado abrazan el mismo flujo, por lo que ste se puede calcular a partir de
V
220
m =
=
= 0,0024 Wb
4,44Nf 4,44 420 50
La seccin del ncleo vale
de d i 2
2
= ( 0,04) = 16 10 4 m 2
q fe =
2
Suponiendo campo homogneo en el interior del ncleo , la induccin se calcula como

0,0024
Bm = m =
= 15
, T
q fe 16 10 4
Las prdidas de fierro especficas correspondientes a este valor de induccin se
determinan mediante interpolacin lineal a partir de la caracterstica dada. En este caso
se obtiene directamente
W
p fe = 2,45
kg
A partir del volumen
d + di
0,18 + 0,10
= 16 10 4
= 7,04 10 4 m 3
Vol = q fe e

2
2
y el peso
kg
G fe = feVol = 7,6
0,704dm 3 = 5,35kg
dm 3
se calcula las prdidas de fierro en el ncleo como
Pfe = p feG fe = 2,45 5,35 = 13,1W
En el circuito equivalente estas prdidas estn representadas mediante una resistencia
a cuyos terminales est aplicada la tensin de 220V, por lo que el valor de esta
resistencia debe ser

Ejercicios y Problemas

9-186

V 2 220 2
=
= 3695
Pfe
13,1
Por otra parte, la corriente magnetizante se determina a partir de la Ley de Ampere,
utilizando el valor de H obtenido mediante interpolacin lineal de la caracterstica dada.
En el presente caso se obtiene directamente
A
Hfe = 4
cm
por lo que a partir de
Rfe =

I m N = Hfe l m
se calcula
400 0,18 + 0,10
Im =

= 0,42A
420
2
El valor de la reactancia correspondiente en el circuito equivalente se calcula como
V
220
Xm =
=
= 524
I m 0,42
b)
Anlisis preliminar
Con el secundario cortocircuitado y con el primario conectado a una fuente de tensin
reducida el flujo en el ncleo tambin se reduce fuertemente con lo que las prdidas en
el fierro se hacen despreciables frente a las prdidas en el cobre y la corriente
magnetizante se hace despreciable frente a la corriente de cortocircuito. El circuito
equivalente se reduce a un circuito serie RL.
Desarrollo
La impedancia de cortocircuito vale
V
14
Zcc = cc =
= 6,7
I cc 2,1
y , debido a la conexin serie de los elementos, la resistencia equivalente de los
devanados se calcula como
P
26,6
Re = cc2 =
= 6,03
2,12
I cc
A partir de estos valores se determina la reactancia equivalente como
X e = Zcc 2 Re 2 = 2,92.
La constante de tiempo de un circuito RL serie se define como
L
X
2,92
T= = e =
= 154
, ms
R Re 314 6,03
Si se aplicara una tensin escaln de 1V al transformador cortocircuitado, la corriente
crecera de acuerdo con la funcin
1
i (t ) =
( 1 e t 0,0015 ) A .
6,03
Nota:
Este problema fue parte del primer certamen 95 para el cual la nota media
fue de 41%. Las principales dificultades fueron :

Ejercicios y Problemas

9-187

Interpretacin de los datos (la seccin cuadrada caus estragos)


Relacin del aparato descrito con el modelo
Adaptacin y manejo del modelo
Incorporacin de conceptos vistos en otras asignaturas (constante de tiempo de
un circuito serie)
Problema 3.2
El ncleo del transformador del problema 3.1 se provee de un entrehierro de 1,5mm
a)
Determine el valor efectivo de la tensin alterna de 50Hz que debe aplicarse a la
bobina de 420 vueltas para que la corriente absorbida sea de 4A
b)
Si el conductor de cobre (=0,018mm2/m) es de seccin circular, determine la
resistencia a corriente continua de la bobina.(Justifique)
Problema 3.3
Un transformador monofsico de 100kVA, 13200/230V, 50Hz, fue sometido a un ensayo
en cortocircuito con corriente nominal, midindose 528V y 1590W. Ensayado en vaco
con tensin nominal se midi 4,5A y 318W.
a)
Determine los parmetros del circuito equivalente T y exprselos en (pu), en
referidos al devanado de alta tensin y en referidos al devanado de baja tensin.
b)
Para una carga de 80kVA, cos=0,8 cap., determine las corrientes en las ramas
del circuito equivalente y exprselas en (pu), en A referidas al lado de alta tensin y en
A referidas al lado de baja tensin.
c)
Exprese las prdidas nominales en (pu) y determine el rendimiento nominal del
transformador.
d)
Determine la regulacin del transformador para las condiciones indicadas en b).
e)
Se desea utilizar el transformador en una red de 60Hz y 13,2kV. Determine las
prdidas en el fierro nominales en esa condicin suponiendo que de las prdidas
medidas a 50Hz 2/3 corresponden a prdidas por histresis y 1/3 a prdidas por
corrientes parsitas.
Resolucin
a)
Anlisis preliminar
Se trata de un transformador de distribucin, por lo que la tensin de cortocircuito
debera ser una pequea fraccin (<10%) de la tensin nominal y la corriente de vaco
debera ser del orden de un 1% de la corriente nominal. En consecuencia se puede
concluir que las mediciones en vaco fueron realizadas en el lado de baja tensin y que
las mediciones en cortocircuito fueron realizadas en el lado de alta tensin. Del ensayo
en cortocircuito slo se puede obtener X y Re , por lo que deber aplicarse la regla:
R1=R 2=Re/2 y X 1=X2=X /2 , para obtener los parmetros del circuito equivalente T.
Desarrollo
Las corrientes nominales en los lados de alta y de baja tensin son respectivamente:
P
P
10010 3
10010 3
I 1n = n =
= 7,58 A
e
I2 n = n =
= 435A
V1n
13200
V2n
230
De las mediciones en cortocircuito se tiene que

9-188

Ejercicios y Problemas

z e = v cc =

528
= 0,040 (pu)
13200

y que re =

Pcu n
Pn

1590
= 0,016 (pu)
10 5

x = z e2 re2 = ( 0,04) 2 ( 0,016) 2 = 0,037 (pu).


Las impedancias base son:
V
13200
Z b 1 = 1n =
= 1741
en el lado de alta tensin y
I1n
7,58
V
230
Zb 2 = 2n =
= 0,53
en el lado de baja tensin.
I 2 n 435
Por lo tanto, las impedancias en son respectivamente:
Re = re Z b 1 = 0,016 1741 = 27,9
X = x Z b 1 = 0,037 1741 = 64,4
referidas al lado de alta tensin y
Re = re Z b 2 = 0,016 0,53 = 0,0085
X = x Z b 2 = 0,037 0,53 = 0,0196
referidas al lado de baja tensin.
Para los parmetros de la rama de excitacin se usar un procedimiento alternativo,
determinando primeramente los valores en referidos al lado de baja tensin:
V2
( 230) 2
Rfe = 2 n =
= 166
Pfe
318
Qo = S02 Pfe2 = ( 230 4,5) 2 ( 318) 2 = 985VA
V22n ( 230) 2
=
= 53,7
Q0
985
Referidos al lado de alta tensin los parmetros toman los valores:
Xm =

V
13200
Rfe = 166 1n = 166
= 547k
230
V2 n
2

2
V1n
13200
X m = 53,7
= 53,7
= 177k
230
V2n
En (pu) resulta:
R
166
rfe = fe =
= 313( pu )
Z b 0,53

x =

X m 53,7
=
= 101( pu )
Zb
0,53

b)
Anlisis preliminar
En la prctica las cargas se suelen dar en kVA y no en , asumiendo tcitamente que la
tensin aplicada es la nominal. No debe perderse de vista el objetivo del clculo

Ejercicios y Problemas

9-189

numrico, por lo que su precisin slo debe ser la mnima necesaria. Un resultado no
puede ser ms exacto que los datos que le sirvieron de base.
Desarrollo
La corriente en la carga se calcula como
80 10 3
Ic =
= 348 A
230
Ic = 348 a cos(0,8 ) = 348 36,9
La cada de tensin en el secundario es
Ic 21 (Re + j X ) = 348 36,9 21 (0,0085 + j 0,0196 ) = 4,129,7
y sumada a la tensin secundaria 230 0da como tensin inducida por el flujo comn
Vi = 233,6 0,5.
La corriente magnetizante vale
Vi
233,6 0,5
Im =
=
= 4,35 89,5
j Xm
53,7 90
La corriente de prdidas vale
V
233,6 0,5
Ife = i =
= 141
, 0,5
Rfe
166
La corriente de vaco vale
I0 = I fe + I m = 4,57 716
,
La corriente primaria, referida al secundario, vale
I1 = Ic + I 0 = 348 36,9 + 4,57 716
, = 352 37,3
Las corrientes referidas al lado de alta tensin se obtienen multiplicando a las corrientes
N
V
230
referidas al lado de baja tensin por 2 = 2 n =
= 0,0174 .
N1 V1n 13200
Las corrientes en (pu) se logran dividiendo las corrientes referidas al secundario por la
corriente base, es decir, I 2 n = 435 A .
c)
Anlisis preliminar
El rendimiento del transformador es el cuociente entre potencia entregada y potencia
absorbida
P
Putil
= util =
Pabs Putil + Pfe + Pcu
Desarrollo
318
Pfen =
= 0,0032 (pu)
100 10 3
1590
Pcu n =
= 0,0159 (pu)
100 10 3
1
n =
= 0,98 = 98% , suponiendo cos=1.
1 + 0,0032 + 0,0159
d)
Anlisis preliminar

Ejercicios y Problemas

9-190

Para la regulacin a plena carga se haba derivado la expresin


n = re cos + x sen .
Cuando la corriente es distinta a la corriente nominal se logra a partir de la definicin:
R I
X I
I
= e 2 cos + 2 sen = 2 ( re cos + x sen )
V20
V20
I 2n
Desarrollo
= 0,8( 0,016 0,8 + 0,037 0,6 ) = 0,028 = 2,8%
e)
Anlisis preliminar
Debido a la relacin V = 4,44 N f , el producto f debe permanecer constante,
por lo que el aumento de frecuencia implica una disminucin del flujo y por ende de la
induccin.
Las prdidas por histresis PH = CH fB 2 = CH B disminuyen linealmente con la
induccin, mientras que las prdidas por corrientes parsitas PF = CF ( fB) 2
permanecen constantes.
Desarrollo
B60 50
PH 60 B60 50
=
=
=
= 0,833
B50 60
PH 50 B50 60
PF 60 = PF 50 = 31 318 = 106W
PH 60 = 0.833 32 318 = 177W
Pfe60 = 106 + 177 = 283W
Problema 3.4
Un transformador monofsico de 50kVA, 1200/240V, 50Hz, vcc =3,6%, tiene prdidas de
cortocircuito nominales de 1,8% de la potencia nominal. El rendimiento nominal es de
97,9%. Determine:
a)
La regulacin mxima y el factor de la potencia de la carga para el cual se
produce.
b)
El rendimiento mximo y la corriente en la carga para la cual se produce.
c)
La corriente de cortocircuito en el lado de alta tensin, expresada en A y en
(pu) .
d)
La regulacin del transformador si en el lado de baja tensin se conecta una
impedancia de (0,65+j0,85).
Problema 4.1
Sea una mquina de rotor cilndrico de 0,2m de dimetro y 0,3m de longitud axial. El
entrehierro mide 0,6mm.
El estator est provisto de un devanado trifsico de 4 polos, 2 ranuras por polo y por
fase y 20 vueltas por bobina.
a)
Determine la inductancia mutua entre dos fases.
b)
Determine la corriente absorbida, si el estator conectado en estrella sin neutro es
alimentado desde una red monofsica de 220V, 50Hz, (una fase queda abierta).

Ejercicios y Problemas

9-191

c)
Si el rotor est provisto de un devanado monofsico de paso completo,
alimentado con 10A de corriente continua, que produce una distribucin espacial de
induccin triangular cuyo valor mximo es de 0,6T, determine el valor de la tensin
entre lneas inducida en el devanado del estator cuando el rotor gira a 1500rpm.
d)
Determine la tensin inducida en el devanado monofsico del rotor, si ste est
detenido y el estator est conectado a una red trifsica de 380V, 50Hz.
Resolucin
a)
Anlisis preliminar
Como se trata de una mquina de entrehierro constante, la inductancia mutua entre dos
fases del estator es constante. El hecho que las dos fases se encuentren en el estator
no cambia ni el razonamiento ni el resultado obtenido en el prrafo 4.4 de los apuntes.
Slo debe considerarse que el ngulo de desplazamiento entre las fases es de 120. El
devanado, al ser de una capa, est hecho con bobinas de paso completo, por lo que el
factor de cuerda vale fc =1.
Desarrollo
N1e f = N1 f d 1 = N1 f z 1
N1 = q 1 p Nbob = 2 2 20 = 80 vueltas por fase
sen(q 1 / 2) sen( 2 / 12)
fz 1 =
=
= 0,966
q 1 sen( / 2) 2 sen( / 12)
p 2

=
=
q1 2p m q1 m 6
N1 e f = 80 0,966 = 77,3
=

L12 =

4
R l
0
( N1e f ) 2 cos( 2 / 3) = 0,06 Hy
2

b)
Anlisis preliminar
La reactancia correspondiente a la conexin serie de dos fases se puede determinar, ya
sea a travs de la inductancia equivalente de dos bobinas acopladas inductivamente
Leq=L1+L2-2L12=3L1 , ya sea como inductancia propia de un devanado distribuido que
ocupa 2/3 de las 24 ranuras del estator. A la vista del resultado del punto a) en este
caso resulta ms directo el clculo de la inductancia equivalente.
Desarrollo
X 1 = 2 f 3L1 = 2 506L12 =113
V 220
I=
=
= 1,95A
X1 113
c)
Anlisis preliminar
La distribucin de induccin triangular producida por el rotor se compone de la
fundamental y de armnicas impares, de las cuales la ms fuerte es la tercera. Las
tensiones inducidas por la tercera armnica en las tres fases estn en fase entre s y
por lo tanto no aparecen en la tensin de lnea, que es la diferencia entre dos tensiones
de fase. En consecuencia basta con limitar el anlisis a la fundamental.

Ejercicios y Problemas

9-192

Desarrollo
La fundamental de la onda triangular vale
8
B1 = 2 Bmax =0,81 0,6 =0,485T

2
2 2R
p = B1 p l = B1
l = B1 R l =0,0146Wb

2p
p n 2 1500
f =
=
=50Hz
60
60
V = 3 4,44f Ne f p = 3 4,44 50 77,3 0,0146 =434V
d)
Anlisis preliminar
Si el rotor, con una corriente de 10A, produce una distribucin de induccin triangular
con valor mximo de 0,6T, se tiene que a partir de la relacin entre B y H y la ley de
Ampere
B
iN
H max = max =
,
0
2p
de donde se puede determinar el nmero de vueltas en serie del rotor. El ancho de
zona del devanado monofsico del rotor es q 22= 180 y el factor de zona, con
q 2 y 0 , es
sen(q 2 2 / 2) sen( / 2) 2
fz =

= ,
q 2 sen( 2 / 2)
/2

por lo que el nmero de vueltas efectivo es


Ne f = fz N ,
si se considera que las bobinas no estn acortadas.
Con el flujo por polo, impuesto por la tensin de la red, y el nmero de vueltas del
devanado del rotor se calcula la tensin inducida en ste.
Desarrollo
B 2 p 0,6 2 0,6 10 3 2
N2 =
=
=114
0 i
4 10 7 10
2
N2 e f = 114 =72,6

V1
380 3
p =
=
=0,0128Wb
4,44 f N1e f 4,44 50 77,3
V2 = 4,44 f N2 e f p = 4,44 50 72,6 0,0128 = 207V .
Problema 4.2
Sea una mquina doblemente cilndrica. El estator, de 12 ranuras, est provisto de un
devanado trifsico de dos polos, 2 capas, donde cada bobina posee 10 vueltas y un
paso igual a 83,3% del paso polar. El rotor, de igual nmero de ranuras que el estator,
est provisto de un devanado bifsico de dos polos, donde cada bobina es de 20
vueltas y tiene su paso acortado en un paso de ranura.

Ejercicios y Problemas

9-193

El rotor tiene una posicin tal que una fase del estator est alineada con una fase del
rotor.
a)
Si el devanado trifsico es excitado con un sistema de corrientes simtricas de
10A valor efectivo, determine el valor efectivo de un sistema de corrientes bifsicas que,
circulando en el devanado del rotor, anule el campo resultante en el entrehierro
b)
Cul es la relacin entre las tensiones de fase del estator y del rotor, inducidas
por el flujo en el entrehierro, si el estator est conectado a una red trifsica simtrica y
el rotor est abierto?
c)
Si el entrehierro es de 0,5mm, el dimetro interior del estator de 20cm y la
longitud axial del rotor de 15cm, determine las inductancias propias y mutuas entre los
diferentes devanados.
d)
cul es la corriente absorbida desde la red trifsica, si el devanado del rotor
est abierto y el valor medio de la induccin en el entrehierro es de 0,55T? Cul es el
valor efectivo de la tensin de 50Hz aplicada al devanado?
e)
Dibuje en forma desarrollada tanto el devanado del estator como el del rotor.
Resolucin
a)
Anlisis preliminar
El devanado trifsico produce una distribucin espacial de fmm cuya amplitud es igual a
3/2 veces la amplitud de la fmm producida por una fase. El devanado bifsico produce
una distribucin espacial de fmm cuya amplitud es igual a la de una fase.
Para que las distribuciones de fmm del estator y del rotor se anulen, sus amplitudes
deben ser iguales y estar en oposicin de fase
3
N I = Nr Ir
2 e e
Desarrollo
Z
2
Con q =
y
=
2p m
Z
el nmero de vueltas efectivo del devanado trifsico de 2 capas es
2
sen(2
)
sen(q/ 2)
12 2

N e =2 q N b e
cos( / 2) = 2 2 10
cos( ) = 37,3

q sen( / 2)
12
2 sen( )
12
El nmero de vueltas efectivo del devanado bifsico vale
3
sen( )
12 cos( ) = 105,6
Nr = 2 3 20

12
3 sen( )
12
En consecuencia la corriente bifsica equivalente vale
3 37,3
Ir =
10 = 5,3 A .
2 105,6
Ntese que un devanado trifsico siempre puede ser reemplazado por un
devanado bifsico equivalente (y viceversa).

Ejercicios y Problemas

9-194

b)
Anlisis preliminar
El valor efectivo de la tensin inducida por fase es, independientemente del nmero de
fases,
V =4,44 Ne f f p .
Como el flujo por polo y la frecuencia son los mismos para ambos devanados, la
relacin entre las tensiones inducidas es igual a la relacin entre el nmero de vueltas
efectivos por fase de los respectivos devanados.
Desarrollo
V2 N2 105,6
=
=
= 2,82
V3 N3
37,3
c)
Anlisis preliminar
Para la inductancia mutua entre dos devanados i y j rige
4
Rl
Li j = 0
Ne f i Ne f j cos( j i ) , expresin que tambin rige para las

p2
inductancias propias si se hace Ne f i = Ne f j
y i = j . Como todos los parmetros
son conocidos, el clculo es trivial.
Desarrollo
La inductancia propia de una fase del estator (fase a) vale
4
0,1 0,15
Le, a = 4 10 7
( 37,3 ) 2 = 66,8 mH
3

0,5 10 1
La inductancia mutua entre dos fases del estator vale
1
Le, a b = Le ,a cos(2 / 3)= Le, a = 33,4mH
2
La inductancia mutua entre las fases alineadas del estator y del rotor vale
Le r = 48 10 6 37,3 105,6 = 189mH
La inductancia propia de una fase del rotor (fase ) vale
Lr , = 48 10 6 (105,6) 2 = 535mH .
La inductancia mutua entre dos fases del rotor vale cero, pues estn desplazadas en
90 elctricos.
d)
Anlisis preliminar
Como la distribucin de la induccin en el entrehierro es sinusoidal, su valor mximo
vale Bmax = /2 Bmed . Suponiendo que la permeabilidad del fierro sea infinita, el valor
mximo de la fmm vale

Fmax = H max =
B
y es igual a 3/2 veces la amplitud de la onda de fmm
0 2 med
producida por una fase, es decir,
3 4 Ne 2 I
Fmax =
.
2p
2p

Ejercicios y Problemas

9-195

1 p
Bmed .
2 3 Ne 2 0
La tensin inducida en una fase del devanado trifsico es igual a la corriente por la
inductancia de campo giratorio
3
1 2 p
3 4
R l 2
V = I 2 f Le =
Bmed 2 f o
Ne
2
2
p2
2 6 0 Ne
V = 4,44 f p Ne
En consecuencia I =

Alternativamente podra determinarse primero la tensin inducida con


Rl
p = Bmed p l = Bmed
p
y calcular en seguida
V
I=
2 f 32 Le
Desarrollo
p = 0,55 0,1 0,15 = 0,026Wb
V = 4,44 50 37,3 0,026 = 215V
215
I=
= 68A
2 50 15
, 66,8 10 3
e)
Anlisis preliminar
Suponiendo que se trata de un devanado de ancho de zona igual a 60, cada fase del
12
devanado de dos capas del estator consta de 2 grupos de q =
=2 bobinas
23
desplazadas en 180 elctricos.
Cada fase est desplazada respecto a la que le antecede en 120 elctricos.
En el devanado del rotor el desplazamiento entre las dos fases es de 90 elctricos y el
12
nmero de ranuras por polo y por fase es q r =
=3 .
22
Problema 4.3
Un estator de 72 ranuras est provisto de un devanado trifsico de 8 polos formado por
bobinas acortadas de 10 vueltas y paso 7/9. Las bobinas correspondientes a cada fase
estn conectadas en serie y las tres fases estn conectadas en estrella. La tensin de
lnea es de 2598V, 50Hz.
Se requiere rebobinar este estator para 4 polos, manteniendo el acortamiento relativo,
la tensin de fase y la frecuencia.
Determine el nmero de vueltas que debe tener cada bobina del nuevo devanado para
que la induccin media en el entrehierro permanezca constante.
Problema 4.4
Una mquina de corriente continua posee un rotor de 8cm de dimetro y 3cm de
longitud axial, provisto de un devanado imbricado simple de 2 polos distribuido en 23

Ejercicios y Problemas

9-196

ranuras. El paso de las bobinas es de 11 ranuras (1-12) y la zapata polar cubre el 70%
del paso polar.
La induccin en el entrehierro sea de 1T bajo la zapata polar y de 0T en el espacio
entre los polos.
a)
Haga un dibujo en desarrollo del devanado indicando tambin la ubicacin de los
polos y la ubicacin de las escobillas sobre el conmutados. El ancho de una escobilla
sea igual al ancho de una delga.
b)
Determine el nmero de vueltas de una bobina del devanado, si la tensin
inducida entre los terminales de la armadura, cuando esta gira a 1500rpm, es de 48V.
c)
Determine la distribucin espacial de fmm producida por el devanado de
armadura, cuando por los terminales de esta circulan 10A.
d)
Repita los puntos a),b) y c) para el caso en que la mquina tenga cuatro polos y
bobinas de paso 5 (1-6).
Problema 5.1
Un imn permanente de 24cm de longitud y seccin circular de 12cm2 posee la
siguiente caracterstica B(H)
B(T)
0,60 0,52 0,40 0,18 0,0
H(A/cm)
0,0
-100 -200 -300 -350
El imn permanente tiene forma de U y el circuito magntico incluye dos entrehierros
iguales y un yugo de fierro ideal, que es mantenido en su posicin por un resorte que
ejerce una fuerza de 240N.
Determine la longitud de los entrehierros.
Anlisis preliminar
La fuerza del resorte es equilibrada por las fuerzas electromagnticas ejercidas por los
dos extremos del imn sobre el yugo. Este hecho permite calcular la tensin de Maxwell
y a partir de ella la induccin en el entrehierro. Conocida la induccin en el imn el
problema se reduce a resolver un circuito magntico serie.
Desarrollo
F
240
N
=
=
= 10 5 2
4
2q
2 12 10
m
Ba = 2 0 = 0,50 T
Suponiendo dispersin magntica despreciable, Ba = Bi , por lo que, de acuerdo con
Ampere, H i l i = 2Ha la .
l
Bi = 0 i H i .
2l a
Interpolando en la caracterstica con B i=0,50T se logra H i=1,17 104A/m y se calcula
Hi li
, 10 4 24
7 117
la = 0
= 4 10
= 3,5mm
Bi 2
0,5
2
Problema 5.2
Sea un dispositivo formado por dos cilindros concntricos de longitud axial 30mm. El
dimetro interior del cilindro exterior es de 40mm y el dimetro exterior del cilindro

Ejercicios y Problemas

9-197

interior (mvil) es de 36mm. Estator y rotor estn provistos de sendas bobinas


diametrales de 200 vueltas, que estn conectadas en serie.
Si un resorte espiral de rigidez 0,06Nm/rad desplaza los ejes magnticos de las dos
bobinas en 30 cuando la corriente por ellas es nula, determine el valor de la corriente
que causa un desplazamiento del rotor de 12 desde la posicin de equilibrio sin
corrientes.
Anlisis preliminar
El torque tiene su origen en la variacin de la inductancia mutua entre las dos bobinas y
, dada la conexin, depende cuadrticamente de la corriente.
La inductancia mutua tiene el valor mximo
Rl
L12 = 0
N1N2
2d
y vara linealmente con la posicin angular del rotor , alcanzando el valor 0 para =/2.
Desarrollo
Cuando el rotor se ha desplazado en 12 el momento vale

T = 0,06 12
= 0,0126 Nm
180
dL12( )
L
Te = i 2
= i 2 12 = 0,0143 i 2
d
2
i =

0,0126
= 0,094 A
0,0143

Problema 5.3
En un dispositivo formado por dos cilindros concntricos, separados por un entrehierro
de 0,5mm, estn dispuestas dos bobinas diametrales, una en el estator y la otra en el
rotor. El dimetro del rotor es de 20cm y su longitud axial es de 25cm. Cada bobina
posee 25 vueltas y una resistencia hmica de 0,2.
La bobina del rotor est cortocircuitada y la del estator est conectada a una fuente
alterna de 220V, 50Hz.
a) Determine la corriente absorbida en condiciones estacionarias por el dispositivo.
Si en el rotor se monta una segunda bobina, cortocircuitada y desplazada respecto a la
primera en 90,
b) determine el ngulo entre la primera bobina del rotor y la bobina del estator.
a)
Anlisis preliminar
El flujo alterno induce tensiones en la bobina cortocircuitada del rotor, en la que
circular corriente. En consecuencia se desarrollar un momento electromagntico que
girar el rotor a la posicin en que la energa del campo en el entrehierro sea mxima.
Esta corresponde a un ngulo de 90 entre las dos bobinas, para el cual la inductancia
mutua y, por lo tanto, la corriente en el rotor es nula.
En consecuencia, la corriente en el estator slo est determinada por la inductancia
propia de la bobina del estator.
Desarrollo

Ejercicios y Problemas

9-198

Rl 2
0,1 0,25
N1 = 4 10 7
25 2 = 61,7mH
2d
2 0,0005
V
220
I1 =
=
= 11,4 A
L1
314 61,7 10 3
b)
Anlisis preliminar
El flujo enlaza a las dos bobinas en sentido inverso, por lo que las respectivas
corrientes estn en oposicin de fase y los momentos tienen sentidos opuestos. La
posicin de equilibrio se produce cuando los momentos son iguales y opuestos, lo que
corresponde a la posicin en que los flujos enlazados son iguales y opuestos, es decir,
para 45.
L1 = 0

Problema 5.4
La curva de magnetizacin de un electroimn con un entrehierro de 2,5mm est dada
por la relacin =2,8i/(0,12+i). Una modificacin del entrehierro cambi esta relacin a
=2,8i/(0,10+i). Dibuje las caractersticas (i) y (i) en un mismo grfico hasta i=1,5A y
determine:
a) El aumento o disminucin del entrehierro. Haga suposiciones razonables.
b)La energa elctrica absorbida y el aumento o disminucin de la coenerga magntica
al variar el entrehierro manteniendo la corriente constante en 0,15A, es decir, en el valor
que tena cuando el entrehierro era de 2,5mm.
c) La fuerza media durante la variacin del entrehierro. El electroimn absorbe o
entrega energa mecnica?
Problema 5.5
Un dispositivo doblemente concntrico de entrehierro constante posee en el estator dos
bobinas diametrales a y b desplazadas relativamente en 90.
El rotor posee una bobina diametral f cuyo eje magntico est desplazado en relacin al
de la bobina a en el ngulo .
Si L aa=Lbb=0,35H , L af =2,0 cos H y Lff =20H,
a) Determine el torque desarrollado para =75, ia=7,07A, i b=-12,25A e i f =1,2A.
b) Si i f =1,2 cos(314 t) y las bobinas a y b estn cortcircuitadas, determine las
velocidades para las cuales el dispositivo desarrolla un momento medio distinto de cero.
c) Determine el momento medio desarrollado por el dispositivo si Ra=Rb=0 en las
condiciones indicadas bajo b).
Problema 5.6
Un condensador de placas paralelas ha sido cargado hasta una tensin V 0 y luego
desconectado de la fuente. Si ahora las placas se separan al triple de su separacin
original, determine:
a) La tensin entre las placas del condensador.
b) La energa acumulada en el campo elctrico despus del desplazamiento.
c) El trabajo mecnico realizado durante el desplazamiento relativo de las placas.

9-199

Ejercicios y Problemas

Problema 5.7
Sea una mquina de rotor cilndrico de 0,2m de dimetro y de 0,3m de longitud axial. El
entrehierro mide 0,6mm.
El estator est provisto de un devanado monofsico de 4 polos que produce una
distribucin espacial de induccin cuya fundamental posee una amplitud de 0,7T. El
devanado del rotor produce una distribucin de induccin similar, pero con amplitud de
0,5T.
a) Determine una expresin para el momento electromagntico debido a las
fundamentales como funcin del ngulo entre los ejes magnticos de los dos
devanados y evale su valor mximo.
b) Si el rotor gira a 1500rpm y es alimentado con corriente alterna de 20Hz, determine
las frecuencias que debera tener la corriente del estator para que el momento medio
sea distinto de cero.
c) Calcule el momento medio para la condicin determinada en b).
d) Calcule la amplitud y la frecuencia de los momentos oscilatorios que se producen
para las condiciones determinadas en b).
Problema 5.8
Un conductor de seccin rectangular de 1cmx5cm se encuentra en una ranura abierta,
practicada en fierro de permeabilidad infinita. Si por el conductor circula una corriente
alterna de valor efectivo 500A y frecuencia 5 Hz,
a) determine la distribucin de la energa magntica con la altura de la ranura y
b) calcule la fuerza por unidad de longitud sobre el conductor.
Problema 6.1
Un motor de corriente continua de excitacin independiente impulsa una carga inercial
que, en conjunto con el rotor del motor, posee un momento de inercia polar de
0,85kgm2. La resistencia de la armadura es de 0,5 y tanto la inductancia de la
armadura como las prdidas debidas a la rotacin sean despreciables.
Si el motor gira a 120rad/s con una tensin de armadura de 60V, determine la velocidad
del motor 1s despus de subir bruscamente la tensin de armadura a 80V, manteniendo
la corriente de campo constante.
Anlisis preliminar
Al producirse el salto en la tensin aplicada, la corriente de armadura ia=(V a-Gfq If )/Ra y
el momento electromagntico Te=GfqIf ia cambian bruscamente, acelerndose el motor.
La velocidad aumenta y con ella vrot=Gfq If , hasta que ia y Te se reduzcan a cero en la
nueva condicin de funcionamiento estacionaria.
Desarrollo
Inicialmente Va=V rot=iGfqIf , de donde se logra GfqIf =60V/120rad/s=0,5Vs/rad.
Posteriormente rige para la armadura
Va=vrot+Raia y para el rotor
Jd/dt=Gfq If ia .
Reemplazando la segunda ecuacin en la primera queda

G fq I f
d
+
dt
Ra J

VaG fq I f
Ra J

9-200

Ejercicios y Problemas

una ecuacin diferencial ordinaria, cuya solucin particular es inmediata


Va
80
rad
f =
=
= 160
y para cuya solucin homognea se postula una
G fq If
0,5
s
funcin exponencial. Se logra la solucin completa
Ra J
0,5 0,85
= ( i f )e t / T + f , donde T =
=
= 17
, s.
2
G fq I f
( 0,5) 2

Finalmente (1) = (120 160)e 1/1,7 + 160 = 137,8 rad s


Problema 6.2
Un motor shunt impulsa una carga inercial en condiciones estacionarias. Suponga que
las prdidas rotacionales y la inductancia de armadura sean despreciables y que el flujo
es proporcional a la corriente de campo. Derive la ecuacin diferencial para la velocidad
cuando la resistencia en serie con el campo shunt es cortocircuitada repentinamente.
Indique las condiciones iniciales.
Problema 6.3
Un motor de corriente continua con excitacin shunt de 7,5kW, 230V posee una
resistencia del circuito de armadura de 0,3 y una resistencia del campo shunt de
160. En vaco y con tensin nominal la velocidad es de 1200rpm y la corriente de
armadura es de 2,7A. A plena carga la corriente de armadura es de 38,4A y causa una
reduccin de flujo de 4% en relacin con el flujo en vaco. Determine la velocidad a
plena carga.
Anlisis preliminar
La tensin rotacional es proporcional al producto del flujo por la velocidad del rotor
Vrot=kpn , por lo que basta obtener la tensin rotacional para la situacin con carga
para determinar con el flujo correspondiente la velocidad en esa condicin.
Desarrollo
En vaco V rot=230-2,7 0,3=229,2V=kp 1200, de donde kp=0,191V/rpm.
A plena carga V rot=230-38,4 0,3=218,5V=k 0,96 p n1 , por lo que n1=1192rpm.
Problema 6.4
Un motor de corriente continua, conexin serie, opera conectado a una red de 230V y
gira a 750rpm absorbiendo 80A. Determine la velocidad para 230V y una corriente de
armadura de 20A para Ra=0,05 y Ra =0,20.
Problema 6.5
Un motor de corriente continua, conexin serie, de 120kW, 600V, 600rpm posee una
resistencia en el circuito de armadura de 0,16. La corriente nominal, a tensin y
velocidad nominales, es de 206A. La curva de magnetizacin a 400rpm es la siguiente
Vrot/V

375

400

425

450

475

If /A

188

216

250

290

333

Ejercicios y Problemas

9-201

Suponiendo que el efecto desmagnetizante de la reaccin de armadura puede ser


expresado en trminos de una fmm equivalente que vara con el cuadrado de la
corriente, determine el torque de partida si la corriente de partida est limitada a 350A.
Problema 6.6
Un motor de corriente continua para traccin posee la siguiente caracterstica de
excitacin a 1000rpm:
Vrot/V
285
349
450
517
546
Ff por polo/A
1800
2400
3600
4800
5400
Las resistencias de la armadura y del campo serie valen respectivamente 0,2 y 0,03.
La resistencia del campo shunt vale 160. El nmero de vueltas por polo del devanado
serie y del devanado shunt es respectivamente, 15 y 1100.
Determine la caracterstica torque velocidad del motor para una tensin de armadura de
525V y para corrientes de armadura de hasta 200A .
Problema 6.7
Una mquina de corriente continua de 6 polos, 900rpm, 550V, 275kW posee un
devanado ondulado distribuido en 180 ranuras, con 8 conductores por ranura. La zapata
polar cubre el 70% de un paso polar. Determine la fmm de reaccin de armadura por
polo para corriente nominal. Determine el nmero de conductores que debera tener un
devanado de compensacin adecuado. Estime el nmero de vueltas que debera tener
cada interpolo para establecer una induccin de 0,5T en un entrehierro de 5mm con
corriente nominal y devanado de compensacin.
Problema 6.8
Un pequeo motor universal (serie) desarrolla un momento de 2Nm con rotor detenido y
una corriente de armadura de 3A, corriente continua. La resistencia del circuito de
armadura es de 2,5 y la inductancia es de 0,04H. Suponiendo linealidad magntica y
prdidas por rotacin despreciables determine , si la mquina se conecta a una red de
115V, 60Hz:
El momento de arranque, la potencia mecnica desarrollada para una corriente de
armadura de 3A y el factor de potencia correspondiente. Dibuje el diagrama fasorial a
escala.
Anlisis preliminar
En el motor serie el momento es proporcional al cuadrado de la corriente de armadura,
por lo que el torque no cambia de sentido cuando lo hace la corriente. Con corriente
alterna el motor desarrolla un momento medio, proporcional al valor efectivo de la
corriente de armadura, al cual est superpuesto un momento pulsatorio cuya frecuencia
es igual al doble de la frecuencia de la corriente. La tensin rotacional est en fase con
la corriente de armadura. La cada de tensin en la reactancia es importante
(2fLa=15,1).
Desarrollo
Como T=GfqIa2 se tiene que Gfq=2/3 2=0,222H. La corriente de partida se obtiene de
115
Ia =
= 7,5A por lo que Tarr=(7,5)2 0,222=12,5Nm.
2
2
2,5 + 15,1

Ejercicios y Problemas

9-202

La ecuacin de equilibrio para la armadura es


115= (2,5+j15,1)Ia+GfqIa
de donde se logra (Pitgoras) para Ia =3A =147,5rad/s y n=1408rpm.
P=T=32 0,222 147,5=295W.
115
Ia =
= 2,76 + j118
, = 3 23,2
35,25 + j15,1
por lo que cos=0,92.
Problema 6.9
Para una mquina de corriente continua se ha obtenido la siguiente caracterstica de
magnetizacin a 1500rpm:
Vrot/V
10
40
80
135
172
199
220
If /A
0
0,5
1,0
2,0
3,0
4,0
5,0
La resistencia del campo shunt es de 44 y la resistencia de armadura es de 0,035.
La mquina posee un devanado serie aditivo, cuyo nmero de vueltas es igual al 0,5%
de las del devanado shunt. Para el funcionamiento como generador con una tensin de
vaco de 200V determine la tensin en los terminales cuando la corriente de armadura
es de 100A.
Si se desconecta el campo serie, cul ser la tensin en los terminales para una
corriente de armadura de 200A , con una tensin de vaco de 200V?
Problema 7.1
Un turbogenerador posee los siguientes valores nominales: P n=30MW, V n=10500V,
In =2350A, cosn=0,7, fn =50Hz, nn=3000rpm, n=97,8%, V fn=132V, Ifn=1400A.
Experimentalmente se determin SCR=0,54 y X 1 =0,18.
Dimensiones principales: D ext=1785mm, D int=840mm, l=1650mm, =45mm,
Z1=42ranuras.
Devanado del estator: bobinas de 2 vueltas con acortamiento 17/21.
Devanado del rotor: bobinas concntricas que cubren 2/3 de la periferia, N2=160vueltas.
a) Determine el entrehierro equivalente .
b) Determine la tensin fundamental inducida en una fase en vaco cuando la corriente
de campo est ajustada a 530A.
c) Dibuje el diagrama fasorial a escala (500V/cm, 500A/cm) para las condiciones
nominales indicando todas las tensiones y todas las corrientes.
d) Cul es la corriente reactiva mxima que puede entregar la mquina sin exceder
sus lmites trmicos?
e) Qu torque debe desarrollar la turbina en condiciones nominales?
f) Cul es el valor de la corriente de cortocircuito estacionaria, si ste se produce a
partir de las condiciones nominales?
g) Determine la corriente de campo necesaria para que la tensin inducida en vaco sea
la nominal.
h) Determine la corriente de campo necesaria para que la mquina desarrolle una
corriente de cortocircuito igual a la corriente nominal.
i) Determine el valor de la corriente de campo para que la mquina, conectada a una
red de tensin y frecuencia nominal, entregue corriente nominal con factor de potencia
unitario.

Ejercicios y Problemas

9-203

Resolucin
a)
Anlisis preliminar
A partir de la razn de cortocircuito, igual al valor recproco de la reactancia sincrnica,
y de la reactancia de dispersin se determina la reactancia correspondiente al campo
en el entrehierro y a partir de la expresin analtica para esta se calcula el entrehierro
equivalente.
Desarrollo
10500
, 0,18)
X 1G = X 1 X 1 = (185
= 4,31
3 2350
l
3 4
0,342
pero X1G =
0 p (N1 f d1 )2 =

2
2
p


sen 7

42
4

con f d1 =
cos = 0,913

42
7 sen
42
=0,08m=8cm !
b)
Anlisis preliminar
Las inductancias L 1G y L 1f comparten la misma permeancia y slo se diferencian en el
nmero de vueltas. El nmero de vueltas efectivo para el rotor se determina
considerando una distribucin de fmm trapezoidal, lo que equivale a considerar que q
tiende a infinito.
Desarrollo
2
Nf
2
160 0,827
L1f = L1G f df = 0,014
= 0,0486H
3
N1f d 1 3
28 0,913
sen(q / 2) sen(2 / 6 )
f df =
=
= 0. 827
q / 2
2 / 6
I
530
V p = 1L1f f = 314 0,0486
= 5684V
2
2
c)
d)
Anlisis preliminar
Las mquinas sincrnicas son trmicamente crticas en el rotor, lo que implica que la
limitacin est en la corriente de campo, que en rgimen permanente no debe superar
su valor nominal.
Desarrollo
1400
V pmax = 314 0,0486
= 15107V o 2,48 pu
2
Suponiendo carga reactiva pura, se tiene que

9-204

Ejercicios y Problemas

Vp max V1 = I1 X 1

de donde

I1 =

2,48 1
= 0,80 pu < 1
1,85

e)
Anlisis preliminar
La indicacin sobre la potencia nominal siempre se refiere a la potencia til, por lo que
en el caso del generador debe dividirse la potencia por el rendimiento para obtener la
potencia que entra por el eje.
Desarrollo
P
30 10 6
T= n =
= 97690 Nm
1 0,978 314
f)
Anlisis preliminar
El valor de Vp se determina del diagrama fasorial para carga nominal, por ejemplo,
mediante el teorema del coseno y con l y la reactancia sincrnica se calcula la
corriente de cortocircuito.
Desarrollo
Vp2 = 1 + 185
, 2 2 1 185
, cos(270 ) Vp = 2,66 pu
I cc =

Vp
X1

2,66
= 1,44pu 3376 A
1,85

g)
Desarrollo
10500
1
If = 2
= 562A
3 314 0,0486
h)
Anlisis preliminar
En cortocircuito la reaccin de armadura es desmagnetizante, por lo que se encuentra
la corriente de campo sumando gI1 a la corriente magnetizante Im, obtenida para
Vi=X 1 I1n de la caracterstica del entrehierro (sin saturacin).
Desarrollo
10500
Vi = X 1I 1n = 0,18
= 1091V
3
2V

0,08
Im = * i
con L*1f = L1f
= 0,0486
= 0,0864H
L1f 1

0,045
Im =

2 1091
= 56,9 A
0,0864 314

3 N1f d 1
3 28 0,913
=
= 0,41,
2 Nf f df
2 160 0,827
por lo que I f = I m + gI1 = 56,9 + 0,41 2350 = 1020 A
i)
Anlisis preliminar
El factor de reaccin de armadura vale g =

Ejercicios y Problemas

9-205

Del diagrama fasorial con factor de potencia unitario se desprende que el tringulo de
las tensiones es un tringulo rectngulo, lo que permite determinar Vp mediante el
teorema de Pitgoras. Para un grado de saturacin dado Vp e If estn relacionados por
la caracterstica lineal equivalente.
Desarrollo
Vp = 1 + 185
, 2 = 2,1pu . Suponiendo, por falta de la caracterstica de magnetizacin,
que la saturacin es igual que en g) se tiene que If =2,10x581=1222A.
Problema 7.2
Un motor sincrnico funciona conectado a una red infinita de tensin nominal
absorbiendo una corriente igual al 50% de su corriente nominal. La reactancia
sincrnica es de 2pu.
a) Al aumentar la corriente de campo en 10% se observ una disminucin de la
corriente de armadura. Antes del aumento de la excitacin, el motor absorba corriente
reactiva capacitiva o inductiva?
b) Determine el valor mnimo de la corriente de armadura alcanzable mediante ajuste de
la corriente de campo, si un aumento en 10% de la corriente de campo inicial determina
una disminucin de la corriente de armadura a 0,474pu.
Problema 7.3
Un generador sincrnico trifsico de 854kVA, 500V, 50Hz, 3000rpm requiere para
funcionamiento en vaco con tensin nominal una corriente de campo de 61A. En
cortocircuito produce 1500A con un corriente de campo de 83A,
a) Determine la corriente de campo necesaria para que, funcionando conectado a una
red infinita de tensin y frecuencia nominal, el generador entregue plena carga con
factor de potencia 0,8 inductivo(visto desde el generador).
b) Si en las condiciones de funcionamiento del punto a) el momento aplicado al eje se
reduce a cero, determine la corriente de armadura en mdulo y ngulo.
Problema 7.4
Un motor sincrnico trifsico de 1500kW, 4160V, 50Hz, factor de potencia nominal 0,9
capacitivo, 16 polos, posee una reactancia sincrnica de 1,15pu. Despreciando la
saturacin del circuito magntico, el efecto de las saliencias y las prdidas determine en
qu % debe variarse la corriente de excitacin - en relacin al valor para funcionamiento
nominal - si se desea ajustar el factor de potencia a la unidad, sin alterar la potencia
activa.
Problema 7.5
Un turbogenerador trifsico de 70,6MVA, 11,5kV, 50Hz, 3000rpm, factor de potencia
nominal 0,85, posee las siguientes caractersticas de vaco y de carga - con factor de
potencia cero y corriente nominal:

9-206

Ejercicios y Problemas

VACO
If
0,10 0,20 0,40 0,60 0,80 1,0
1,2
Vp
0,13 0,23 0,45 0,69 0,87 1,0
1,09
FACTOR DE POTENCIA CERO Y CORRIENTE NOMINAL
If
1,2
1,3
1,4
1,6
1,7
1,8
2,0
V1
0,015 0,13 0,25 0,49 0,61 0,69 0,83

1,4
1,15

1,6
1,21

pu
pu

2,2
0,92

2,4
0,99

pu
pu

Corriente de campo base = 350A


Tensin base = 11500V
a) Determine la reactancia de Potier
b) Determine la reactancia sincrnica nominal
c) Determine la corriente de campo para el punto de funcionamiento nominal.
d) Determine el lmite de estabilidad estacionario para las condiciones en c).
e) Determine el momento de inercia conjunto de generador y turbina, si la frecuencia de
oscilacin del rotor despus de un rechazo de carga es de 1,4Hz..
Problema 7.6
Un motor sincrnico de 32 polos, 3500kW, 4,16kV, 50Hz, factor de potencia nominal 0,9
capacitivo, posee una reactancia sincrnica de 1,25pu.
a)
Despreciando los efectos del entrehierro irregular, la saturacin y las prdidas,
determine para el motor conectado a una red de tensin y frecuencia nominal la
potencia que puede desarrollar con factor de potencia 0,8 , si las corrientes de campo y
de armadura no deben exceder sus respectivos valores nominales. (0,73pu)
b)
Si el motor arranca asincrnicamente y la corriente de campo se conecta cuando
la frecuencia de las tensiones inducidas en el rotor ha disminuido a 2Hz, determine la
velocidad del rotor (en r.p.m.) para el instante en que se conecta la corriente de campo.
(180rpm)
Problema 8.1
Sea un motor asincrnico trifsico de 225kW, 50Hz, p=8 cuyo rotor est equipado con
un devanado trifsico. La resistencia medida entre cada par de anillos rozantes es de
0,035. Con los anillos cortocircuitados el deslizamiento a plena carga es de 2,5%. Si el
motor impulsa a un ventilador, cuyo momento en el rango 0,5>s>0 es proporcional al
cuadrado de la velocidad, que demanda 225kW a la velocidad nominal del motor, qu
valor deben tener las resistencias externas conectadas en serie con las fases del rotor
para que la velocidad se reduzca a 300rpm?
Resolucin
Anlisis preliminar
Dado el tamao del motor, la resistencia del devanado del estator es despreciable.
Como el momento es igual o menor que el momento nominal y el deslizamiento final
3000 8 300
s=
= 0,20 es ,debido a la inclusin de resistencias externas,
3000
presumiblemente mucho menor que el correspondiente deslizamiento para momento
mximo, la caracterstica torque-velocidad puede ser reemplazada por su aproximacin
lineal. La inclusin de las resistencias externas cambia la pendiente de la recta y por lo

9-207

Ejercicios y Problemas

tanto el punto de interseccin de esta con la caracterstica del ventilador, que


corresponde al nuevo punto de trabajo.
Desarrollo
T
s
Con el rotor cortocircuitado se tiene que 1 = 1 , mientras que con resistencias
2TM s M 1
T
s
T
s sM 2
R
externas se tiene que 2 = 2 , por lo que 1 = 1
, pero s M = 2 , por lo que
2TM s M 2
T2 s 2 sM 1
Xe 2
sM 2
sM 1

R2,2
, ya que la reactancia permanece constante. En consecuencia
R2 ,1

R2, 2 T1 s 2 n12 s 2 3000 / 8 2 (375 300) / 375


=
=

=
= 12,5

R2,1 T2 s1 n 22 s1 300
0,025
0,035
R2,ad = R2, 2 R2,1 = 115
, R2, 1 = 115
,
= 0,201 .
2

Problema 8.2
Un motor asincrnico trifsico de 6 polos, 440V, 60Hz est provisto de un rotor con
devanado trifsico con el mismo nmero de vueltas efectivas que el del estator. Los
parmetros son R10, L 1 =3mH, L 2 =3mH, L m1=0,1H, R2 =0,3. Si el motor est
conectado a una red con tensin y frecuencia nominales, determine:
a) El momento de arranque con rotor cortocircuitado
(85,6Nm)
b) El momento mximo y la velocidad a la que se produce
(322Nm)
c) La velocidad, si la carga requiere un torque de 2Nm/rad/s.
d) Por qu el momento en la partida (s=1) no es igual al momento mximo, si en esas
condiciones la corriente absorbida s es mxima?
f) Si el motor es alimentado por el rotor con el devanado del estator cortocircuitado,
cul ser el sentido de giro del rotor en relacin con el del campo giratorio?
Problema 8.3
Un motor asincrnico trifsico tiene los siguientes datos: V n=380V, In=28A, fn=50Hz, 6
polos, conexin tringulo, rotor de anillos rozantes conectado en estrella, resistencia por
fase del estator=1,1.
El ensayo en vaco con tensin nominal arroj: I0=10,5A, P 0=1078W.
El ensayo con rotor detenido y corriente nominal arroj: V cc =88,7V, P cc =1470W.
Determine:
a) Los parmetros del circuito equivalente.
b) La velocidad nominal.
c) La capacidad de los condensadores que, conectados en tringulo, corrijan el factor
de potencia nominal a 0,9.
d) El % de aumento de la resistencia por fase del rotor para que la velocidad con carga
nominal se reduzca a 900rpm.

Ejercicios y Problemas

9-208

Problema 8.4
Los datos de placa de un motor asincrnico trifsico conectado en tringulo y con rotor
tipo jaula son: 22kW, 1460rpm, 500V, 50Hz, cos=0,85, rendimiento=89%.
Se conoce adems I(s=1)=5,6In y T(s=1)=2,2Tn.
Determine:
a) La corriente de arranque en pu y en A.
b) El momento de arranque en pu y Nm.
c) La potencia mecnica entregada a una carga que ofrece un momento igual al 25%
del momento nominal.
d) La frecuencia de las corrientes en rotor para las condiciones de funcionamiento
nominales.
Problema 8.5
Un motor asincrnico trifsico con rotor tipo jaula pesa 636kg y posee los siguientes
datos de placa: 55kW, 985rpm, 108A, Iarr/In=6,3, TM/Tn=2,4, cos=0,84, =0,95.
Suponga que las prdidas del estator sean despreciables y que la caracterstica
momento-velocidad tenga la forma derivada para el motor con rotor devanado.
a) Determine la potencia mecnica, la potencia del campo giratorio y la potencia
disipada en el rotor cuando se invierten dos fases de la alimentacin estando el motor
funcionando en vaco. Explique el origen de la potencia mecnica.
b) Si la mquina es impulsada a 1020rpm, determine la potencia del campo giratorio, la
potencia elctrica absorbida de la red, la potencia mecnica y la potencia disipada en el
rotor.
c) Determine el momento de arranque del motor.
Problema 8.6
Una mquina asincrnica trifsica de 22kW, 380V, 50Hz, 12 polos posee un rotor
provisto de un devanado tipo jaula de 44 barras, cada una con una resistencia de
0,002. El rotor gira a 508rpm.
Las prdidas en el fierro y las prdidas rotacionales sean despreciables.
Determine:
a) Las prdidas de cobre en el rotor.
b)
La velocidad del campo giratorio del rotor respecto al rotor.
c)
La velocidad del campo giratorio del estator respecto al rotor.
d)
La potencia del campo giratorio.
e) El diagrama de Sankey para este estado de funcionamiento.

Anda mungkin juga menyukai